You are on page 1of 287

F ARMAKOLOGJI 1.

Ne nje pneumoni nga Staphylococcus aureus meticiline-rezistent, antibiotiku i zgj edhur eshte: A) vanko micina B) oksacilina C) genta micina D) piperacilina E) k la ritr o m ic i n a

2. Medikamentet hipoglicemiante orale te klases se sulfanilureve: A) perdoren gjithmone si monoterapi B) stimuloj ne sekretimine e insulines dhe mund te shkakoj ne hipoglicemi te r e nd e C) indikohen ne diabetim mellitus tip 1 D) ndryshe nga biguanidet, nuk kane nevoje per pranine e qelizave beta pankreatike funksionuese E) mund te antagonizojne efektin hipoglicemiant te biguanideve

3. Te tere agj entet antidiabetike te meposhtem mund te shkaktoj ne drejteperdrejte hipoglicemi, me perjashtim te: A) glipizidit B) klorpropamidit C) t olbuta mid it D) m etform ines E) insulines Lantus

4. eshte A) B) C) D) E)

Tregoni se cili prej medika menteve antitumorale te meposhtme nuk antimetabolit: cisplatini metotreksati 5-fluorouracili citarabina 6-merkaptopurina

5. Cili prej medikamenteve te meposhtme mund te shkaktojne agranulocitoze te rende: A) klozapina B) nizatidina C) h a l op er i doli D) karboksicisteina E) kodeina

6. Cili prej medikamenteve te meposhtme mund te zhvilloje tolerance pas perdorimit te zgj atur: A) betabllokuesit B) statinat C) antidiabetiket orale D) benzodiazepina E) antagonistet e kalciumit

7. A) B) C) D) E)

Salbutamoli eshte: beta2-adrenergj ik stimulues beta-bllokues antiserotoninik antihistaminik antikolinergj ik

8. A) B) C) D) E)

Ne cilin rast mund te perdoret ne menyre te pershtatshme imipramina: pagjumesi epilepsi enureze noktune te adoleshentit glaukome mani

9. Inhibimi enzimatik: A) kerkon nje kontakt te perseritur ose te zgj atur te medika mentit me enzimen B) eshte nje fenomen me interes toksikologjik por jo terapeutik C) eshte nje fenomen pa asnje interes praktik D) percakton efektet farmakologjike dhe terapeutike te shume medikamenteve E) eshte gj ithmone i parikthyeshem

1 0. A) B) C) D) E)

Cili prej antibiotikeve te meposhtem eshte aktiv kundrejt haemophilusit: bacitracina teikoplanina cikloserina ampicilina penicilina G

1 1. A) B) C) D) E)

Tregoni cili eshte avantazhi i rruges se administrimit sublingual: ben te mundur nje kontroll te sakte te posologjise absorbimi nuk eshte i shpejte medikamentet nuk i nenshtrohen metabolizmit gj ate kalimit te pare ka risk te ulet per efekte te padeshiruara lehteson absorbimin e medikamenteve shume pak liposolubel

1 2. Mund te pritet nefrotoksicitet nga te tere medikamentet e meposhtme, me perj ashtim te: A) amfotericines B B) cisplatinit C) gentamicines D) amoksicilines E) vankomicines

1 3. Kortikosteroidet indikohen ne te tera patologjite e meposhtme, me perjashtim te: A) herpes simplex okular B) statusit astmatik C) sindromes nefrosike D) kolagjenopative E) insuficience renale kronike

1 4. A) B) C) D) E)

Cili eshte efekti i padeshiruara me i rende i klozapines: diskinezia tardive k on stipa ci oni shenja ektrapiramidale agranulocitoza aritmite

1 5. Te tere medikamentet e meposhtme j ane agoniste beta-adrenergjike, me perjashtim te: A) adrenalines B) izoproterenolit C) noradrenalines D) fentolam ines E) dobuta m ines

1 6. Aktualisht ne kardiopatine ishemike perdoren te tere medikamentet e meposhtme, me perjashtim te: A) kortizonikeve B) nitroderivateve C) beta-bllokuesve D) A C E - in i b i t o r e v e h E) antiagreganteve trombocitare

1 7. Tregoni se cili prej efekteve te meposhtme nuk induktohet nga g lukokor tik oi det: A) shperndarja e alteruar e indit adipoz B) rritja e sintezes se proteinave ne nivel kutan C) inhibimi i sintezes se leukotrieneve D) rritja e glicemise E) i m u n o su presio ni 1 8. Cili prej diuretikeve te meposhtme eshte me i pershtatshmi per traj timin e hiperkalciurise: A) furosemidi B) triamtereni C) manitoli D) acetazolamidi E) hidroklortiazidi

1 9.

Cili prej medikame nteve te meposhtme eshte nje beta-bllokues selektiv:

A) B) C) D) E)

praktololi atenololi sotaloli propranololi pindololi

2 0. Farmakovigjilenca eshte: A) observatori kombetar per monitorimin e shpenzimit farmaceutik B) faza IV e eksperimentimit klinik te medikamenteve (faza postm a r k e ti n g ) C) vleresimi i efikasitetit klinik te medikamenteve te reja D) enti europian pergjegjes per autorizimin e perdorimit klinik te medikamenteve te rej a C) faza e fundit e eksperimentimit preklinik te medikamenteve

2 1. renal: A) B) C) D) E)

Cili prej medikamenteve te meposhtme eshte pergjegjes per toksicitet bikarbonati i natriumit cimetidina ciklosporina azitromicina diazepami

2 2. A) B) C) D) E)

Suksinilkolina perdoret si: ganglioplegjik vagolitik bllokues i receptoreve alfa-adrenergjike bllokues neuromuskulor bllokues i receptoreve beta-adrenergjike

2 3. A) B) C) D) E)

Amiloridi eshte: nje diuretik i anses nje kursyes i kaliumit nje diuretik tiazidik nje inhibitor i anidrazes karbonike nje diuretik osmotik

2 4. Ne hipertiroidizem perdoren te tere medikamentet e meposhtme, me perjashtim te: A) metimazolit

B) C) D) E)

propiltiouracilit karbamazepines karbimazolit j odit

2 5. A) B) C) D) E)

Gj ysmej eta (t) e nje medikamenti eshte: ne perpjesetim te drejte me klirensin total ne perpjesetim te zhdrejte me vellimin e shperndarjes ne perpjesetim te drejte me vellimin e shperndarjes varet nga biodisponueshmeria mund te reduktohet ne dozat toksike

2 6. A) B) C) D) E)

Te nje pacient me hipoakusi dhe/ose vertigo nuk duhet te j epen: penicilina cefalosporina a m inoglukozide karbapenike m akrolide

2 7. Te nje pacient qe merr simvastatine, marrja e nj ekohshme e klaritromicines mund te shkaktoje: A) gastralgji B) shfaqjen e miopatise dhe te rhabdomiolizes C) rregullime te akomodimit D) cefale E) vertigo

2 8. A) B) C) D) E)

Pacientit me astme nuk i jepet: salbuterol digoksine propranolol bretil lidokaine

2 9. A) B) C) D) E)

Nepermj et cilit prej mekanizmave vepron metformina: rrit numrin e receptoreve te insulines rrit kapjen e glukozes nga indet rrit lirimin e insulines nga pankreasi rrit lipolizen dhe glikogj enolizen rrit prodhimin e acidit laktik

3 0. A) B) C) D) E)

P r o p ilti o u r a c ili: inhibon dejodizimin e T 4 stimulon formimin e hormonit tiroidien shkakton reduktimin e vellimit te tiroides inhibon transportin aktiv te j odit shkakton nje grumbullim j odi ne tireoglobuline

3 1. A) B) C) D) E)

Adrenalina shtohet ne anesteziket lokale per: te nxitur hemostazen te stimuluar riparimin lokal te indeve te vonuar absorbimin e anestezikeve lokale te lehtesuar penetrimin e tyre pergj ate degeve nervore te fuqizuar aksionin bllokues te kanaleve te Na+

3 2. Cili prej medikame nteve te meposhtme perdoret ne menyre me te pershtatshme ne diabetin insipid: A) furosemidi B) tria mtereni C) manitoli D) acetazola m idi E) h i d r o k l o rtia z i d i

3 3. A) B) C) D) E)

Cili prej FANS te meposhtem ka potencialin me te ulet gastrotoksik: acidi acetil salicilik diklofenaku indometacina paracetamoli s u li n d a k u

3 4. Cili prej medikamenteve perdoret per profilaksine e semundjes se me mbranave hialine: A) insulina B) kortikosteroidet C) hormonet e tiroides D) androgjenet E) beta2-stimuluesit

3 5. Neuroleptiket tipike (haloperidoli, klorpromazina, etj) mund te provokoj ne reaksinet ekstrapiramidale si: A) sindroma e ngj ashme me Parkinsonin B) distonite akute (spazme te gj uhes, grimasa, torticollis) C) diskinezia tardive (levizje stereotipike dhe te pavullnetshme) D) sindroma malinje nga neuroleptiket (e rralla, me katatoni, temperature te larte, hipertension) E) te tera pergjigjet jane te sakta

3 6. Te tera patologj ite e meposhtme jane indikacion per dhenien e j odit, me perjashtim te: A) kontaminimit aksidental me izotope radioaktive B) krizes tireotoksike C) tiroiditit Hashimoto D) pergatitj es per nderhyrje kirurgjikale ne tiroide E) strumes endemike

3 7. A) B) C) D) E)

Ne rast shoku anafilaktik duhet te j epet menj ehere: kardiotonike adrenaline dhe kortizonike diuretike dopamine dhe antihistaminike a n si o liti k e

3 8. A) B) C) D) E)

Shenj estra molekulare e nje medikamenti j ane: receptoret enzimat acidet nukleike molekulat e transportit (kotransportuesit dhe antitransportuesit) te gj itha pergj igj et jane te sakta

3 9. Mekanizmi i veprimit te medikamenteve antipsikotike konsiston ne bllokimin e siteve receptoriale te neurotransmetuesit: A) hista m ine B) dopa mine C) acetilkoline D) adrenaline E) GABA

4 0.

Cila cefalosporine ka gjysmejeten me te gjate:

A) B) C) D) E)

cefamandoli cefsulodina ceftriaksoni cefonicidi cefatrizina

4 1. A) B) C) D) E)

Medikamentet e meposhtme jane antituberkulare, me perjashtim te: izoniazidit rifampicines e tamb utolit azitromicines rifabutines

4 2. Te nje pacient me astme bronkiale ose SPOK, per anginen pectoris duhet te perdoret: A) nje beta-bllokues B) verapam ili C) warfarina D) aspirina E) a m lo dipina

4 3. A) B) C) D) E)

Toksiciteti kardiak kufizon perdorimin klinik te: aktinomicines D doksorubicines bleomicines cisplatinit amfetaminave

4 4. Cili prej medikame nteve antidiarreike te meposhtem shkakton me pak efekte kolaterale te lidhura me sistemin nervor qendror: A) kodeina B) loperamidi C) morfina D) difenoksilati E) parakodeina

4 5. A) B)

Cili antibiotik perdoret ne rast meningiti nga streptococcus pneumoniae: oksitetraciklina fosfomicina

C) D) E)

gentamicina cefotaksime klindamicina

4 6. Te tere faktoret e meposhtem ndikojne mbi transportin placentar te nje medikamenti, me perjashtim te: A) fluksi placentar te gj akut B) peshes molekulare C) hidrosolubilitetit te medikamentit D) liposolubilitetit te medikamentit E) pH fetal

4 7. A) B) C) D) E)

Efekti kolateral kryesor i terapise me hekur eshte/jane: rregullimet gastrointestinale: kolla h i pe rt en sioni pergjumj a alterimet e diurezes

4 8. A) B) C) D) E)

Ne terapine e dhembjes kronike, udhezuesit indikojne: morfine me rruge subkutane morfine me rruge orale b up r enorf i n e sekuencen FANS opioide te dobeta opioide te forta nje FANS

4 9. Te tere medikamentet e meposhtme j ane potencialisht nefrotoksike, me perjashtim te: A) streptom icines B) amfotericines B C) cisplatinit D) vankomicines E) cip r o flo k s a cin e s

5 0. Cili prej medikamenteve antihipertensive te meposhtme mund te shkaktoje kolle: A) antagonistet alfa1-adrenergjike B) sartanet C) A C E-inhibitoret D) antagonistet e kalciumit

E)

nitroderivatet

5 1. Tregoni se cila prej patologjive te meposhtme eshte kundraindikacion per perdorimin e metoklopramidit: A) diabeti B) patologjite anksioze C) nauzeja dhe te vjellat nga kemioterapia D) morbus Parkinson E) artriti reumatoid

5 2. Cili eshte antipiretiku i zgj edhur per nje sindrome febrile me natyre te mundshme virale te nje femije 5 vje: A) acidi acetil salicilik B) piroksika m i C) fenilbutazoni D) celekoksibi E) paraceta moli

5 3. A) B) C) D) E)

Nitro glice ri na: dilaton kryesisht arteriolat dilaton kryesisht venat rrit pasngarkesen nuk provokon tolerance nuk duhet te perdoret se bashku me nje beta-bllokues

5 4. A) B) C) D) E)

Acidi klavulanik eshte i rendesishem sepse: penetron lehtesisht te Gram+ eshte specifik per Gram+ eshte nje inhibitor i transpeptidazes se paretit qelizor inaktivizon beta-laktamazat bakteriale ka nje spekter veprimi te ngj ashem me ate te penicilines

5 5. A) B) C) D) E)

Cili efekt kolateral eshte karakteristik ne terapine me a mpiciline: k o ns tipacion i hiponatremia ekzantema makulopapulare anemia hemolitike nefriti

5 6. Qe te shfaqet efekti terapeutik i antidepresiveve triciklike, eshte e nevoj shme qe nga fillimi i marrj es se medikamentit te kaloj ne: A) 3 minuta B) 3 ore C) 1 di t e D) 2-3 jave E) 2 muaj

5 7. Nje i moshuar me diabet insulino-vartes duhet te marre nje trajtim me beta-bllokues per shkak te hipertensionit dhe te angines. Cili mund te jete medikamenti me i pranueshem: A) propranololi B) pindololi C) atenololi D) nadololi E) timololi

5 8. A) B) C) D) E)

Nje prej penicilinave te meposhtme eshte betalaktamaze-rezistent. Cila: a m oksicilina meticilina piperacilina tikarcinilina feneticilina

5 9. Cili prej medikame nteve te meposhtme eshte barbiturik qe perdoret si antiep il ep tik: A) pe nt b a r bitali o B) tiopentali C) f enob a r bitali D) diazepa m i E) sekobarbitali

6 0. Medikamente me te efektshme per reduktimin e perqendrimeve plazmatike te triglicerideve jane: A) s t a t i na t B) probukoli C) fibratet D) rezinat sekuestruese te acideve biliare E) inhibitoret e lipazes pankreatike

6 1. A) B) C) D) E)

Medikamenti me aktiv per nderprerj en e nje ataku hemikranie eshte: propranololi m etisergidi klo nidina triptanet amitriptilina

6 2. A) B) C) D) E)

Induktimi enzimatik: pershpejton metabolizmin e medikamente ve ndodh brenda nje kohe te shkurter pas marrj es se substances induktuese eshte nje efekt qe shkaktohet vetem pas marrjes se medikamenteve nuk shkakton modifikime te dozes asnj era

6 3. A) B) C) D) E)

Neuroleptiket jane medika mente qe indikohen ne trajtimin e: pagjumesise hipertensionit manise akute konvulsioneve dhembjes

6 4. Pacientet me ulere peptike duhet ti evitoj ne te tere medika mentet e meposhtme, me perjashtim te: A) kafeines B) F A NS C) nikotines D) prednizonit E) m etoklopramidit

6 5. A) B) C) D) E)

Efekti maksimal (Emax) qe arrin nje medikament eshte nje matje e: fuqise efikasitetit pergjigjes kuantale madhesise se antagonistit treguesit terapeutik

6 6. A) B)

Kontraceptivet hormonale femerore mund te provokoj ne zhvillimin e: adenomes hepatike kisteve te ovarit

C) D) E)

karcinomes se ovarit gastritit glakomes

6 7. A) B) C) D) E)

Ne intoksikimin nga substancat opioide, pupilat jane: mi ot ike midriatike anizikorike h i pe rfleksik e te ndermjetme

6 8. A) B) C) D) E)

Nje perdorim klinik i benzodiazepinave eshte: morbus Parkinson glaukoma cefalea pagjumesia ulera peptike

6 9. A) B) C) D) E)

Medikamentet gjenerike kane keto karakteristika, me perjashtim te: prodhohen me rruge industriale jane te dobishme per patologji multiple pasi jane jospecifike jane bioekuivalente per sa i perket specialitetit te caktuar nuk jane te mbrojtura me patente nuk j epen detyrimisht me recete

7 0. Cili prej medikamenteve perdoret per trajtimin e infeksione fungale siste m ike: A) ciprofloksacina B) anfotericina B C) penicilina D) amiodaroni E) streptomicina

7 1. Cili prej te meposhtmeve eshte nje indikacion klinik i medikamenteve antihistaminike H2? A) astma bronkiale B) ulera peptike C) ankthi D) diarrea E) riniti sezonal

7 2. Studimi i faktoreve qe percaktoj ne luhatjet e perqendrimit plazmatik te nje medikamenti quhet: A) far m akodina m ika B) mbikeqyrja farmakoceutike C) farmakogjenetika D) far m akokinetike E) farmakovigj ilenca

7 3. Cili prej te meposhtmeve eshte nje indikacion klinik i medikamenteve antihistaminike H1: A) konvulsionet B) ulera gastrike C) ankthi D) diarrea E) riniti sezonal

7 4. Cili prej medikamenteve te meposhtme mund te perdoret per traj timin e tumoreve te gjirit: A) ta m oksifeni B) propranololi C) p r ednizolo ni D) vitamina D E) nizatidina

7 5. A) B) C) D) E)

Metabolizmi i kalimit te pare: rrit biodisponueshmerine e medikamentit ka nje rendesi metabolike te pakte rrit gj ithmone toksicitetin e medikamentit redukton biodisponueshmerin e medikamentit redukton gj ithmone toksicitetin e medikamentit

7 6. Cili prej medikamenteve te meposhtme eshte aktiv ne infeksionet nga pseudomonas aeruginosa, por mund te shkaktoje ototoksicitet dhe n efrotoks icit et: A) karbenicilina B) genta micina C) kloramfenikoli D) r if ampic in a E) eritromicina

7 7. Me fare mund te antagonizohet depresioni respirator i induktuar nga morfina: A) buprenorfine B) pentazocine C) nalokson D) metadon E) kodeine

7 8. Cili prej efekteve kolaterale eshte i mundshem gjate terapise me k arbe nicil ine: A) nefrotoksiciteti B) disfunksioni trombocitar C) ototoksiciteti D) mielodepresioni E) hiperkal e mi a

7 9. A) B) C) D) E)

Glukokortikoidet nderveproj ne me: kanalet j onike tirozine kinazen proteinat G receptoret brendaqelizore lipidet

8 0. A) B) C) D) E)

Cili prej medikamenteve te meposhtme mund te shkaktoje vartesi: kalcitonina benzodiazepinat b e tabl lokuesit antikoagulantet orale an tihista m iniket

8 1. A) B) C) D) E)

Imipenemi eshte: nje inhibitor i beta-laktamazes nje aminoglukozid nje antifungal nje makrolid nje antibiotik beta-laktamik

8 2. Cili prej medikame nteve te meposhtme antagonizon efektin bronkodilatator dhe vazodilatator te izoproterenolit: A) kokaina B) feoksibenzam ina C) f e nt ola mina D) propranololi E) reserpina

8 3. Ne te tera semundjet e meposhtme ne te ardhmen mund te perdoret terapia gj enike, me perjashtim te: A) fibrozes kistike B) h emof ilis e C) uleres peptike D) tumoreve E) AIDS

8 4. A) B) C) D) E)

Cili prej medikamenteve te meposhtme eshte analgjezik: h a lope ri do li pentazocina k a n a m icin a adrenalina suksinilkolina

8 5. te: A) B) C) D) E)

Te gj ithe medikamentet e meposhtme j ane beta-stimulues, me perjashtim salbutamolit orciprenalines iz oprena line s propranololit f o r m e t o r o lit

8 6. A) B) C) D) E)

Cili prej FANS perdoret si antiagregant trombocitar: paraceta m oli nim esulidi nabumetoni aspirina ibuprofeni

8 7. Kur deshirohet te arrihet dilatacioni pupilar por jo cikloplegjia, medikamenti i zgjedhur eshte:

A) B) C) D) E)

o m atropina p ilo ka rpin a izoproterenoli tropikam idi fenilefrina

8 8. Ne cilin prej indeve te meposhtme shperndahen pergjithesisht me ngadale medikamentet: A) m ushkeri B) meli C) ind kockor D) tiroide E) muskul skeletik

8 9. A) B) C) D) E)

Cili prej te meposhtmeve nuk eshte nje neurotransmetues: acetilkolina noradrenalina penicilina dopa m ina 5-hidroksitriptamina

9 0. A) B) C) D) E)

Amiodaroni eshte nje medikament: hipnoinduktor antiacidik antiaritmik beta-mimetik beta-bllokues

9 1. A) B) C) D) E)

Eliminimi i penicilines G ndodh: kryesisht nepermj et sekretimit tubular kryesisht nepermj et filtrimit glomerular kryesisht me rruge biliare me filtrim glomerular dhe sekretim tubular ne raport pothuajse te njejte me rruge pulmonare

9 2. Nje reaksion alergjik ndaj nje medikamenti: A) zakonisht eshte ne varesi te dozes B) varet nga doza vetem kur medikamenti jepet me rruge sistemike por j o kur jepet me rruge lokale C) ndermj etesohet gjithmone nga prodhimi i antitrupave

D) E)

ka gj ithmone gravitet te lehte ose te moderuar induktohet betem nga medikamentet e sintezes

9 3. A) B) C) D) E)

Sekretimi i acidit gastrik mund te ndikohet nga: antagonistet e receptorit H2 inhibitoret e pompes protonike antagonistet muskarinike, p.sh. pirenzepina te tera pergjigjet jane te sakta alkooli

9 4. Te tera efektet e meposhtme jane manifestime te mbidozimit te barbiturateve, me perj ashtim te: A) eksitimit B) konfuzionit mendor C) letargjise D) hipoterm ise E) hipote nsion it

9 5. A) B) C) D) E)

Te nje pacient me glaukome kundraindikohet perdorimi i: barbiturateve: antidepresiveve triciklike antibiotikeve benzodiazepinave hipnogj ene a m feta m in a v e

9 6. A) B) C) D) E)

Cila prej klasave te meposhtme inhibon HMG-CoA reduktazen: s t a t i na t beta-bllokuesit fibratet estrogj enet ACE-inhibitoret

9 7. A) B) C) D) E)

Cila prej substancave mund te shkaktoje hepatit toksik: trikloretileni tetrakloruri i karbonit halotani izoniazidi te gj itha pergj igj et jane te sakta

9 8. Terapia me estrogj ene shoqerohet me te tera efektet e meposhtme te padeshirueshme, me perj ashtim te: A) hiperplazise endometriale B) nauzese C) dhembjeve te gj injve D) hipoglice m ise E) h e m ikra nias

9 9. A) B) C) D) E)

Efektet e padeshirueshme me te rendesishme te fenotiazines jane: cefalea paraliza spastike korea fotofobia rregullimet ekstrapiramidale

100. A) B) C) D) E)

Kokaina ka veprim te tipit: sim patikolitik m iorela ksa nt parasimpatikomimetik anestetik lokal spazmolitik

HEMA TOLO GJI A 1. Nje dj ale 25 vj e veren, ndersa po rruhej, shfaqjen e nje zmadhimi linfonodal ne nivelin laterocervikal. Nuk ankon asnje simptome. Ne egza minimin obj ektiv pervec linfonodulit qe ka pare djali j ane prezent edhe linfonoduj te tjere ne te njejtin regj ion. Realizohet nje biopsi dhe diagnoza histologjike flet per Morbus Hodgkin,tipi sklerose nodulare. Djali i nenshtrohet nje stadifikimi te thelluar te semundj es se tij perfshire edhe nje laparotomi, dhe nga te gjitha hetimet e kryera rezulton se ka linfonoduj prezente edhe ne nivel te mediastinit , por jo ne abdomen, dhe shpretka eshte normale. Cili eshte stadi i semundjes se pacientit? A) I B) IE C) II D) III E) IV

2. Kriter kryesor per diagnozen e krizes blastike gj ate ecurise se leukozes mieloide kronike eshte prezenca e : A) blastet ne palcen e kockes mbi 30%

B) C) D) E)

hiperurice m ia reduktimi i numrit te rruazave te kuqe limfoblastoza ne palcen e kockes limfocitoza periferike

3. Nje pacient diabetic 50 vj e shtrohet per shkak te shfaqjes se nje krize hipoglicemike. Gj ate ecurise evidentohet nje anemi e shkalles se moderuar me nje trombocitoze te lehte [527000/ micron/L]. Hemoglobina e tij eshte 10 g/dL, MCV [volume korpuskular mesatar] eshte 76fL, egzaminimi i lama ve te gj akut periferik tregon pranine e rruazave te kuqe te vogla me citoplazem te zbehte [ anulocite]. Pervec kesaj eshte present nje sideremi e ulur dhe TIBC [ Total Binding Iron Capacity] i rritur dhe nje ferritinemi poshte vlerave normale. Cila eshte procedura me e mire per kete pacient? A) te fillohet terapia me preparate te hekurit B) te pritet 6 muaj dhe te rivleresohet C) ta nxjerresh nga spitali pa I bere asgj e D) te kerkohet gjaku okult ne fece dhe te planifikohet per nje fibrogastroskopi dhe nje kolonoskopi te plote E) te dergohet tek mj eku kurues qe te vazhdoje me analiza

4. eshte: A) B) C) D) E)

Ne prani te petekieve dhe te purpurave duhet vleresuar se pari nese h i perkoles terol e mi a mungesa e faktoreve te koagulimit trombocitoza tro m bocitopenia alterimi i funksionit trobocitar

5. A) B) C) D) E)

Faktori Hageman eshte : faktori XII faktori X faktori trombocitar III kalikreina faktori ristocetinik

6. Trajtimi i perzgjedhur i leukemise mieloide kronike tek moshat e reja e s ht e : A) r a di oter ap ia B) m onokimioterapia C) polikim ioterapia D) terapia me interferon E) transplanti allogj enik i palces se kockes

7. Nje grua 56vje me nje histori progresive te karcinomes se ovarit te trajtuar me kimioterapi disa vite me pare, referon shfaqj en koheve te fundit te astenise dhe te hemorragj ive te vogla ne nivel te kraheve. Vlerat e hemogrames jane si me poshte: Hemoglobina 9,6g/dL, trombocitet 56000/micronL, leukocitet 2900/micronL. Ne mielograme , e cila paraqitet hipercelulare, jane present 10% elemente blastike me aspect megaloblastik te prekursoreve eritroide e megakariocite me anomali berthamore. Egzaminimi citogenetik tregon pranine e nje delecioni te krahut te gj ate te kro mozomit 7. cila eshte diagnoza me probable per kete paciente? A) sindrom mielodisplazik B) anemia megaloblastike C) leukemia limfoblastike akute D) leukemia mieloblastike akute E) leukemia mieloide kronike

8. A) B) C) D) E)

Deformimi si draper i eritrociteve i detyrohet : sinteza difektoze sasijore e hemoglobines crregullimet elektrolitike antikorpet alterimet mitokondriale alterimet cilesore te hemoglobines

9. A) B) C) D) E)

Diagnoza e leukemise acute mund te vihet vetem ne baze te : biopsise se linfonodulit egzaminimit te formulas leukocitare biopsies se palces se kockes ose mielogrames biopsies splenike hem ogram es

1 0. Pacienti me hemolize intravaskulare mund te paraqese te gj itha shenjat klinike dhe laboratorike te meposhtme , PERVEC : A) niveleve te ulta te haptoglobines B) shtimit te bilirubines indirekte C) splenom egalise D) hemoglobinurise e hemosiderurise E) shtimit te Laktatdehidrogjenazes

1 1. Ne remisionin komplet te leukemise akute numri I blasteve ne palcen e kockes nuk duhet e te jete me i larte sesa :

A) B) C) D) E)

5% 20% 30% 40% 50%

1 2. A) B) C) D) E)

Cfare eshte hiatus leucaemicus: nje komplikacion i leukemise mungesa e formave me maturim te ndermj etem ne gjakun periferik nje vartiant i nendarj eve te leukemive nje medikament antiblastik mungesa e formave mature ne palcen e kockes

1 3. A) B) C) D) E)

Te gjithe pacientet me sindrom mielodisplazik paraqesin : sideroblaste unazore nje numer blastesh >5% anomali kromozomike jo rastesore transformim ne leukemia acute displazi ne te gjitha linjat qelizore te palces se kockes

1 4. Cila prej anemive te meposhtme ka mundesi me te medha ti pergjigjet administrimit te eritropoietines : A) anemia sideropenike B) anemia pernicioze C) anemia sideropenike D) anemia hemolitike autoimmune E) anemia nga insuficienca renale

1 5. A) B) C) D) E)

Trupzat Auer gj enden ne menyre karakteristike : ne linfoblastet ne mielocitet ne mieloblastet ne eritroblastet ne linfocitet

1 6. A) B) C) D)

Rritja e IgE serike mund te observohet me shpesh ne : leukemine mieloide kronike anemine hemolitike nga difekti i PK favizem parazitoze

E)

saturnizem

1 7. A) B) C) D) E)

Anemite megaloblastike karakterizohen nga : mikrocitoza e e elementeve te figuruar te gjakut e shoqeruar me deficit te folateve dhe /ose vitamins B12 mungesa e folateve dhe/ose vitamins B12 retikulocitoza ikteri megakoloni toksik

1 8. A) B) C) D) E)

Ne anemine nga mungesa e hekurit transferrina eshte : normale e rritur e ulur pothuajse mungon pa rendesi

1 9. A) B) C) D) E)

Morbus Cooley eshte : nje alfa-talasemi nje beta-talasemi nje anemi enzimodeficitare nje disprotidemi deficit i vitamines B12

2 0. A) B) C) D) E)

Hiperuricemia eshte e shpeshte ne : leukemine mieloide kronike nen trajtim citoreduktues mononukleozen infective hemofili sindromin Gilbert aplazine medulare

2 1. A) B) C) D) E)

Ne hemofili cilat prej testeve te meposhtme j ane te alteruar ; koha e hemorragjise koha e koagulimit koha e protrombines fraxhiliteti kapilar eritrosedimenti

2 2. A) B) C) D) E)

Kontrolli i terapise heparinike realizohet me : vleresimin e trombociteve kohen e hemorragjise proven e lakut kohen e tromboplastines parciale dozimin e fibrinogjenit

2 3. Ne egza minimin obj ektiv, nje pacient me Purpur trombocitopenike idiopatike pervec petekieve dhe ekimozave mund te prezantoje edhe : A) splenomegali B) linfadeno m egali C) hepatomegali D) nje egzaminim fizik normal E) temperature te larte

2 4. Cila nga situatat e meposhtme te ben te dyshosh per pranine e nje leukocitoze neutrofilike : A) infeksioni viral B) infeksioni bakterial C) parazitoza D) alergjia E) asgje patologjike

2 5. A) B) C) D) E)

Biosinteza e hemit nga ana e eritroblasteve ndodh : ne mitokondria ne ribosome ne unazat Cabot ne trupzat Heinz ne nukleolat

2 6. A) B) C) D) E)

Hemofilia C vjen si pasoje e mungeses se: faktorit II faktorit V faktorit VIII faktorit IX faktorit XI

2 7. Ngrenia e batheve mund te provokoje nje anemi grave he molitike ne pacientet te cilet kane nje deficit te ciles prej enzimave te meposhtme: A) piruvatkinazes [PK]

B) C) D) E)

glukoze-6-fosfat dehidrogj enazes [G6PDH] glutation sintetazes gamma glutamilcistein sintetazes laktatdehidrogjenazes

2 8. Nje pacient 18 vje me nje hemofili A te lehte [ niveli i faktorit VIII=15%] ediagnostikuar nga nje histori familiare positive por qe nuk ka pasur asnjehere nevoje per trajtim meqenese nuk ka pasur fenomene hemorragj ike,duhet ti nenshtrohet nje interventi dentar per te hequr dhemballen e pjekurise. Trajtimii zgj edhur eshte : A) k r ioprecip itat B) koncentrat I faktorit VIII C) plazmee fresket e ngrire D) desmopressina [ DDAVP] E) asnje trajtim

2 9. Te gjitha komplikacionet e meposhtme mund te gj enden ne nje pancitopeni si pasoje e mosfunksionit te palces se kockes PERVEC: A) sepsis B) asthenia, te ndjerit keq ne pergj ithesi C) hemorragj ite gingivale D) tromboza E) zbehja

3 0. A) B) C) D) E)

Trombocitoza esenciale dhe policitemia vera j ane : sindroma mielodisplasike sindroma mieloproliferative kronike aplazi medulare semundje sekondare nga inflamacioni kronik shpesh te lidhura me imunodeficiencat

3 1. A) B) C) D) E)

Antitrupat antifosfolipidike gj enden shpesh gjate : anemise aplastike tumoreve gastrike tumoreve ovariene lupusit eritematoz sitemik leukemise mieloide kronike

3 2. A)

Testi i Coombs indirect pozitiv tregon per : pranine e antitrupave antieritrocitare jokomplete ne serum

B) C) D) E)

reaksion imunologjik tip III deficit te piruvat kinazes ekspozim progresiv ndaj primaquines reaksion te rende transfuzionine veprim

3 3. A) B) C) D) E)

Retikulocitet jane : qeliza te sistemit retikuloendotelial qeliza me system retikuloendoplazmik shume te zhvilluar eritrocite te vjeter granulocite te vjeter eritrocite te reja

3 4. A) B) C) D) E)

Kriza retikulocitare verifikohet gjate : anemise hemolitike eritroleukemise policitemise vera nen trajtim me hidroksiure trombozes intravaskulare te diseminuar anemise pernicioze ne trajtim me vitaminen B12

3 5. Cila prej semundjeve te meposhtme nuk i perket te ashtuquajturave syndrome mieloproliferative kronike : A) mielofibroza idiomatike B) leukemia mieloide kronike C) leukemia me qeliza me qime [ tricoleukemia] D) policitemia vera E) trombocitemia esenciale

3 6. A) B) C) D) E)

Cila nga situatat e meposhtme haset ne policitemi ? : shtimi i serise se kuqe ne palcen e kockave hemoviskozimetria normale l e ucopeni a hypotensioni sitemik poliuria

3 7. A) B) C) D) E)

Cila nga rezultatet e meposhtme tregon per nje anemi aregj enerative : vellimi korpuskular mesatar [MCV] normal shtimi i shperndaries se gjeresise se ruazave te kuqe [RDW] numri i reduktuar i retikulociteve niveli i ulur i hemoglobines sideremia e rritur

3 8. Makroglobulinemia Waldentrom eshte e karakterizuar nga nje shtim monoclonal i A) IgG B) IgA C) IgM D) IgD E) IgE

3 9. A) B) C) D) E)

Rezistenca globulare rritet ne : anemine pernicioze anemine hemolitike m ikrocitoze sarkoidoze sferocitozen hereditare

4 0. A) B) C) D) E)

Leukemia lifoide kronike eshte : nje semundje me akumulim te limfociteve B poliklonale nje semundje me nje linfopeni te moderuar nje semundje me akumulim te plazmociteve nje semundje me akumulim te limfociteve B monoklonale nje semundje e karakterizuar nga prania e kromozomit Philadelphia

4 1. A) B) C) D) E)

T ransplanti palces se kockes indikohet ne : aplazine medulare grave dhe ne leukemite akute Pancitopenine periferike me palce te pasur Anemine pernicioze Anemine sideropenike Leukemine linfoide kronike

4 2. A) B) C) D) E)

Fillimi i leukemise acute mund te jete i karakterizuar nga : prania ne serum e nje piku monoclonal Hipergammaglobulinemia poliklonale Infeksioni febril rezistent ndaj mj ekimit H iperkalce m ia Hiperkolesterolemia shume e larte

4 3. Endokrinopatite e meposhtme mund te j ene tegjitha shkak direct I nj e anemie aregj enerative , PERVEC : A) diabeti mellitus B) hipotiroidizmi C) hipopituitarizmi D) hiperparatiroidizmi E) hipogonadizmi

4 4. Ne cil en prej patologjive te meposht me verifikohet me shpesh trombocitoza: A) leukemine mieloide kronike B) leukemine mieloblastike acute C) leukemine limfoblastike acute D) morbus Waldentrom E) morbus Werlhof

4 5. Prezenca e artritit rheumatoid, splenomegalise, neutropenise eshte karakteristike gjate : A) konektivitit miks B) sindromit Felty C) sindromit Goodpasture D) semundjes Chediak-Higashi E) LES

4 6. A) B) C) D) E)

Heparina fuqizon veprimin e : Fibrinogjenit Antitrombines III Faktorit VIII Tromboplastines Tro m bones

4 7. Ne organet e meposhteme gj enden ne sasi te madhe limfocitet T , PERVEC : A) Linfonoduj ve B) Shpretkes C) Timusit D) Traktit gastrointestinal E) Gjakut periferik

4 8. Nje pacient me deficit te Glukoze-6fosfat dehidrogj enazes [G-6-PD] duhet te evitoje te gj itha keto me poshte , PERVEC : A) C efalotines B) Chinines C) Clorochines D) Batheve E) S ulfa m idikeve

4 9. Cila nga pergjijet e meposhtme NUK SHOQEROHET me nje anemi hemolitike me atitrupa te ngrohte : A) Linfomat jo hodgkiniane B) S plenomegalie C) Sf erocitoza D) Testi Coombs direct pozitiv E) Fenomeni Raynaud

5 0. Cili nga situatat e meposhtme eshte ME PAK PROBABEL qe te shoqerohet me nje hipereozinofili ? A) Sindomi hipereozinofilik B) Leukemia mieloide kronika C) Infeksionet parazitare D) Infeksionet nga stafilokoku aureus E) Astma

5 1. A) B) C) D) E)

Shizocitet jane elemente te figuruar te gjakut : H ipokro m ike Me dimensione te medha Ne forme rrakete Te fragmentuar Target

52. A) B) C) D) E)

Sindromi I hiperviskozitetit observohet me shpesh gjate : Anemive hemolitike autoimmune Leukemise limfoblastike acute Insuficiences respiratore kronike Mielomes multiple Hemoglobinopative

5 3. Cila prej ketyre shenjave dhe simptomave mielomen multiple :

shoqeron me shpesh

A) B) C) D) E)

Dhimbjet kockore Linfade nopa ti a Rubeosis Splenomegalia Petekiet

5 4. A) B) C) D) E)

Koha e protrombines influencohet prej : Uljes se numrit te trombociteve Shtimit te faktorit antiheparinik Deficiti I faktorit VII Deficiti I faktorit VIII Deficiti I faktorit IX

5 5. A) B) C) D) E)

Zhdukja e nukleolave verehet ne : Mie loblast et Promielocitet Mielocitet Ne metamielocitet Ne granulocitet e pasegmentuar

5 6. A)

Hapi i pare per te diagnostikuar policitemine vera eshte : Te percaktohet masa e rruazave te kuqe te gjakut dhe e volumit p la z m a ti k * B) Te percaktohet niveli serik I eritropoietines C) Te behet nje ekografi renale D) Te behet astrupograma E) Te matet fosfatazaalkaline leukocitare

5 7. A) B) C) D) E)

Varianti histologj ik me malinj i M. Hodgkin eshte : Me qeliza te klivuara Me deplecion linfocitar Me prevalence linfocitare Skleroza nodulare Si qiell me yj e

5 8. gj itha A) B) C)

Manifestimet klinike te nje anemie nga mungesa e vitamins B12 jane te reversible me administrimin e vitamins B12, PERVEC : Demet ne palcen e kockes Anemia Neutropenia

D) E)

Neuropatia periferike Tro mbo citopenia

5 9. Pacienti me hemolize akute mund te paraqese te gjitha te dhenat klinike e laboratorike ,PERVEC : A) Litiazes biliare B) Hiperplazise ne nivel medular C) Shtim i MCV me polikromatofili D) Hiperbilirubinemia indirekte E) Shtimi i laktatdehidrogjenazes

6 0. A) B) C) D) E)

J ane shkak i poliglobulise sekondare : Hemofilite Insuficienca respiratore kronike Hemorragjia kronike Leukemia mieloide kronike Policitemia vera

6 1. A) B) C) D) E)

Organi madhor hematopoietik ne jeten embrionale eshte : Palca e kockes hematopoietike Melcia Shpretka Linfonodujt Sakusi Vitelin

6 2. Probabiliteti qe nje femije mashkull i nje pacienti me hemofili grave te kete hemofili eshte : A) 0% B) 25% C) 50% D) 75% E) 100%

6 3. Kromozomi Philadelphia [Ph] eshte nje marker citogenetik ne nje prej situatave te meposhtme : A) Leukemia mieloide kronike B) Hairy cell leukemia C) Leukemia prolinfocitike D) Kriza blastike me fenotip eritroid ne LMC E) Eritroleukemia

6 4. A) B) C) D) E)

Deficit i vitamins K evidentohet nga nje prej analizave te meposhtme : Numri i trombociteve Koha e protrombines [PT] Koha e tromboplastines parciale [PTT] Koha e trombines Koha e hemorragjise

6 5. Gj ate periudhes qe pason transplantin allogjenik e palces se kockes, komplikacionet e meposhtme i detyrohen gjendj es imunitare te marresit , PERVEC : A) Infeksionet mykotike B) Reaksionet e flakjes C) Herpes Zoster D) Graft versus host disease E) Pneumonia intersticiale

6 6. kohes A) B) C) D) E)

Cila nga situatat e meposhtme NUK SHOQEROHET me nje zgjatje te se hemorragj ise : Uremia Trombocitopenia grave Mieloma multiple Hemofilia Morbus vonWillebrandt

6 7. A) B) C) D) E)

Splenomegalia eshte nje hasje frekuente ne diagnose gjate : Mielomes multiple Mykozes fungoide Anemise aplastike Leukemise mieloide kronike Hemofilise

6 8. Cila nga situatat e me poshtme i perket sindromeve mieloproliferative kronike ? A) Mielofibroz idiopatike B) Leukemia mieloblastike C) Leukemia me qeliza me qime D) Leukemia plazmocelulare E) Mieloma multiple

6 9. Probabiliteti qe vajza e nje pacienti me hemofili B te jete nje bartese e hemofilise eshte : A) 0% B) 25% C) 50% D) 75% E) 100%

7 0. A) B) C) D) E)

Eozinofilia mund te haset me shpesh gjate : Leukemise mielomonocitike Mielomes multiple Leukemise limfoide kronike Morbus Hodgkin A granulocitozes

7 1. A) B) C) D) E)

Sindromi Sezary eshte : Nje linfome me linfocite T Nj eleukemi linfoide kronike me infiltrim kutan Nje variant i talasemise Nje difekt congenital i IgG Nje variant i leukemise mieloide kronike

7 2. A) B) C) D) E)

Nder tumoret primitive te shpretkes, cilet prej tyre jane me te shpeshtet: Fibroma Miomat Hemangiomat Linfom at Linfangiomat

7 3. A) B) C) D) E)

Nje situate e nje anemie te rende sideropenike mund te shkaktohet nga : Menorragjite persistente Bronkopneumonite recidivante Dietat vegj etariane Mosmarrje adekuatee folateve Hepatopatia alkoolike

7 4. A)

Faktori kryesor qe rregullon aktivitetin eritropoietik eshte : Veshka

B) C) D) E)

Ferritina Unitete formuese te kolonive eritroide Oksigj eni Palca hemopoietike

7 5. Cila prej ketyre anemive hemolitike eshte e shkaktuar prej difektit congenital te nje enzyme e ritrocitare [G6PDH]: A) Favizmi B) Sf erocitoza C) Elipsocitoza D) Stomatocitoza E) Hemoglobinuria paroksisitike e te ftohtit

7 6. Vitamina K eshte e nevojshme per sisntezen e secilit prej faktoreve te me poshtem te koagulimit, PERVEC se per sintezen e faktorit : A) II B) V C) IX D) VII E) X

KARDIOLOGJI 1. A) B) C) D) E) Kriza lipotimike kan nj incidence t rritur n: Insuficencn e aortas Stenozn e aortas Stenozn e istmusit t aortas Insuficiencn mitrale Steno-insuficiencn mitrale

2. A) B) C) D) E)

Cfar kuptoni me termin miokard spongioz? Mungesa e trabekulave normale n pj esn e brendshme t ventrikulit t maj t si pasoj e nj defekti maturimi n jetn intrauterine Prania e trabekulave n pjesn e brendshme t ventrikulit t majt si pasoj e nj defekti maturimi n jetn intrauterine Ekzistenca e foramen ovale ndrventrikulare pas lindjes Zvendsimi i miokardit me ind dhjamor Situat klinike q karakterizohet nga zgj erimi i madh i ventrikulit

3. Rritja e vllimit qarkullues (volemis), vj en si pasoje e t gjith gj endjeve t posht shnuara me prjashtim t: A) Rritjes t filtrimit glomerular B) Uljes t sekretimit t aldosteronit C) Uljes t sekretimit t ADH D) Uljes t prthithjes t natriumit E) Rritjes t aktivitetit t renins plazmatike

4. N infarktin e miokardit cila enzime paraqitet n vler m t rritur fillimisht? A) Mioglobina B) SGOT C) LDH D) CK-MB E) Troponina

5. t: A) B) C) D) E)

N stenozn e valvuls mitrale evidentohen t gj itha kto me prj ashtim Hipertrofi e arterieve pulmonare Hipertension venoz pulmonare Hipertrofi e atriumit t maj t Hipertrofi e ventrikulit t majt Zhurm diastolike n apex

6. A) B) C) D) E)

Angina gj oksore mund t shfaqet: si pasoj e t ftohtit gjat sforcimit n qetsi gjat stresit t gj itha prgj igjet m lart j an t sakta

7. A) B) C) D) E)

Cila shenj nuk zbulohet n stenozn mitrale? zhurm diastolike me prforcim presistolik pulsi i shpejt theksim i tonit t dyt n apex theksim i tonit t dyt n baz zhurm n hapje t valvuls

8.

N stenozn e aorts cila nga shenjat e mposhtme vihet re?

A) B) C) D) E)

pulsi krcyes pulsimi i arterieve karotide pulsimi i arterive femorale pulsi i vonuar asnj nga kto

9. A) B) C) D) E)

N stenozn e aortas, dgj ohet: toni i II-t n sistol krcitje n fillimin e hapjes t valvuls zhurma diastolike zhurm sistolike zbrazse n hapsirn interkostale t II-t t gj itha kto

1 0. Cili nga pohimet sht I vrtet pr bllokun atrio-ventrikular t grades t II-t tip II? A) Zgj atje e intervalit PR mbi 0.20 sek pr do kompleks QRS B) Zgj atje progresive e intervalit PR n tri komplekse QRS t nj pas nj shme, shoqruar me munges t vales P pasuese t QRS C) Val P ndjekur nga komplekse QRS me intervale PR normale ose t alteruar nga vale P q nuk pasohen me komple kse QRS D) Nuk ka lidhje midis vales P dhe komplekseve QRS. Frekunca e komplekseve QRS sht 30 bpm E) Mungesa e valve P t dukshme

1 1. A) B) C) D) E)

N ciln prej ktyre smundj eve degj ohet nj klik mezo-sistolik? insuficience mitrale prolaps i valvuls mitrale infarkt aneurizm aortale defekt interventrikular

1 2. N nj pacient me fibrilacion atrial kronik cili sht niveli i INR n mnyr t till q t evitohet nj iktus embolik? A) 2 B) 3 C) 4 D) 6 E) 1

1 3. Nj njeri 57 vje, hipertensiv, me hiperkolesterolemi dhe mbipesh ankon pr nj dhimbje shtrnguese n kraharor n sforcime t lehta (si ngj itje shkallve, dhnia e biiklets). Cilin ekzaminim do t krkonit fillimisht? A) Shintigrafi me Talium 201 B) Cikloergometri C) K oronarografi D) E kokardiograma E) TAC

1 4. Cili nga ndrlikimet e infarktit t miokardit mund t na oj drejt tamponads kardiake? A) Insuficienca mitrale akute B) Ruptura kardiake C) Ruptura e septumit interventrikular D) Insuficienca kardiake akute E) Insuficienca mitrale

15. A) B) C) D) E)

Cili sht ndrlikimi m i rnd q vrehet n stenozn mitrale? Fibrilacioni atrial Infarkti pulmonar Infarki miokardit Embolia sistemike Asistolia

1 6. A) B) C) D) E)

N EKG nj ekstrasistol ventrikulare karakterizohet nga: kompleksi QRS i ngusht ndjekur nga nj pauz ko mpesatore kompleksi QRS i gjer ndjekur nga nj pauz ko mpesatore kompleksi QRS i gjer ndjekur nga nj pauz jo kompesatore kompleksi QRS i ngusht ndjekur nga nj pauz jo kompesatore kompleksi QRS i gjer pa pauz

1 7. Cila situat klinike nuk karakterizohet nga ulja e fraksionit t mbushjes t ventrikulit t majt shoqruar me kongj estion pulmonar? A) Stenoza mitrale B) Kardiomiopatia hipertrofike C) Tamponada kardiake D) Insuficienca mitrale E) Infarkti akut i miokardit

1 8. infarkt A) B) C) D) E)

Cili sht niveli i kolesterolit LDL q duhet t arrij nj pacient me miokardi? < 130 mg/dl < 200 mg/dl < 100 mg/dl < 180 mg/dl < 110 mg/dl

1 9. A) B) C) D) E)

Efekti inotrop pozitiv n zemr realizohet nga: acidoza rritja e hormoneve tiroides hiperkale m ia hipokalemia frenimi beta-adrenergjik

2 0. A) B) C) D) E)

Prcaktuesi kryesor i konsumit t oksigj enit nga miokardi sht: presioni arterial sistolik presioni arterial diastolik kontraktiliteti miokardit frekuenca kardiake vllimi ventrikular telediastolik

2 1. A) B) C) D) E)

Shkaku m i shpesht i pamj aftueshmris t zemrs sht: disfunksioni diastolik i ventrikulit t majt disfunksioni sistolik i ventrikulit t majt hipovolemia disfunksioni i ventrikulit t djatht shoku

2 2. A) B) C) D) E)

Raporti ventilim/perfuzion tregon raportin e: kapacitetit vital dhe fluksit t gj akut qarkullues n mushkri PCO2 dhe Po2 n gj akun arterial ventilimit dhe fluksit t gjakut qarkullues n njsin e mushkris vllimin e gazeve dhe vlimin e gjakut qarkullues n mushkri kapacitetin pulmonar total dhe prurjen kardiake

2 3. Cilat j an karakteristikat elektrokardiografike t infarktit subendokardial? A) subnivelimi i ST dhe shfaqja e vals Q B) supraniveli ST dhe shfaqja e vals Q

C) D) E)

subnivelimi i ST dhe mungesa e vals Q subnivelimi i ST dhe shfaqja e vals U ST n vij n izoelektrike dhe shfaqja e vals Q

2 4. N lidhje me nj pacient 65 vje, hipertensiv, me fibrilacion atrial kronik t stabilizuar, pa faktor t tjer riskant, jan t vrteta t gj itha pohimet e m poshtme me prjashtim t: A) Me prjashtim t rasteve kur ka nj kundrindikacion absolut, terapia do t konsistonte n prdorimin e antikoagulantve oral B) Pacient nuk mund t trajtohet vetm me aspirin C) Pacienti duhet t traj tohet vetm me barna antihipertensiv D) Duhet realizuar konvertimi i ritmit E) Pacienti duhet trajtuar vazhdimisht me heparin subcutane

2 5. A) B) C) D) E)

Cili nga kta faktor nuk lidhet me trombfilin? Defiiti i proteins C Defiiti i proteins S Defiiti i antitrombins III Hiperhomocisteinemia Neoplazia

2 6. A) B) C) D) E)

Prkufizohet si angina nga eforti: Angina de nuova Angina e qndrueshme Angina Prinzmetal Angina e qetsis Angina si pasoj e ngarkess

2 7. A) B) C) D) E)

Refluksi pozitiv hepato-j ugular flet n favor t: Pamjaftueshmris zemrore me natyr kongjestive Obstruksionit t vena cava superior Obstruksionit t vena ports Obstruksionit t vena kava inferior Situat fiziologjike

2 8. Terapia n insuficiencn kardiake prfshin prdorimin e t gj ith barnave t posht shnuar me prj ashtim t: A) beta-bllokuesve B) A C E-inhibitorve C) Statinave

D) E)

Eritropoetins Diuretikve

2 9. A) B) C) D) E)

Cili nuk sht efekt ansor i terpais me ace-i nhibitor? H ipotensioni Kolla e that Ndryshimet n shije H iponatre m ia Edema angioneurotike

3 0. A) B) C) D) E)

Zhur ma e Flint dgjohet n: fibrilacion atrial insuficienc mitrale stenoz aortale insuficienc aortale duktus Botalli

3 1. me: A) B) C) D) E)

N pacientin hipertensiv graviteti i dmtimit organor lidhet ngushtsisht presionin arterial t monitoruar n klinik presionin arterial t monitoruar n shtpi presionin arterial t monitoruar dy her n dit presionin arterial t monitoruar pr 24 or presionin arterial t monitoruar gjat nats

3 2. A) B) C) D) E)

Nga pikpamja klinike fibrilacioni ventrikular karakterizohet nga: dispnea e rnd infarkti akut arresti kardiocirkulator cianoza gishtat si shkop tamburi

3 3. Cili sht elementi q haset m shpesh n kuadrin klinik t stenozs mitrale? A) prekordialgia B) he m optizia C) dispnea D) iktus E) ngjirja e zrit

3 4. Cili lloj i infarktit akut t miokardit (IAM) mund t shoqrohet me nj aktivizim vagal? A) IAM septal B) IAM lateral C) IAM anterior D) IAM diafragmatik E) Asnj nga kto

3 5. Cili prej elementve t mposhtm mund t prdoret pr t diferencuar insufiirncn ventrikulare t majt nga insuficienca ventrikulare e djatht? A) Rritja e enzimave hepatike B) Edema pulmonare C) Asciti D) Edema periferike E) Tendosja e venave jugulare

3 6. Cili duhet t j et bari q i duhet dhn fillimisht nj pacienti me hiperkalemi dhe komplekse QRS t zgjeruar ne EKG? A) klorur kaliumi B) kayexalate C) digoksin D) furosemid E) bikarbonat natriumi

3 7. A) B) C) D) E)

Cili sht ndrlikimi m i rnd q vrehet n stenozn mitrale? Fibrilacioni atrial Infarkti pulmonar Infarki miokardit Embolia sistemike Asistolia

3 8. A) B) C) D) E)

Angioplastika koronare percutane indikohet n: stenozn koronare insuficiencn kardiake difektin interatrial artropatin e ansis t poshtme amiloidozn kardiake

3 9. A) B) C) D) E)

N disekacionin e aorts e dhna semiotike m e rndsishme sht: mungesa e pulseve arterial hipokratizmi digitalik cianoza bllok i degs s djatht pectus excavatum

4 0. A) B) C) D) E)

N ciln nga kto kardiopati dgj ohet nj klik mezosistolik? Insuficienca mitrale Prolaps i valvuls mitrale Kardiopati infarkti Aneurizma aorts Defekti interventrikular

4 1. Cili nga kta faktor nuk lidhet me vdeksh mrin e rritur t nj pacienti me pamjaftueshmri kardiake? A) Ulja e intervlait RR B) Prdorimi i ACE-inhibitorve C) Rritja e BNP (brain natriuretik peptide) qarkulluese D) Ventrikuli i maj t merr form sferike E) TA m pak se 80/60 mmHg

4 2. A) B) C) D) E)

Shkaku m i shpesht i stenozs t valvuls aortale sht: Smundje t lindura Smundje reumatizmale Degj enerimet fibrokalike t pleqris Degj enerimet fibrokalike t val vuls aortale bikuspidale Endokarditi infektiv

4 3. A) B) C) D) E)

Cili sht simptomi i menjhershm i rupturs t aneurizms t aorts? Dhimbja e menjhershme dhe/ose shoku Melena Hematemeza Anuria Zhdukje e pulseve periferike

4 4. A) B)

Pulsi parvus et tardus sht i pranishm n: Stenozn mitrale Insuficiencn e valvuls aortale

C) D) E)

Insuficiencn e valvuls pulmonare Stenozn aortale Inseficiencn mitrale

4 5. N nj pacient me kardiopati ishemike dhe diabet cila sht vlera q duhet t ket tensioni arterial q t quhet optimale? A) = 130/80 B) = 120/80 C) = 100/80 D) < 120/80 E) < 130/80

4 6. N ekzaminimin objektiv t nj pacienti me flater arterial nuk mund t degj ohet: A) toni i I-r B) toni i II-t C) toni i IV-t D) zhurm e insuficiencs mitrale E) zhurm e stenozs mitrale

4 7. N insuficiencn e valvuls aortale vihen re t gjith elementt e poshtshnuar me prjashtim t: A) zbehje e lkurs dhe mukozave B) rritje e diferencs midis presioneve arterial C) pulsi tardus D) pulsim i dukshm arterial E) zhurma Flinit

4 8. A) B) C) D) E)

Ulerimi i pllaks ateromatoze koronare lidhet me: ishemin vazokonstriksionin fokal koronar grumbullimin e trombociteve anginn e paqndrueshme t gjitha

4 9. Cili nga elementt e posht shnuar nuk sht nj faktor rreziku kardiovaskular? A) H iperhom ocisteinem ia B) Proteina C reaktive > 1mg/dl C) Dislipidemi

D) E)

Cirkumferenca abdominale= 102 cm n meshkuj dhe =88cm n femra Rritja e TNF qarkulluese

5 0. A) B) C) D) E)

Cili nuk sht faktor rreziku pr emboli pulmonare? Trau ma/fraktura Mosha e madhe Insuficienca kronike venoze Hipotiroidizmi Obeziteti

5 1. A) B) C) D) E)

T gj itha jan karakteristika t angins t qndrueshme m prjashtim t: Ndjesia e rndess ose e djeies n regjionin retrosternal Fillon n qetsi dhe prkeqsohet me sforcimet Nuk zgj at m shum se 20 min Qetsohet pas prdorimit t nitrateve Gjat gj umit shprthen m shum gj at fazs REM t tij

5 2. A) B) C) D) E)

Studimi fiziologjik endokavitar sht i domosdoshm n : Pacientt me ekstrasistola t shpeshta Pacientt me insuficienc kardiake Pacientt me bllok t degs t djatht Pacientet me bllok dege Pacientet me aritmi ventrikulare malinj e

5 3. Nse pacienti ka br pr her t par nj episod anginoz, si do ta klasifikojm smundjen? A) Angina parainfarktit B) Angina stabile C) Angina j ostabile D) Angina me rrezik t lart E) Asnj

5 4. Nj dj al 30 vje paraqitet tek mjeku me temperatur me frisione, palpitacione, djersistje e shtuar; n ekza minim objektiv evidentohet nj zhurm sistolike. Pr far do t dyshonit? A) Ruptur e kordave tendine B) Insuficienc aortale C) Mixoma atriale D) Endokardit bakterial E) Infarkt

5 5. N nj t smur i cili mban proteza valvulare t tipit mekanik, far terapie do t ndiqnit? A) Terapi me antikoagulant, duke rruajtur INR 2.5-3.5 pr 6 muaj ndjekur nga terapia me antiagregant B) Terapi me antikoagulant, duke rruajtur INR 2.5-3.5 pr nj koh t pa prcaktuar C) Terapi me antikoagulant, duke rruajtur INR 2-3 pr nj koh t pa prcaktuar D) Terapi me antikoagulant, duke rruajtur INR 2-3pr 6 muaj ndjekur nga terapia me antiagregant E) Terapi me antikoagulant, duke rruajtur INR 2.5-3.5 pr 1 vit ndj ekur nga terapia me antiagregant

5 6. A) B) C) D) E)

Kontraktiliteti i miokardit zvoglohet si pasoj e: Prdorimit t glukozidve digitalik Angiotenzins II g l ukagonit katekola m inave hiperkapnis

5 7. Dhimbja torakale q fillon n mnyr t papritur shoqruar me polipne dhe hemoptizi modeste vihet re n: A) pleurite hemorragjike B) pleurite eksudative C) pachipleurite D) infarkt pulmonar E) infarkt miokardial

5 8. A) B) C) D) E)

Vendidgjimi i valvuls aortale, n kushte normale, sht: hapsira e II-t interkostale maj tas parasternale hapsira e III-t interkostale maj tas parasternale hapsira e IV interkostale maj tas parasternale hapsira e II-t interkostale djathtas parasternale hapsira interkostal subklavikulare

5 9. A) B) C)

Palpitacionet, cefalea pulsante, djersitja natn j an tipike t: insuficiencs aortale stenozs aortale insuficiencs pulmonare

D) E)

insuficiencs trikuspidale stenozs mitrale

6 0. Cila nga kto situata konsiderohet si me rrezik t lart pr zhvillim t endokarditit bakterial: A) endokardit bakterial i mparshm B) kardiomiopati hipertrofike C) ndrhyrje kirurgjikale pr by-pass D) prolaps i valvuls mitrale pa disfunksion valvular E) prolaps i valvuls mitrale me insuficienc mitrale

6 1. A) B) C) D) E)

Toni i II-t kardiak korrespondon me: mbyllj en e valvulave atrioventrikulare kontraksionin atrial mbyllj en e valvulave semilunare kontraksionin ventrikular asnj nga kto:

6 2. A) B) C) D) E)

Cfar tregon prania e vals P n derivacionet II-III dhe Vf t EKG-s? IAM anterior IAM inferior IAM n zhvillim e siper n pjesn inferiore Fibrilacion ventrikular Flater atrial

6 3. A) B) C) D) E)

Insuficienca ventrikulare e djatht sekondare sht pasoj e: Stenozs t istmusit t aorts Insuficiencs valvulare aortale Stenozs valvulare aortale Sarkoidozs pulmonare kronike Hipertensioni arterial malinj

6 4. N meshkujt e popullats, manifestimi i par i nj smundje kardiake shpesh her prfaqsohet nga: A) Angina kronike e paqndrueshme B) Infarkti C) Vdekja e papritur D) Angina e qndrueshme E) Insuficienca kardiake post-ishemi ke

6 5. A) B) C) D) E)

Vlersimi kryesor n kardiomiopatin dilatatuese sht: EKG E kokardiograf ia K oronarogra fi a Shintigrafia e miokardit Ci kl oerg ome tria

6 6. Niveli serik i peptidit atrial natriuretik n nj pacient me insuficinc kardiake kronike sht n prputhje me: A) moshn e pacientit B) gjinin dhe moshn e pacientit C) shkalln e dmtimit t funksionit t zemrs D) vlerat e presionit sistolik E) nivelin e aterosklerozs n arteriet koronare

6 7. Cili prej barnave t poshtshnuar sht bari i zgjedhur pr parandalimin e fibrilacionit ventrikular gjat trajtimit t infarktit akut t miokardit? A) lidokaina B) digoksina C) kinidina D) flrkanidi E) propanololi

6 8. t: A) B) C) D) E)

T gj itha t mposhtme j an shkaqe t ndalimit t zemrs me prj ashtim Bllok atrioventrikular grada III-t rregullim i primit Asistolia Takikardia ventrikulare Shok volemik

6 9. N rastet me angina pectoris t paqndrueshme, cili bar duhet konsideruar si terapia mbajtse, prve barnave t nevojshm pr t kontrroluar simptomat e angins? A) aspirina 100 mg/dita B) aspirina 325 mg/dita + calciparina 0.5 mg per 2/dita C) diltiazem 360 mg/die D) verapamil 240 mg + nifedibin 40 mg/dita E) nitroderivate nn gjuh sipas nevojs

7 0. T gj ith barnat e mposhtm j an t nevoj shm n traj timin e angins gj oksore t paqndrueshme me prjashtim t: A) Nitrateve B) B etabllokuesve C) O2 D) D igoksins E) K alibllokuesve

7 1. Duke patur parasysh fispatologj in e shokut hipovolemi k, cila nga veprimet e poshtshnuara nuk duhet ndrmarr? A) terapia me oksigjen B) pozicionimi i pacientit shtrir me shpin dhe me kokn e ulur C) aplikimi i terapis me vazo konstriktor pr t ruajtur presionin arterial D) aplikimi i terapis endovenoze me solucione koloidale E) monitorimi i presionit venoz qndror

7 2. Tamponada kardiake sht nj situat emergjence e cila evidentohet gj at nj: A) goditje t fort t toraksit B) stenoze t rnd mitrale C) nj aksidenti rrugor D) grumbullimit t likidit t presuar n hapsirn perikardiale E) stenoze t rnd aortale Prgjigj ia e sakt sht:D 7 3. Cili prej faktorve t mposhtm riskant pr zhvillimin e smundjeve kardiovaskulare prgjigjet edhe pr disfunksionin erektil: A) d u h an pirja B) dislipidem ia C) diabeti melitus D) t gj itha prgj igjet e msiprme E) jeta sedentare

7 4. Cila nga ndryshime e poshtshnuara sht karakteristike e cor p ul m o n ar: A) hipertrofia e ventrikulit t djatht B) zgj erimi i ventrikulit t majt C) shunti interventrikular majtas-dj athtas D) persistimi i pulsit arterial E) sindroma Claude-Bernard-Horner

7 5. T gjith barnat antihipertensiv t shnuar m posht j an t kundrindikuar n shtatzani, me prjashtim t: A) betabllokuesit B) A C E-inhibitort C) k a liantagon istt D) nitratet E) diuretikt

7 6. A) B) C) D) E)

Hipertensioni arterial n insufiencn renale kronike vj en si pasoj e: stenozs t arteries renale t njrs veshk stenozs t arteries renale n t dy veshkat rritjes t vllimit jashtqelizor si pasoj e retensionit sodik hiperaldosteronizmit primar sindroms Barter

MJEKESIA E FAMILJES 1. Mashkull 40 vj e, ka dhimbje t forta t kockave t kmb ve, zverdhje t syve, urinim me ngj yr t errt dhe anemi. Lieni nuk preket. N serin e kuqe ka qeliza target dhe n form drapri, si dhe retikulocite t larta. Diagnoza e mundshme: A) Hepatit viral akut B) Leukemi akute C) Thalasemi maj or D) D repanocitosis 2. Femr 35 vj ee, ka anemi, thonj t thyeshm, glosit atrofik. Gjaku periferik: hipokromi, mikrocitoz e eritrociteve. Diagnoza e mundshme A) Anemi nga mungesae vit.B12 B) Anemi hemolitike C) Anemi nga mungesa e acidit folik D) Anemi nga mungesa e hekurit

3. Burr 75 vje, ka gj endrra t rritura n t gjitha regionet. Shpretka 4 cm, hepari 4cm. Gjaku periferik: er 4000000, HB 12 gr/dl, Tb 230.000 mm, leukocite 100.000 mm, Formula: Limfocite 97%, segmente 3%. Diagnoza e m undsh m e A) Leukemi limfoblastike akute B) Mononukleoz infektivi C) Leukemi limfoide kronike D) Leukemi mieloblastike akute

4. Grua 50 vjee, pas nj udhtimi vj en n urgj enc me dhi mbje t fort gj oksi, dispne, hemoptizi. Ekzaminimi radiologj ik normal. ECG takikardi sinusale. Diagnoza e mundshme: A) Pneumotoraks spontan B) Infarkt akut i miokardit C) Trombemboli pulmonare D) Me d iastinit

5. Burr 50 vje, duke vrapuar i filloi nj dhimbje e fort gj oksi q u shoqrua edhe me marrje fryme. Pacienti ndaloj vrapin dhe dhimbja u qetsua. ECG n qetsi normal, TA 130/80mmHg. Diagnoza e mundshme: A) Refluks gastro-esofagal B) Angina pectoris C) Pleurizi D) K ostokondritis

6. Mashkull 60 vj e, alkolist, paraqitet me hematemez masive dhe gjendje shoku. Ka ascit dhe yje vaskulare. Shkaku i hematemezs: A) Ulcer peptike kronike B) Varicet e esofagut C) Cancer i stomakut D) Sindroma Mallory- Weiss

7. Mashkull 40 vje, paraqitet me melen, nuk ka dhimbje barku, nuk ka rn n pesh. N ekza minim anusi dhe rektumi j an normal. sht alkolist dhe pi duhan. Shkaku i melens: A) Carcinoma e rektumit B) Ulcera peptike C) H emorhoidet D) Smundja divertikulare

8. Pacient 34 vj e, hera e katrt q vjen me kolika biliare si pasoj e kalkulozs s kolecists. Cili sht mjekimi i zgj edhur pr t smurin: A) Prdorimi i spazmolitikve B) Prdorimi i gur shkrirsve C) Prdorimi i analgj ezikve D) Kolecistectomia

9. Femr 60 vj ee, ka dhimbje koke bitemporale, turbullim t shikimit n nj sy, dhimbje t koks kur kreh flokt. Eritrosedimenti sht i lart. Shkaku i dhimbj es s koks: A) M eningitis B) Arterit me qeliza gjigante C) Hemoragji suaraknoidale D) Spondiloz cervikale

10. Grua multipare, paraqitet me hemoragji uterine n j avn e 9 t shtatzanis. Ostiumi intern nuk sht i dilatuar. Cerviksi sht normal. Diagnoza e m undsh m e A) Missed abortion B) Rrezik aborti C) Abort inkomplet D) Abort habitual

11. Gravide 30 jav, ka disuri, temperatur 38, dhimbje mesi. Analiza e urins, leukocite me shumic si dhe bakterie. Urokultura jep antibiotikt sensibl si m posht. Zgj idh mj ekimin e prshtatshm A) C hloramphenicol B) D oksacillini C) Rifa mpicin e D) Ceporine

12. Grua hipertonike, sht shtatzan. Zgj ith mj ekimin m t prshtatshm pr hipert onin. A) aldomet B) Adalat C) Enalapril D) Lasix 13. Nulipare n javn e 20 t shtatzanis, ka dhimbje koke, dhe dhimbje epigastrike. Tensioni arterial sht159/100 dhe ka tendenc t rritet. Diagnoza e mundshme A) Shkolitje placente B) Pre-ecalampsi fulminante C) P yelonefrit D) H ydra m nios 14. Diabetike 40 vjeare. Gravide n muajin e 7 t shtatzanis. Zgjidh mjekimin pr diabetin.

A) B) C) D)

Metformin +diet D aoni l +diet Metformin +daonil Insulin+ diet

15. Fe mr 35 vj e, ka koll, temperatur, faringitis. Radiografi pulmoni (Ro):zona t prhapura konsolidimi bilaterale, n trajt nj ollash. Testi pr aglutinina t ftohta+. Diagnoza e mundshme A) Grip B) pneumonia atipike C) mykoplazma pneumonie D) metastaza karcinomatoze

16. Mashkull 65 vj e, me koll kronike produktive, vjen n urgj enc me dispne, dhe i prgj umur. Pulmone me rale bronkiale, sibilanca, ulje e lvizshmris s pulmoneve. Diagnoza e mundshme A) astma bronkiale B) acarim i bronkitit kronik C) edem pulmonare akute D) pneumonia atipike

17. Mashkull 55 vj e, zgjohet me dispne, koll me pshtym me shku m, ka kaluar infarkt akut t miokardit 3 jav m par. Ka takipne, cianotik, rale krepitante n pulmone. Diagnoza e mundshme A) astma bronkiale B) edema pulmonare akute C) embolia pulmonare D) acarim i bronkitit kronik

18. Fmij 1.5 vje q shkon n cerdhe, ka koll, wheezing. Ai ka lvizje t flegrave t hunds dhe sibilanca bilaterale n auskultim. Diagnozat e m undsh m e A) astma bronkiale B) bronkiolitis C) pneumoni atipike D) pneumoni bakteriale

19. Dj al 10 vj e, vj en n ambulanc sepse n fiskultur pati gj enddje t fikti me dridhje. N eksaminim ka zhurm t lart sistolike t ejeksionit q shoqrohet me trill (dridhje). Cila sht diagnoza m e mundshme A) Ethe rheumatizmale akute

B) Stenoz e aorts C) m yokardit D) Insuficienca kardiake kongestive

20. Cila nga karakteristikat e posht shnuara nuk u prket konvulsioneve febrile t thjeshta: A) B) C) D) Shfaqen n temperaturn >38 grad celsius Jan toniko klonike Nuk ln deficite neurologjike Shfaqen n moshn mbi 6 vje

21) Cili nga antibiotikt e poshtshnuar sht zgjedhja e par pr mjekimin e anginave streptokoksike, n se pacienti ka alergji nga antibiotikt betalakta m id: A) a mp ici lina B) gentamicina C) a mikacina D) eritromicina

22. Cili nga ushqimet e mposhtme nuk absorbohet n smundj en celiake: A) B) C) D) Kosi Mishi Orizi Buka e grurit

23. Smundja reumatike shfaqet n prgj ithsi : A) B) C) D) Pas nje infeksioni stafilokoksik Pas nje infeksioni nga fruthi Menjher pas difteris Disa jave pas nj infeksioni streptokoksik

24. Gj etja e uremis s lart, por e kreatinins normale tek nj i smur flet pr: A) isuficienc renale obstruktive B) isuficienc renale organike C) isuficienc pre renale

D) asnj nga keto

25. N qoft se glicemia prcaktohet n gj akun e plot venoz do kemi prishje t tolerancs ndaj glukozs kur: A) glicemia esll 126 mg/dl dhe/ ose 2 or pas ngarkess 180mg/dl ose t dyj a B) glicemia esll< 110 mg/dl dhe 2 or pas ngarkese 140mg/dl por < 180 C) glicemia esel 100 mg/dl dhe <110mg dhe 2 or pas ngarkeses <140mg D) asnjra

26. HTA n radh t par dmton: A) B) C) D) arteriet arteriet ar t er i et arteriet veshkore koronare cerbrale periferike

27. Kush nuk sht ndrlikim madhor i HT A? A) B) C) D) infarkti I miokardit insulti cerebral insuficienca kardiake tromboembolizmi sistemik

28. N astmn bronkiale t forms s mesme nuk nevoj itet pr mj ekim: A) B) C) D) B2 mimetik inhalator Korticosteroid parenteral K romoloine Ksantina

29. Cili nga medikamentet e m poshtme antihipertensive jep me shpesh konstipacion: A) B) C) D) propanololi lasiksi izoptina aldometi

30) Femr 30 vjee, paraqitet me dhimbje shpine, n regjionin gluteal dhe n anesin e poshtme. Ajo ishte duke ngritur obj ekte t rnda n shtpi. Ajo qndron e prkulur n njrn an dhe ngritja e kmbs s shtrir sht e kufizuar dhe e dhimbshme. Diagnoza e mundshme sht : A) B) C) D) Mialgji Prolaps i diskut lumbar Stenoz spinale Spondiloz lumbare

31) Mashkull 20 vj e, paraqitet me dhimbje kolike periumbelikale, q zhvendosen drejt foss iliake t djatht, temperatur, humbje oreksi. Diagnoza e mundshme sht : A) B) C) D) Kolecistit akut Divertikulit akut Appendicitis akut Ulcer peptike e perforuar

. 3 2. Grua 47 vj ee, astmatike dhe hipertonike pr t cilin merr methyldopa, ankon pr anoreksi, dhimbje barku dhe ikter q po theksohet. Diagnoza e mundshme sht : A) B) C) D) Pankreatit akut Kolecistit akut Hepatit kronik aktiv Hepatit viral akut

3 3. Grua 78 vj ee me me angin t qndrueshme, 10 dit pas nj operacioni pr fraktur t qafs s femorit ankon pr distension masiv t barkut. Diagnoza e mundshm sht: A) B) C) D) Aneurisma e aorts abdominale Ulcer peptike e perforuar Kolit ulceros Pseudo- obstruksion

34. Burr 60 vje, koht e fundit sht trajtuar pr tuberkuloz t veshks, paraqet rnie n pesh, diarhe, anoreksi, hypotension. Ai paraqet

hiperpigmentim t mukozs s gojs dhe vijave t pllmbve t duarve. Cilin eksaminim do t krkoni pr vendosj en e diagnozs: A) B) C) D) ACTH dhe kortizolin plazmatik G licemi ECHO abdominale Testet e funksionit te tyroides

35. Grua 50 vj ee, ankon pr rnie n pesh, oreks t shtuar, djersitje, palpitacione, preferenc pr mot t ftoht, ka vap, duar t dj ersira si dhe tremor. Cilin ekzaminim do t krkoni pr vendosjen e diagnozs: A) B) C) D) ACTH dhe kortizolin plazmatik H emokultur ECHO abdominale Testet e funksionit t tyroides

36. Burr 65 vje, ka fillim t menj hershm t diabetit, anoreksi, rnie n pesh, dhimbje epigastrike dhe shpine. Cilin eksaminim do t krkoni pr vendosj en e diagnozs: A) B) C) D) Gjak komplet G licemi ECHO abdominale Testet e funksionit t tyroides

37. Grua 70 vj ee, ankon pr rnie n pesh, disfagi progresive, dhe ndj enjn se ushqimet i ngecin n fyt. Cilin ekzaminim do t krkoni pr vendosjen e diagnozs: A) B) C) D) Gjak komplet Skopi me barium ECHO abdominale Testet e funksionit t tyroides

38. Dj al 17 vj e, ankon pr temperatur t lart q dy j av, dhimbje fyti, zmadhim t gj ndrave n qaf. N ekzaminim objektiv ka ndj eshmri n kuadratin e siprm abdomi nal t djatht, subikter n sklera, SGOT dhe SGPT, lehtsisht t rritura. Cilin ekzaminim do t krkoni pr vendosjen e diagnozs: A) Hemokultur

B) Mantuox C) ECHO abdominale D) Monospot (gj ak pr MI)

39. Burr 54 vje, ka 2 muaj q ka zmadhim t tonsils s maj t, temperatur fluktuante, kryesisht mbas dite ,si dhe zmadhim t limfonoduj ve n qaf. Cilin ekzaminim do t krkoni pr vendosj en e diagnozs: A) B) C) D) Gjak komplet Mantuox Biopsi e limfonodulit CT toraksi

40. Femr 25 vj ee, ndihet smur, ka ethe, poliartralgji, eritem nodoze. Radiografia e thoraxit tregon lymphadenopathy mediastinale hilare. Cilin ekzaminim do t krkoni pr vendosjen e diagnozs: A) B) C) D) Mantuox M onospot HIV 1+2 Testi Kvein

41. Mashkull 29 vj e, prdorues droge iv, paraqet temperatur, adenopati cervikale q rrjedh nj substanc t bardh si gjiz. Cilin eksaminim do t krkoni pr vendosjen e diagnozs: A) B) C) D) Mantuox M onospot HIV 1+2 Testi Kvein

42. M 35 vje, ka koll, hemoptizi. Ro: mas n hilusin dex. dhe nj zon konsolidimi n lobin superior t djatht, lateralisht. Diagnoza e mundshme: A) Tbc B) Sarkoidoz C) Metastaz karcinomatoze D) Granuloma e Wegener

43. M 65 vj e, ka koll, dispne. Pulmone me reale bronkiale, subkrepitante, sibila nca. Ro: fibroz pulmonare difuze, trashje dhe kalcifikime pleurale bilaterale. Diagnoza e mundshme: A) Bronshit kronik B) Ca bronkial C) Pneumokonioz D) Astm bronkiale

44. M 14 vje, ka koll dhe infeksione t prsritura pulmonare, gishta tamburi, rnie n pesh, steatorhe. Rodiografi : trashje t mureve bronkiale, hije n form rrethi t bronkektazis si dhe hije t shprndara n pulmone. Diagnoza e mundshme: A) Bronshit kronik B) Pneumoni atipike C) Fibroza kistike D) Astm bronkiale

45. Stj uardes 34 vj ee, ka 7 dit me temperatur, koll t that , dispne, takipne. N auskultim respiracion i pastr. Diagnoza e mundshme: A) grip B) Pneumoni atipike C) Mykoplazma pneumonie D) Astme bronkiale

46. M 65 vje me koll kronike produktive, vjen n urgjenc me dispne, dhe i prgjumur. Pulmone me rale bronkiale, sibilanca, ulje e lvizshmris s pulmoneve . Diagnoza e mundshme: A) Bronshit kronik B) Ca bronkial C) Pneumokonioz D) Astme bronkiale

47. M 55 vj e zgj ohet me dispne, koll me pshtym me shkum, ka kaluar IAM 2 jav m par. Ka takipne, cianotik, rale krepitante n pulmone. Diagnoza e mundshme: A) Edem pulmonare akute B) Trombemboli pulmonare C) Pneumoni atipike D) Astme bronkiale

48. F 25 vj e, ka adenopati n qaf, temperatur, dj ers natn, rnie n pesh. Biopsia e limfonodulit : ka qeliza t mdha me dy brthama, imazhi i pasqyrs, qelizat e Reed- Sternberg. Diagnoza e mundshme A) Mononukleoza infektive B) Leukemi akute C) Morbus Hodgkin D) Sarkoidoze

49. M 70 vje, prdor omnik pr adenomn e prostats. Ka dhimbje mesi dhe kockash, anemi, insuficienc renale. Mielograma ka qeliza plazmatike 38 %. Diagnoza e mundshme A) Metastaza kockore nga ca prostate B) Leukemi akute C) Mielome multiple D) Leukemi mieloide kronike

50. M 18 vje, ka dobsi, zbehje, zverdhje t syve, urinim me ngjyr t errt dhe anemi . Lieni preket 10cm. Gjaku periferik ,Hb 7 gr/dl. N serin e kuqe: hypykromi dhe mikrocitoz e shprehur, ka qeliza target, anizocitosis dhe poikilocitosis t shprehur si dhe retikulocite t larta. Diagnoza e mundshme: A) B) C) D) Hepatit viral akut Leukemi akute Thalasemi maj or D repanocitosis

51) F 35 vj ee, ka anemi, rnie t flokve, ragade n kndet e buzve. Gjaku periferik: hipokromi, mikrocitoz e eritrociteve. Diagnoza e mundshme

A) B) C) D)

Anemi nga mungesa e hekurit Anemi nga mungesae vit.B12 Anemi hemolitike Anemi nga mungesa e acidit folik

52. Burr 75 vje, ka dhibje kockash. Preken, Shpretka 14 cm, hepari 7cm. Gjaku periferik: er 4000000, HB 13 gr/dl, Tb 730.000 mm, leukocite 300.000 mm, Formula: ( bl 3%, PM 10%, MC 17%, MM 20/, SH 10%, Seg 20%, eo7%, Ba 3%, Limfocite 10%. Diagnoza e mundshme A) Leukemi limfoblastike akute B) Mononukleoze infektivi C) Leukemi mieloide kronike D) Leukemi mieloblastike akute

53. Mashkull 23 vj e, ankon pr temperatur 38 grad, dhimbje kockash, vshtirsi n glltitje. Ka angin ulcero-nekrotike, lieni 4 cm. Gjaku periferik anemi, trombocitopeni, leukociti 26.000, ka 34% blaste. N disa blaste ka trupza Auer. Diagnoza e mundshme: A) Leukemi limfoblastike akute B) Leukemi myeloide kronike C) Mononukleoze infektive D) Leukemi myeloblastike akute 54. M 55 vj e, ka dhimbje t fort n sternum dhe marrje fryme. Frekuenca kardiake 40/ min., ECG: dhmb Q dhe rritje t ST n D III, DII, AVF. Mjekimi i mundshm: A) Adalat B) Atropine C) Lanatosid c D) Atenolol 55. M 20 vj e, kthehet nga udhtimi j asht shtetit. Ka dispne dhe shqetsim n gj oks n ann e majt. Radiografia : ka nj zon t vogl n apexin e pulmonit t majt pa ind pulmonar. Diagnoza e mundshme A) Aneurizm disecante e aorts B) Emboli pulmonare C) Pneumotoraks nga tensionimi D) Tbc 56. Grua 60 vj ee ,pasi ngrihet nga banja pati marrje fryme shum t theksuar, dhimbje t fort gj oksi, hemoptizi. ECG takikardi sinusale. Diagnoza e m u n dsh m e: A) Pneumotoraks spontan B) Infarkt akut i miokardit

C) Trombemboli pulmonare D) Me d iastinit 57. Grua 60 vjee me artrit rheumatoid ka splenomegali. N gjakun periferik leukociti 1500 mm3. Diagnoza e mundshme: A) Lymphom malinje B) LMC C) Sindroma Felty D) Ciroz e heparit 58. Mashkulli 60 vj e. Para 15 vitesh ka kaluar hepatit viral B. Paraqitet me hematemez masive dhe gjendje shoku. Ka ascit. Shkaku i hematemezes: A) Ulcer peptike kronike B) Varicet e esofagut C) Cancer i stomakut D) Sindroma Mallory- Weiss 59. Diabetik 60 vj ear , traj tohet me insulin, paraqitet me kruarj e, t prziera, t vj ella. Ai ka edema periferike dhe anemi normokrome. Cilin eksaminim do t kryeni pr diagnoz: A) Azotemi, kreatinine, elektolit B) G licemi C) Calcemi D) ECHO abdominale

60. Grua 50 vjee, njihet se vuan nga karcinoma e gjirit. Ajo paraqet t prziera dhe t vj ella akute, polidipsi, konfusion, prgj umsi. Cilin sht eksaminimi m dallues pr diagnoz: A) Azotemi, kreatinine, elektolit B) G licemi C) Calcemi D) Testet e tyroides

61. Grua 30 vj ee me Morbus Hodgkin, paraqet dobsi me fillim t pa qart, rnie n pesh, t prziera, t vj ella. Ajo ka hiperpigmentim t vij ave t duarve. Cili sht eksaminimi m dallues pr diagnoz: A) Azotemi, kreatinine, elektolit B) T3+T 4+TSH C) Calcemi D) Niveli i kortizolit plazmatik

62. Mashkull 30 vj e, ka vite q n pranver dhe vj esht ka dhimbje barku sidomos nga mesi i nats q e zgjoj n nga gj umi. Aktualisht ka dhimbj e epigastrike shum t fort, t menj hershme, dhimbje n shpatulln e maj t dhe t vjella. Nuk dgj ohen zhurma perisaltike. Cili sht ekzamini m dallues pr diagnoz : A) Azotemi, kreatinine, elektolit B) Eksaminim radiologjik i toraksi n pozicion vertikal C) ECHO abdominale D) Gjak komplet

63. Mashkull 30 vje, ankon pr hemoragji rektale t przier me mu kus, e pa dhimbshme. Ai ka n anamnez diarhe kronike intermitente. N eksaminim nuk ka t dhna pr smundje perianale. Cila sht diagnoza m e mundshme: A) Koliti ulceros B) Smundja divertikulare C) H emorhoidet D) Fistula ne smundj en e Chrons

64. Burr 40 vj e, ankon pr melena t pa dhimbshme. Anusi dhe rektumi j an normal n ekzaminim.Pacienti nuk ka rn n pesh. Ai sht fumator dhe alkolist. Cila sht diagnoza m e mundshme: A) Karcinoma e kolonit B) Smundja divertikulare C) H emorhoidet D) Ulcera peptike

65. Dj al 25 vj e, ankon pr dhimbje anale, shkarkime me gjak dhe mukus. N eksaminimin obj ektiv ka cikatrice t shumta punktiforme rreth anusit. Cila sht diagnoza m e mundshme: A) Koliti ulceros B) Fisura n anus C) Fistula n smunddjen e Chrons D) Karcinoma e rektumit

66. Burr 50 vje, nxjerr 500 ml gj ak t freskt nga rektumi. Ai pati ndj enjn e defekimit, por n vend t feceve nxori gjak. Pacienti ka nj dikomfort kronik n ann e maj t t abdomenit. Nuk ka rn n pesh, nuk palpohet mas. Cila sht diagnoza m e mundshme: A) Karcinoma e cekumit B) Smundja divertikulare C) H emorhoidet D) Ulcera peptike

67. Burr 70 vje, paraqet hemoragji rektale t pa dhimbsh me. Gjaku sht n formn e rremave n fece. Ai ankon pr tnesma. N tushe rektale palpohet nj mas e but, e fiksuar, e pa dhimbshme. Nuk preken limfonoduj inguinal. Cila sht diagnoza m e mundshme: A) Karcinoma e kolonit B) Karcinoma e anusit C) H emorhoidet D) Karcinoma e rektumit

68. Burr 75 vje, ankon pr dhimbje epigastrike dhe defekim me ngjyr t zez (melen) me fece pothuajse t formuara, ka rn n pesh. Ka nj lifonodul t zmadhuar n regionin supraklavikular t maj t. Cila sht diagnoza m e m u n dsh m e: A) Karcinoma e stomakut B) Ulcer e stomakut C) Ulcer e duodenit D) Karcinoma e kolonit

69. Femr 30 vj ee, ankon pr dhimbje shum t fort t menj hershme t shpins . Ajo udhton me skolioz kompesatore. N eksaminim, ajo ka dhimbj e nga gluteusi tek kyci i kmbs, humbje t ndjeshmris s shputs , dhe pulps s kmbs s maj t. Cila sht diagnoza m e mundshme: A) Mieloma multiple B) Stenoza spinale C) Prolaps lumbar dhe sciatica D) Sp ondilo lis tes is

70) Burr 50 vj e, ankon pr dhimbje t shpins q prhapet drejt t dy ansive. Dhimbja prkeqsohet nga ecja dhe qetsohet nga pushimi dhe prkulja prpara. N eksaminim, ai ka kufizim t ngritjes lart t kmbs s shtrir dhe munges t reflekseve t kyceve t kmbs. Cila sht diagnoza m e mundshme: A) Osteoart rit B) Stenoza spinale C) Prolaps lumbar dhe sciatica D) Spondiliti ankilozant

71. Grua 50 vj ee, ankon pr dhimbje shpine. Ajo ka anemi normokrome, normocitare me sediment shum t lart. Proteina totale 11 gr/dl. Cila sht diagnoza m e mundshme: A) Mieloma multiple B) Stenoza spinale C) Prolaps lumbar dhe sciatica D) S pondiloarthrosis

72. Burr 67 vj e, ka dhimbje mesi sidomos gjat ecjes. sht i zbeht, ka azotemi t lart. Proteinuria Bence -Jonce pozitiv. Cila sht diagnoza m e mundshme: A) Mieloma multiple B) Stenoza spinale C) Prolaps lumbar dhe sciatica D) Metastaza kockore

73. Mashkulli 60 vje, ankon pr dhimbje t kockave, t shpins n regionin lumbar dhe n kofsht e tij. Fosfataza alkaline serike 1000 u/L. Calciumi dhe phosphori jan normal. Pacienti ka vshtirsi me dgjimin. Cila sht diagnoza m e mundshme: A) Mieloma multiple B) Stenoza spinale C) Prolaps lumbar dhe sciatica D) Smundja e Paget

74. Mashkulli 50 vj e, ankon pr dhimbje t forta t kockave sidomos t shpins q e zgjojn nga gjumi. Ka t prziera, t vj ella. Obj ektivisht, n palpim epigastri duket i zn. N eksaminimin radiologjik duke dmtime osteolitike t vertebrave. Fosfataza alkaline dhe Eritrosedimenti jan i lart. Cila sht diagnoza m e mundshme: A) Mieloma Multiple B) Stenoza spinale C) Metastaza n kocka D) Sp ondilo lis tes is

75. Mashkulli 68 vj e, ankon pr dhimbje shpine dhe kockash. Ka djegie gj at urinimit dhe vshtirsi n urinim. Urina komplet nuk ka shenja t infeksionit. Fosfataza alkaline e lart. PSA e rritur. Gjaku komplet: leuko-erithroblastik, seddiment i lart. Cila sht diagnoza m e mundshme: A) Metastaza kockore nga ca prostate B) Mielom multiple C) Prolaps lumbar dhe sciatica

D) Smundja e Paget

76. Femra 50 vjee ankon pr episode t prsritura t dobsis s papritur n ansit e poshtm q shoqrohet m rrzim, por nuk e humbet koshiencn. Zgj idh eksaminimin m dallues pr diagnoz: A) glicemi B) ECG me 12 lidhje C) Elektrolitt n gj ak D) CT e koks

77. Burr 60 vje, paraqet tremor, djers, dhe gjendje t fikti n mngjes. Zgjidh eksaminimin m dallues pr diagnoz: A) glicemi B) ECG me 12 lidhje C) Tensioni arterial D) CT e koks

78. Femra 65 vj ee me insuficienc renale kronike, paraqet kriza tetanie dhe t fikt. Ajo ankon pr krampe n ansi dhe mpirje rreth gojs. Zgjidh eksaminimin m dallues pr diagnoz: A) glicemi B) ka lcemi C) gjak komplet D) urin komplet

79. Femra 65 vj ee paraqet marrje mendsh dhe errsim t pamjes. Pulsi 50/1min. Zgj idh eksaminimin m dallues pr diagnoz: A) Glicemi B) ECG me 12 lidhje C) Elektrolitt n gj ak D) CT e koks

80. Femra 70 vj ee mjekohet me levodopa pr Parkinson, ankon pr gj endje t fikti kur ngrihet natn. Zgj idh eksaminimin m dallues pr diagnoz: A) glicemi B) h emo ku ltur C) tensioni arterial D) elektrolitt n gj ak

81. Femra 40 vj ee ka fibrom t ovarit, hidrotoraks t djatht dhe ascites. Cila sht diagnoza m e mundshme: A) Sindromi Budd-Chiari B) Sindromi Meigs C) Pseudomiksoma peritoneale D) Peritonit tuberkular

82. Femra 25 vj ee ka t prziera, t vjella, dhimbje abdominale. Objektivisht ka hepatomegali t dhimbshme dhe ascit. Koht e fundit ajo ka filluar prdorimin e kontraceptivve oral. Cila sht diagnoza m e mundshme: A) Sindromi Budd-Chiari B) Sindromi Meigs C) Tromboz e vens porta D) Peritonit tuberkular

83. Mashkulli 30 vj e, zhvillon ascites dhe dhimbje kolike n kuadratin e dj atht t poshtm. Likidi ascitik ka natyr viskoze dhe mu koide. Cila sht diagnoza m e mundshme: A) Pamj aftushmri e zemrs s djatht B) Shtypja e vens porta nga limfonodujt C) Pseudomiksoma peritoneale D) Peritonit tuberkular

84. Fe mra 40 vj ee ka ascit. N anamnez, ka kaluar ethe rheumatizmale. N auskultim, ka nj ton t dyt pulmonar t lart. Volumi i pulsit n arteriet periferike i ult. Cila sht diagnoza m e mundsh me: A) Pamj aftushmri e zemrs s djatht si pasoj e stenozs mitrale B) Shtypja e vens porta nga limfonodujt C) Cirrhosis D) Peritonit tuberkular

85. Femra 50 vj ee ka ascites, dobsi. Ajo ka puls t crregullt, t shpej t, me volum t vogl. Eksaminimi radiologjik tregon zemr t vogl me kalcifikime q duken n proj eksionin lateral. ECG demonstron QRS me voltazh t ult dhe val T t invertuara. Cila sht diagnoza m e m u n dsh m e: A) Pamj aftushmri e zemrs s djatht si pasoj e stenozs mitrale B) Perikarditi constriktiv C) Cirrhosis

D) Peritonit tuberkular

86. Femra 40 vj ee pas lvizjes s nj tavoline t rnd ka dhimbje t shpins; ajo nuk ka shenja t prekjes s rrnjs nervore. Cili sht trajtimi m i p rs htats h m : A) Referim urgj ent te ortopedi/ neurukirurgu B) Radiografi toraksi C) Regj im shtrati pr dy jav D) Dy dit pushim + analgjezik

87. Fe mra 50 vj ee ankon pr dhimbje shpine q 2 muaj, q prkeqsohen natn. Radiografia tregon fraktur kompresive t dy vertebrave pa shkak traumatik. Cila sht trajtimi m i prshtatshm: A) Referim urgj ent te ortopedi pr dekomrimim nervi B) Eksaminime t mtej shme pr tumor apo pathologji t tj era kockore C) Regj im shtrati pr dy jav D) Fizioterap i

88. Mashkulli 60 vj e, ka dhimbje shpine q prhapet posht t dy gj unjve bilaterali sht. N eksaminim, ka anestesi t shals, inkontinenc urine dhe humbje t tonusit anal. Cili sht trajtimi m i prshtatshm: A) Referim urgj ent tek neurokirurgu pr dekomprinim kirurgj ikal urgj ent B) Eksaminime t mtejshme pr tumor apo pathologji t tj era kockore C) Referim rutin te neurokirurgu/ ortopedi D) ECHO abdominale

89. Femra 50 vj ee ankon pr dhimbje kronike mesi q prhapen drejt gluteuseve.Vjen shpesh n ambulanc pr dhimj en e mesit. Nuk ka prekj e t rrnjs nervore. Cila sht trajtimi m i prshtatshm: A) Referim urgj ent tek neurokirurgu B) Eksaminime t mtejshme pr tumor apo pathologji t tj era kockore C) Referim rutin te neurokirurgu/ ortopedi D) Fizioterap i

90. Mashkulli 30 vje, ankon pr dhimbje shpine q prhapet posht gj urit. N eksaminim ka humbje t ndj eshmris lateralisht pulps s djatht dhe medialisht kmbs s djatht. Nuk bn dot dorsifleksionin e gishtit t madh. Ai ka qndruar regjim shtrati pr 6 jav. Cila sht trajtimi m i p rs htats h m : A) Referim urgj ent tek neurokirurgu

B) Eksaminime t mtej shme pr tumor apo pathologji t tj era kockore C) Referim rutin te neurokirurgu/ ortopedi D) Fizioterap i

91. Mashkulli 26 vj e, ka nj fryrje (xhung) t pa dhimbsh me q 6 j av n testin e majt.Nuk ka adenopathi inguinale. Alfa fetoproteina sht e lart. Cila sht diagnoza m e mundshme A) Torsion i testit B) Hydrocele C) Tumor i testit D) Epididimo-orkit akut

92. Fmij 6 vj e, ka hematuri t padhimbshme, afebril, dhe edem skrotale q 2 dit. Porcioni i mesit t urins ka mbetje granulare. Cila sht diagnoza m e mundshme A) Isuficienc renale kronike B) Pielonefrit akut C) Nekroz tubulare akute D) Epididimo-orkit akut

93. M 70 vje, ka paksim t urinimit, urinim natn, sht i zbeht, ka edema n anesi, ascit, TA 170/100 mmHg. Cila sht diagnoza m e mundshme A) Isuficienc renale kronike B) Karcinom e prostats C) Nekroz tubulare akute D) Karcinom e veshks

94. M 75 vj e, ankon pr urinim t shpesht natn, dhimbje shpine. Preket glob vezikal, dhe n tushe rektale prostat e zmadhuar. Fosfataza alkaline dhe acide n serum j an t rritura. Cila sht diagnoza m e mundshme A) Insuficienc renale kronike B) Karcinome e vezi kes C) Karcinom e veshks D) Karcinom e prostats

95. Fmij 12 vje paraqet njtje, skuqj e, dhimbje t skrotumit. Porcioni i mesit t urins sht normal. Cila sht diagnoza m e mundshme A) Torsion i testit B) Hydrocele C) Tumor i testit

D) Epididimo-orkit akut

96. Femij 10 vj e, ka stridor. Ai ka histori t njtjes s duarve dhe kmbve si dhe diarhe. T njjtn histori e ka edhe e motra. Cila sht diagnoza m e m undsh m e A) Ethe rheumatizmale akute B) Sindrom nefrotik kongenital C) Angioedema hereditare D) Insuficienca kardiake kongestive

97. Vaj z 6 vj ee, ka temperatur q ngrihet e ulet (temp. me maj ), enjtje t artikulacioneve t gishtave. Gishtat e fryr kan marr form boshti. Cila sht diagnoza m e mundshme A) Ethe rheumatizmale akute B) Sinovit toksik C) Angioedema hereditare D) Artrit rheumatoid juvenil

98. Dj al 12 vje, paraqet poliartrit dhe dhimbje abdominale. Ai kishte bajamet e smura rreth 2 jav para. N buzn e ans s majt t sternumit dgj ohet zhurm e hershme diastoloke shfryrse. Cila sht diagnoza m e m undsh m e A) Ethe rheumatizmale akute B) Sindrom nefrotik kongenital C) Arthrit rheumatoid forma j uvenilde D) Insuficienca kardiake kongestive

99. Vajz 12 vjee, ka zbehj e, dispne, puls 190/1. Ajo ka hepatomegali dhe cardiomegali, edema n ansi. Cila sht diagnoza m e mundshme A) Perikardit B) Sindrom nefrotik kongenital C) M yokardit D) Insuficienca kardiake kongestive

100. Femr 18 vj ee, nulipare, 12 javshe; do t prfundoj shtatzanin. Zgjidh trajtimin m t mir: A) histerectomi e thjesht B) K olposkopi C) Dilatacion dhe kyretazh D) Antibiotik iv

101. Plak 70 vjee, ka 3 muaj me hemoragji vaginale. Histeroskopia dhe biopsia e endometrit konfirmoj n adenokarcinom . Zgj idh trajtimin m t mir: A) histerectomi e thjesht B) K olposkopi C) Dilatacion dhe kyretazh D) Histerektomi totale abdominale, salpingo- oophoroectomy bilaterale

102. Grua 30 vjee, vjen n ambulanc m dhimbje t fort n ann e maj t t barkut dhe shpatulls s majt. Para dy muaj sh ka ndaluar kontraceptivt dhe ka dy muaj pa cikl menstrual. TA 90/50 mmHg, pulsi 120/1. Zgjidh trajtimin m t mir: A) histerectomi e thjesht B) Reanimim urgj ent dhe loparatomi. C) Dilatacion dhe kyretazh D) Histerektomi totale abdominale, salpingo- oophoroectomy bilaterale.

103. Grua 30 vj ee, nulipare, ankon pr infertilitet. Ajo ka perioda t rregullta, zgj atin 8 dit e me sasi t shtuar. Ajo dhe bashkshorti kan nj vit q tentojn pr shtatzani. Burri ka br vizit te urologu dhe nuk ka probleme dhe ajo krkon t dij n se defekti sht te ajo. ECHO tregon fibrinoid uterin subseroz. Zgj idh trajtimin m t mir: A) Laparaskopi B) Reanimim urgj ent dhe loparatomi. C) Dilatacion dhe kyretazh D) Testi Papanicolou

104. Grua 28 vjee, ka eksaminim jo normal t qafs s mitrs. Ajo ka condilom dhe displazi t moderuar. Zgj idh trajtimin m t mir: A) Colposkopi e ndjekur nga krem 5- Fluouroucil apo excizion me diatermi B) Antibiotik iv+ flagyl ovul C) Dilatacion dhe kyretazh D) Cone biopsi

1 05. Fmij dy vj e, ka anemi, dhimbje barku. Nna thot se e ka par t grvisht muret dhe ka frik se ha gelqere nga suvaja. Diagnoza m e m undsh m e A) Skizofreni B) Delirium

C) Fuga D) Pica

106. Grua 23 vjee, ka frik t lr shtpin e vet. Ajo funksionon (sillet) normalisht vec mos daljes jasht shtpis s saj. Diagnoza m e mundshme A) S kizofreni B) A garaphobia C) Delirium D) A lgophobia

107. M 18 vje, ka diarhe, dj ersitje, t prziera, pupila t zgj eruara, ngritje t TA. Ai ka qn prdorues dhe ka pasur vartsi nga drogat. Diagnoza m e m undsh m e A) Intoksikim nga cocaina B) Delirium C) Lnia e opoidve (kriz abstinence) D) Intoksikim nga drogat

108. F 30 vj ee, sht gjetur n nj gj endje amnesie. Burri i saj referon se ka disa dit q ka humbur, q kur mori letrn e divorcit. Diagnoza m e m undsh m e A) S kizofreni B) Fuga C) Delirium D) Intoksikim nga alkoli

109. Mashkulli 30 vj e, me crregullime bipolare nn mj ekim me litium. Koht e fundit mj eku i familjes i ka nisur hydrochlorthiasid pr nj tension arterial t moderuar. Aktualisht ai sht konfus, me ataxi, mjegullim t pamjes, tremor jo fin. Diagnoza m e mundshme A) Intoksikacion nga cocaina B) Lnia e alkolit C) Perdorim opiatesh D) Intoksikacion medikamentos

110. F 28 vj ee, ankon pr dhimbje t fundit t barkut. Obj ektivisht ka shum cikatrikse kirurgj ikale n abdomen. Obj ektivisht, abdomeni dhe pelvisi normal. Ajo kmbngul se ka nevoj pr laparaskopi. Diagnoza m e m undsh m e A) Skizofreni

B) Sindromi Munchausen C) Delirium D) Neuroz histerike

111. M 54 vje, skizofrenik, ka nisur haloperidol. Nj muaj pas fillimit ai ka jarg, ecn duke hequr kmbt zvarr. Ai vuan gj ithashtu nga lvizj e prtypse t pa vullnetshme. Diagnoza m e mundshme A) Efekte ansore autonomike t medika mentit B) Efekte ansore antikolonergj ike t med. C) Efekte extrapiramidale t meddikamentit D) Neurosis

112. Femra 40 vj ee ankon pr tharje t goj s, turbullime t pamj es, konstipacion. Objektivisht ka zgjerim t pupilave. Ajo ka nisur amitiptilin pr depresion madhor. Diagnoza m e mundshme A) Efekte ansore autonomike t medika mentit B) Efekte ansore antikolonergj ike t med. C) Efekte extrapiramidale t meddikamentit D) Neurosis

113. M 45 vje, ka dhimbje koke, etje t theksuar, urinim t shpesht. Ai ka pucrra (akne t theksuara), lkur t ashpr, strum. Diagnoza m e m undsh m e A) H ypothyroidis m B) Sindrom Cushing C) A kromegali D) Neurosis

114. Mashkulli 30 vj e, paraqitet tek mj eku i familj es se ka nxjerr shpatulln. N eksaminim, shpatulla sht n rregull. Nuk ka dislokacion t saj, por pacienti kmbngul se e ka t nxjerr. Diagnoza m e mundshme A) Depresion B) Simulant C) S kizofreni D) Neurosis

115. M 25 vje, ka 5 dit me temperatur t lart, dhimbje koke sidomos n regionet temporale, diarhe, njolla roz n trunk. Ju dyshoni pr tifo. Cilin eksaminim do t krkoni pr diagnoz: A) Widal B) ELISA C) Weil-Felix D) Paul Bunnell

116. Blegtor nga Saranda, ka 12 dit me temperatur ondulante, dj ers t shtuara, me er t rnd, dhimbje kockash. Lieni 2 cm , hepari 2 cm. Gjaku periferik leukocite 3800 mm3 , deviacion i majt, lifo- monocitosis relative. Cilin eksaminim do t krkoni pr diagnoz: A) Widal B) Wright C) Weil-Felix D) Paul Bunnell

117. M 23 vje, paraqet rash makulopapular n shputat e kmbve dhe papula anale. Cilin eksaminim do t krkoni pr diagnoz: A) Widal B) Wright C) VDRL (Slide test i laboratorit t smundj eve veneriane D) Paul Bunnell

118. Mashkulli 30 vj e, ankon pr ethe, zverdhje, dhimbje t pulpave. Ai sht i apasionuar pas notit. Ju dyshoni pr smundjen e Weil. Cilin eksaminim do t krkoni pr diagnoz: A) Widal B) Wright C) Testi i aglutinimit Schuffner D) ASO

119. F 20 vj e, ankon pr dhimbje fyti, temperatur, adenopati n qaf, petikie n palatum. Cilin eksaminim do t krkoni pr diagnoz: A) Widal B) Wright C) Weil-Felix D) Paul Bunnell

120. Mashkulli 60 vje, ka dhimbje gj oksi, dispne. ECG: bradikardi sinusale, 45 rrahje/1min. Zgj idh trajtimin m t prshtatshm: A) Atropin iv B) Pacemaker extern C) Pacemaker transvenos D) Oksigj eno terapi

121. Mashkulli 60 vj e, pson kolaps n rrug. Nuk e ka par njeri si ndodhi. Aktualisht ai nuk ka puls. Zgj idh trajtimin m t prshtatshm: A) Atropin iv B) CRP (Ringjallje zemr- mushkri), adrenalin iv 1:1000 push C) CRP (Ringj allje zemr- mushkri) derisa t jet gati defribilatori. D) Oksigj eno terapi

122. F 55 vj ee, ka ritm t ngadalt, por sht asimptomatike. ECG tregon mos prputhje midis ritmit atrial dhe ventrikular. Ritmi ventrikular sht 40 rrahje/ min. Komplekset QRS j an t zgj eruara. Zgj idh trajtimin m t p rs htats h m : A) Atropin iv B) Godditje me grusht n sternum C) Pacemaker transvenos D) Defibilim me 200 J oules

123. Mashkulli 30 vj e q praktikon numra shpejtsie, sht gjetur pa koshienc n sken. N ambulanc ai bn frymmarrje n mnyr spontane. Monitori ECG tregon ritm t crregullt ku nuk dallohen valt P,QRS, ST, T. Ritmi sht i shpejt. Zgj idh trajtimin m t prshtatshm: A) Atropin 1mg iv push B) CRP (Ringjallje zemr- mushkri), adrenalin iv 1:1000 push C) CRP (Ringj allje zemr- mushkri) derisa t jet gati defribilatori. D) Oksigj eno terapi

124. M 53 vje, vj en n ambulanc me dhimbje shum t fort gj oksi. Ai ka pasur histori t angins. Dhimbja nuk qetsohet nga trinitrina. TA 120/70mmHg, ritmi 100 rrahje 1/min. ECG: ritm sinusal. Zgjidh trajtimin m t prshtatshm: A) Atropin 1mg iv bolus B) Morfin im + Oksigj eno terapi 4 LPM C) CRP (Ringjallje zemr- mushkri), adrenalin iv 1:1000 push D) Lignocaine iv

125. Mashkulli 50 vj e, vjen n ambulanc se ka infarkt akut t miokardit. P a pritur, TA bie 70/45mmHg. N t dy bazat pulmonare dgj ohen rale. Zgjidh trajtimin m t prshtatshm: A) Atropin 1mg iv bolus B) Morfin im + Oksigj eno terapi 4 LPM C) CRP (Ringjallje zemr- mushkri), adrenalin iv 1:1000 push D) Infuzion Dopamin

126. Mashkulli 50 vje, n ECG (monitor)ka takikardi paroksismale supraventri kul are. Zgjidh trajtimin m t prshtatshm: A) Lignocain B) Masazh i sinusit carotid C) Trinitrin D) A ddrena lin

127. Mashkulli 50 vj e, vjen me dispne.Pulsi sht i crregullt n frekuenc dhe ritm. Skopia tregon cardiomegali dhe zmadhim t atriumit t maj t. ECG: fibrilacion atrial me frekuenc 180/min. Zgj idh trajtimin m t p rs htats h m : A) Antikoagulim + digitalizim B) p r opr an olol C) Atropine D) Lignocaine iv

128. M 75 vje, humb pa pritur koshiencn n ambulanc. ECG tregon takikardi ventri kulare. Zgj idh trajtimin m t prshtatshm: A) Atropin 1mg iv bolus B) Morfin im + Oksigj eno terapi 4 LPM C) Zgjidh trajtimin m t prshtatshm: D) Convertim cardiak me Defibrilator me 200 j oules

129. F 20 vj ee, ka diare rekurente me gj ak dhe krampe abdominale. Sigmoidoskopia dhe biopsia konfirmojn diagnozn: kolit ulceros. Zgjidh mj ekimin m specifik A) M esalazin B) likide iv, antibiotik iv dhe analgj ezik C) Panproctocolectom y D) Subtotal colectomy, fistul mukozale dhe ileostomi permanente

130. M 25 vje, gjat brjes t nj numri shpejtsie ka psuar nj traum t topitur n pjesn e siprme t abdomenit. Ai ka dhimbje n shpatulln e maj t dhe dhjimbje abdominale difuze. Ai paraqet takikardi n rritj e, hypotension dhe shfaqen shenja peritoneale. Zgj idh mj ekimin m specifik A) L aparatom i B) Likide iv, antibiotik iv dhe analgjezik C) Panproctocolectom y D) Cholecystectomy me laparoskop

131. Femra 40 vj ee obeze ankon pr dhimbje n kuadratin e siprm t djatht, temperatur t lart. Leukocite 14 000mm3 me deviacion t majt e neutrofiloz. Eksaminimi radiologjik i pulmonit dhe abdomenit sht negativ. Zgjidh mjekimin m specifik a) L aparato m i b) Likide iv, antibiotik iv dhe analgjezik c) Panproctocolecto m y d) Cholecystectomy me laparoskop

132. Femra 50 vj ee ka ethe, dhimbje n kuadratin e siprm t djatht t abdomenit, ikter. ECHO: dilatacion i duktusit biliar komun. Zgjidh mj ekimin m specifik A) ERCP dhe sfinkterotomi endoskopike B) Likide iv, antibiotik iv dhe analgjezik C) Panproctocolectom y D) Cholecystectomy me laparoskop

133. Mashkulli 30 vj e, ka dhimbje shum t forta abdominale t pa dominuara. Ai ka temperatur, takikardi dhe distension t shprehur t barkut. N eksaminimin radiologj ik koloni transvers ka diametr 7 cm.( M egakolon toksik). Zgj idh mjekimin m specifik A) M esalazin B) likide iv, antibiotik iv dhe analgj ezik C) Panproctocolectom y D) Subtotal colonectomy, fistul mukozale dhe ileostomi per manente

134. M 40 vje, ankon pr dhimbje t penisit. Objektivisht, lafsha sht e trhequr pas glans penis dhe ka enjtje glandulare. Diagnoza m e m u n dsh m e: A) Smundja Peroni ( Peyronies)

B) Paraphimosis C) K ondilo m a D) Priapism

135. Fmij 12 vje, me histori t infeksioneve urinare t prsritura, aktualisht ka vshtirsi n urinim. Objektivisht, hapja e lafshs sht e ngusht dhe e skuqur. Diagnoza m e mundshme: A) Smundja Peroni ( Peyronies) B) P himosis C) K ondilo m a D) Priapism

136. Mashkulli 50 vj e me insuficienc renale kronike, trajtohet me dializ; vj en tek mj eku i familjes se ka dhimbje t penisit. Objektivisht penisi sht n ereksion: trupat kavernoz jan erekt, corpus spongios sht i flashkt. Diagnoza m e munddshme: A) Smundja Peroni ( Peyronies) B) Paraphim osis C) Balanitis xerotica obliterans D) Priapism

137. M 40 vje, me tyroidit Reidel, ankon se kontakti seksual sht i dhimbshm.Ai ka sj ell nj foto ku duket curvatura e penisit n ereksion. Diagnoza m e munddshme: A) Smundja Peroni ( Peyronies) B) Paraphim osis C) K ondilo m a D) Priapism

138.M 52 vje, vj en tek mjeku sepse nuk arrin dot ereksion. Koht e fundit ai ka br prostat ectomi totale. Diagnoza m e munddshme: A) Smundja Peroni ( Peyronies) B) Paraphim osis C) I m potenc D) Priapism

139. F 30 vj ee, paraqet nj mas 3 cm, t fort, t lmuar, t pa adderuar me lkurn.Citologjia realizuar me citopunksion t drejtuar nuk ka qeliza malinje. Diagnoza m e mundshme A) Abces i gjirit

B) Fibroadeno m a C) Displazia mamare beninj e D) Dukt ektasia

140. F 20 vj ee, ankon se para ciklit ka gj ndra n gjinj dhe ka dhimbje t gj oksit. N eksaminim gj okset jan t ndjeshm n kuadratet e jashtme me disa nodularitete. Citologj ia me punksion t drejtuar tregoi fibrosis, adenosis dhe ndryshime cistike. Diagnoza m e mundshme A) Smundja cistike B) Fibroadeno m a C) Displazia mamare beninj e D) Dukt ektasia

141. Femra 50 vj ee paraqet nodozitete t vogla t lmuara diskrete t gj irit. N aspirim doli lng i verdh. Citologjia nga Citopunksioni i drejtuarnuk ka qeliza malinje. Diagnoza m e mundshme A) Smundja cistike B) Fibroadeno m a C) Displazia mamare beninj e D) D uktektasia

142. Femra 50 vj ee ankon pr rrjedhje t vazhdueshme si gjiz nga thithi. Ajo ka vn re trheqje t thithit por nuk ka kokrr. Mamografia tregon ntrashje t duktusit. Diagnoza m e mundshme A) Smundja cistike B) Fibroadeno m a C) Displazia mamare beninj e D) Dukt ektasia

143. Femra 65 vj ee ka karcinom t gj irit; her mbas here ka dhimbje barku. Ajo eleminon sasi t mdha urine dhe pi uj m shum se za konisht. Diagnoza m e mundshme: A) H ypercalce m i B) Diabetes mellitus C) Ankth D) Fibroz post raddioterapie

144. Mashkulli 26 vj e, vjen n urgjenc me dhimbje shum t forta n ije q kan 3 or. Eksaminimi i urins tregon hematuri. Diagnoza m e mundshme:

A) B) C) D)

Hypercalcemi cistitis Pielonefrit Kolik renale

145. F 33 vj ee, krkon vizit shtpie. Ajo flet pr disuri pr disa dit, kurse tani ka ethe, dhimbje n ij en e djatht. Diagnoza m e mundshme: A) H ypercalce m i B) cistitis C) Pielonefrit D) Kolik renale

146. M 38 vje, vj en pr kontroll t zakonshm. Ai ndihet mir. BMI e tij sht 2 4. TA 124/80 mmHg. Ai prmend se nna e tij u diagnostikua si diabetike kur ishte 76 vje. Zgj idh trajtimin m t prshtatshm: A) T filloj Metformin B) T filloj hydrochlorthiasid C) T filloj prep. t sulphanylures D) Kshillim dietetik

147. Mashkulli 60 vj e, vjen se ka nj glicemi t rastsishme 170mg/ dl dhe sht i shqetsuar nse sht apo jo diabetik. Zgj idh trajtimin opsionin m t p rs htats h m : A) Testi i tolerancs s glukozs. B) T filloj hydrochlorthiasid C) T filloj sulphanyluren D) Kshillim djetetik

148. Diabetik 60 vj e; mer metformine 3x 0.5 n dit.HbA1c vjetore sht 8.8 dhe ju vini re se vitin e kaluar, n kohn kur nisi metforminn, ajo ishte 8.5. Pacienti referon se zakonisht nuk harron t marr mj ekimin. Zgj idh trajtimin m t prshtatshm: A) T filloj insulinn B) T filloj hydrochlorthiasidin C) T filloj prep. t sulphanylures (daonil) D) Kshillim djetetik

149. Diabetik 58 vje. Ai e ka diabetin nn kontroll me dj et. Aktualisht paraqet TA 155/94 mmHg. Zgj idh trajtimin m t prshtatshm: A) T filloj Metformin

B) T filloj hydrochlorthiasid C) T filloj preparate t sulphanylures D) T filloj glitazon

150. M 56 vje jo insulinovarts, trajtohet me glibenclamid. Ai ka rritje t moderuar t transaminazave si pasoj e prdorimit kronik abuziv t alkoolit, por aktualisht e ka ln dhe hepari sht stabl. HbA1c sht duke u ngritur lart, (9,2) megj ithse merr dozn maksi male t glibenclamidit. Zgjidh trajtimin m t prshtatshm: A) T filloj insulinn B) T filloj hydrochlorthiasidit C) T filloj sulphanyluren (daonil) D) Kshillim djetetik

151. Vaj z astmatike 5 vj ee ,ndihet mir duke prdorur 100g beclomethason bd dhe salbutamol. Ajo ka nj infeksion t siprm respirator, por rrjedha kulmore e ajrit t saj( peak flo) vazhdon t jet 80% e normales. Zgj idh trajtimin m t prshtatshm A) T vazhdoj t njjtin trajtim B) T filloj 2- agonistt me veprim t zgj atur C) Krkohet t filloj 2- agonistt D) T filloj steroidt oral

152. M 25 vje, prdor 200g beclomethason bd dhe salbutamol sipas nevojs pr t kontrolluar astmn. Muajt e fundit ai e ka prdorur salbutamolin ditn dhe natn e ka zgj uar nga gj umi kolla e that. Zgj idh trajtimin m t p rs htats h m A) T vazhdoj t njjtin trajtim B) T filloj 2- agonistt me veprim t zgj atur C) Krkohet t filloj 2- agonistt sipas nevoj s D) T filloj steroidt oral

153. Vaj z 15 vjee, sht trajtuar pr astmn e provokuar nga ushtrimet me terbutalin sipas nevoj s. Ajo e prdor at mesatarisht 4 her n jav. Zgj idh trajtimin m t prshtatshm A) T vazhdoj t njjtin trajtim B) T filloj 2- agonistt me veprim t zgj atur C) Krkohet t filloj 2- agonistt D) T filloj steroidt oral

154. M 51 vje, prdor 800g beclomethason dita dhe eformoterol bd ai vj en te mj eku sepse ka koll produktive me sputum mukopurulent, pr t ciln ka nisur antibiotik, si dhe dispne n sforcim t vogl.Rrjedha kulmore e ajrit ( peak flo i tij) sht 50% e vlers s preddikuar. Zgj idh trajtimin m t p rs htats h m A) T vazhdoj t njjtin trajtim B) T filloj 2- agonistt me veprim t zgj atur C) Krkohet t filloj 2- agonistt D) T filloj steroidt oral

155. Djal 14 vj e, ka filluar t ket pak Weezing(frym marrje me fishkllima) kur ai merr pjes n gara sportive. Ai ka koll t zgjatur pas virozave n dimr, por sidoqoft ai ndihet mir. Zgjidh trajtimin m t p rs htats h m A) T vazhdoj t njjtin trajtim B) T filloj 2- agonistt me veprim t zgj atur C) Krkohet t filloj t prdor 2- agonistt sipas nevoj s D) T filloj steroidt oral

156. Fmij 3 vje, vetm nga nj ra vrim e hunds ka rrjedhje me er t keqe e t gj akosura. Cila sht diagnoza m e mundshme A) Trup i huaj B) Rinit alergj ik C) Hematom septale D) Rinit medikamentos

157. M 19 vje, sht goditur n hundd duke lozur si portier. Ai i ka t dy hundt e bllokuara. N eksaminim ka nj fryrje t kuqe t celur n linjn mediane q dallohet nga t dy vrimat e hunds. Cila sht diagnoza m e m undsh m e A) Trup i huaj B) Rinit alergjik C) Hematom septale D) Rinit medikamentos

158. M 43 vje ,vuan nga bllokimi kronik i hudve dhe rrjedhje. Ai ka prdorur shum medika mente spray pr muaj t tr, por sheh q gj ndja sht duke u prkeqsuar. Cila sht diagnoza m e mundshme A) Trup i huaj B) Rinit alergjik C) Hematom septale

D)

Rinit medikamentos

159. M 78 vje, ka disa muaj me bllokim vetm t ans s dj atht t hunds t shoqruar me rrjedhje t gjakosura. Cila sht diagnoza m e mundshme A) Trup i huaj B) Rinit alergjik C) Hematom septale D) C arcinoma

160. M 21 vje, ankon se cdo vit, n t njjtn periudh t vitit ka bllokim t hundve shoqruar me rrjedhje t ujshme t shumta. Cila sht diagnoza m e mundshme A) Rinit vazomotor B) Rinit alergjik C) Hematom septale D) Rinit medikamentos

161. F 30 vj ee, ankon pr lodhje q ka ardhur duke u shtuar. Ajo ka 6 muaj me menstruacione t shtuara q kur lindi fmijn e dyt. Cili sht shkaku m i m undsh m A) Anemia B) Pamjaftueshmria e zemrs C) Dobsia akute post virale D) M alinjitet

162. Grua 82 vjee, ka 6 jav me dobsi, rnie n pesh, diarhe. Cili sht shkaku m i mundshm A) H ypotyroidism B) Pamjaftueshmria e zemrs C) Dobsia akute post virale D) M alinjitet

163. Studente 19 vj ee, ka lodhje t theksuar dhe vshtirsi prqndrimi q nga koha kur kaloi nj infeksion n fyt, 2 muaj m par. Cili sht shkaku m i m undsh m A) H ipertyrosis B) Pamjaftueshmria e zemrs C) Dobsia akute post virale D) Malinjitet

164. F 73 vj ee, obeze, ka 2-3 muaj me loddhje, marrje fryme n sforcim. Aj o ka edema t lehta rreth kyceve t k mbve si dhe rale inspiratore krepitante n t dy bazat e pulmonit. Cili sht shkaku m i mundshm A) Isuficienca renale B) Pamjaftueshmria e zemrs C) Ciroz hepatike D) Miksed ema

165. F 23 vj ee, ankon pr fryrj barku dhe dhimbje q qetsohen pas defekimit. Pacientja defekon vetm ditn. Nuk ka rn n pesh. Cila sht diagnoza m e mundshme: A) Smundja inflamatore e zorrs B) Sindromi i zorrs s irritushme C) H iperthroidizmi D) Smundja e Celiakis

166. Femra 25 vj ee ankon pr diarhe q 4 muaj, ka rn n psh, ndjen vap, ka pagj umsi, tremor. Cila sht diagnoza m e mundshme: A) Smundja inflamatore e zorrs B) Sindromi i zorrs s irritushme C) H iperthroidizmi D) Smundja e Celiakis

167. F 23 vj ee, ka dirhe, rnie n pesh pr pak muaj. Ajo ka nj rash pruriginoz n t dy brrylat. Cila sht diagnoza m e mundshme: A) Malinjitet colo-rectal B) Sindromi i zorrs s irritushme C) H iperthroidizmi D) Smundja e Celiakis

168. M 80 vje, ka nj jav me diarhe dhe krampe abdominale. T njjtn histori pati dhe gruaja, por asaj shqetsimet i zgjatn vetm 2 dit. Cila sht diagnoza m e mundshme: A) Smundja inflamatore e zorrs B) Sindromi i zorrs s irritushme C) H iperthroidizmi D) Gastrenterit infeksioz

169. M 67 vje, paraqet letargji t prgjithshme, rnie n pesh, shpeshtim t defekimit q 4 muaj. Ka erithrosediment t lart. Cila sht diagnoza m e m u n dsh m e: A) Malinjitet kolorectal B) Sindromi i zorrs s irritushme C) H iperthroidizmi D) Smundja e Celiakis

170. M 67 vje, ka 2 muaj q sht kaps, megjithse prdor ushqime me fibra. Ka rn n pesh.Nuk prdor medikamente q bjn kaps. Fece pr gjak ocult rezultoj pozitiv. Cila sht diagnoza m e mundshme: A) Malinjitet colo-rectal B) Sindromi i zorrs s irritushme C) H iperthroidizmi D) Smundja e Celiakis

171. Cili mund t jet efekti ansor fatal nga mbidoza e ferri sulphatit: A) Aritmi B) K onvulsione C) Insuficienc respiratore D) Insuficienc hepatike

172. Cili mund t jet efekti ansor fatal nga mbidoza e paracetamolit A) Aritmi B) Insuficienc kardiake C) Insuficienc respiratore D) Insuficienc hepatike

173. Cili mund t j et efekti ansor fatal nga mbidoza e Diamorfins A) Aritmi B) K onvulsione C) Insuficienc respiratore D) Insuficienc hepatike

174. Cili mund t jet efekti ansor fatal nga mbidoza e acidit mefenamic ( po nstan it): A) Aritmi B) K onvulsione C) Insuficienc respiratore

D) Insuficienc hepatike

175. Cila sht diagnoza m e mundshme n qoft se pacienti ka kto analiza: Biluribin(N: <0.9) 5mg/dl ALT ( N: 5-35) 1250 U/L FAL(fosfataza alkaline) (N: 30-300u/L) 500 U/L Albumina g/l : normal HB: normal Retikulociti: normal A) B) C) D) Hepatit viral akut Ikter kolestatik Smundje kronik e mlcis nga alkoli Anemia hemolitike autoimune

176. Cila sht diagnoza m e mundshme n qoft se pacienti ka kto analiza: Biluribin(N: <0.9) 10 mg/dl ALT ( N: 5-35) 195 U/L FAL(fosfataza alkaline) (N: 30-300u/L) 900 U/L Albumina g/l : normal HB: normal Retikulociti: normal A) B) C) D) Hepatit viral akut Ikter kolestatik Smundje kronik e mlcis nga alkoli Anemia hemolitike autoimune

177 Cila sht diagnoza m e mundshme n qoft se pacienti ka kto analiza: Biluribin(N: <0.9) 4.5mg/dl ALT ( N: 5-35) 50 U/L FAL(fosfataza alkaline) (N: 30-300u/L) 200 U/L Albumina g/l: normal HB (N 12-17 gr/dl): 8 gr/dl Retikulociti( N: 8-20%o) 100%o A) B) C) D) Hepatit viral akut Ikter kolestatik Smundje kronik e mlcis nga alkoli Anemia hemolitike autoimune

178) Cila sht diagnoza m e mundshme n qoft se pacienti ka kto analiza:

Biluribin(N: <0.9) 1.5mg/dl ALT ( N: 5-35) 80 U/L FAL(fosfataza alkaline) (N: 30-300u/L) 400 U/L Albumina( 35-50 g/l) 28g/l HB (12-17gr/dl) 10.5 gr/dl Retikulociti: normal A) B) C) D) Hepatit viral akut Ikter kolestatik Dmtim kronik i mlcis nga alkoli Anemia hemolitike autoimune

179. Sa sht mesatarish sasia m minimale e gj akut t humbur n hemoragjit e siprme GI q t ngj yroset fecja n t zi, pra t shfaqet melena: A) 50 ml B) 30ml C) 70 ml D) 100 ml

180. Vetm nj nga kushtet e m poshtme nuk ndikon pr brjen korekte t eksaminimit t feceve pr gj ak ocult: A) Diet pa mish 3 dit rresht B) T przihet fecja para marrjes s mostrs C) Fecja t merret 3 dit rresht kur eksamini pararends sht neg. D) Ndalimi i terapis me preparate Hekuri

181. Mbas nj hemoragj ie t vetme gastro- intestinale dhe pr sa dit fecja do t jet e errt : A) Deri 1 dit B) Deri 2 dit C) Deri 3 dit D) Deri 7 dit

182. Mbas nj hemoragj ie t vetme gastro- intestinale dhe pr sa dit fece pr gj ak ocult do t j et pozitive : A) Deri 1 dit B) Deri 2 dit C) Deri 3 dit D) Deri 7 dit

183. Ekza minimi fece pr gj ak ocult mund t rezultoj falls pozitiv nga: A) Marrja e hekurit B) Marrja e bismuthit C) Konsumi i mishit D) Konsumi i spinaqit

184. Vendi i gjakrrjedhjes n melena mund t jet n: A) Sigm B) Proksimalisht colon transvers C) Ampula rectale D) Coloni zbrits

185. Kur kundrindikohet sildenafil (grup me viagrn): A) Pacienti merr nitrate B) Pacienti ka histori t smundjes ishemike t zemrs C) Personi ka deformime t penisit D) Personi sht mbi 68 vje

186. Kur mund t kundr indikohet sildenafil( grup me viagrn): A) Pacienti ka br prostat ectomi totale. B) Pacienti ka histori t smundjes ishemike t zemrs C) Personi ka deformime t penisit D) Personi sht mbi 68 vje

187. Vetm nj nga meddikamentet e m poshtme nuk bn pjes n mjekimin e anafilaksis A) Adr en alina B) Lanatosid c C) Chlorpheniramina 10 mg iv D) Hydrocortison 100-200 mg iv

188. Cila sht doza korrekte e adrenalins n mj ekimin e anafilaksis. A) 10 ml t 1:1000 im B) 1ml t 1:1000 iv C) 1 ml t 1:1000 im D) 10 ml t 1:1000 im

189. Sa sht sasia e gjakut q duhet t humbas pr tu br diagnoza objektive e Menorhagjis A) 80 ml B) 40 ml C) 100 ml D) 20ml

190. Cili nga medikamentet e m poshtme prdoret me sukses n profilaksin sekondare t insultit(TIA). A) Aspirina me doz t lart B) Warfarina edhe pa fibrilacion atrial C) Aspirina me doz t ult D) Heparina

191. Cili nga medikamentet e m poshtme prddoret me sukses n profilaksin sekondare t insultit (T IA). A) Aspirina me doz t lart B) Warfarina edhe pa fibrilacion atrial C) Sintron D) ACE inhibitort pr t pasur nj TA m t ult

192. Cili nga medikamentet e m poshtme mund t prddoret me sukses n profilaksin sekondare t insultit (TIA). A) Aspirina me doz t lart B) Warfarina edhe pa fibrilacion atrial C) Statinat D) Heparina

193. N qoft se pr profilaksin sekondare t insultit prdorim njrin nga medikamentet e m poshtme, rreziqet kaloj n prfitimet: A) Aspirina me doz t lart B) Statina C) Aspirina me doz t ulta D) ACE inhibitort pr TA m t ult

194. N qoft se pr profilaksin sekondare t insultit, prdorim njrin nga medikamentet e m poshtme, rreziqet kaloj n prfitimet: A) Aspirina me doz t ulta B) Prdorimi i Warfarins pa fibrilacion atrial C) Nj statin

D) ACE inhibitor pr TA m t ult

195. Pacienti 63 vjear, del nga spitali pasi u trajtua pr IAM( infarkt akut miokardi). Cili nga 4 medikamentet e m poshtme sht pranuar t j et esencial n profilaksin sekondare t IAM: A) Aspirina B) Diltiazeni C) Acidi folik D) Isosorbide mononitrate

196. Pacienti 63 vjear del nga spitali pasi u trajtua pr IAM( infarkt akut miokardi). Cili nga 4 medikamentet e m poshtme sht pranuar t j et esencial n profilaksin sekondare t IAM: A) D oxazosin B) Diltiazeni C) Atenololi D) Isosorbide mononitrate

197. Pacienti 63 vjear del nga spitali pasi u trajtua pr IAM( infarkt akut miokardi). Cili nga 4 medikamentet e m poshtme sht pranuar t j et esencial n profilaksin sekondare t IAM: A) Ramipril B) Diltiazeni C) Acidi folik D) Isosorbide mononitrate

198. Paciente 67 vj eare del nga spitali pasi u trajtua pr IAM( infarkt akut miokardi). Cili nga 4 medikamentet e m poshtme sht pranuar t j et esencial n profilaksin sekondare t IAM: A) Trinitrina B) Diltiazeni C) Acidi folik D) S im va sta tina

199. Nga 4 mundsi, zgjidh antitrupin m specifik pr vendosj en e diagnozs n anemin pernicioze: A) ANCA B) Anti cardiolipin C) Anti qeliza parietale gastrike D) dsDNA

200. Nga 4 mundsi, zgjidh antitrupin m specifik pr vendosjen e diagnozs t lupusit eritematos sistemik : A) ANCA B) Anti cardiolipin C) Scl 70 D) dsDNA

201. Nga 4 mundsi, zgjidh antitrupin m specifik pr vendosjen e diagnozs t smundjes s Celiakis : A) Anti- retikulin B) Anti cardiolipin C) RhF D) dsDNA

202. Nga 4 mundsi, zgjidh antitrupin m specifik n rastin e aborteve t prsritura : A) ANCA B) Anti cardiolipin C) RhF D) dsDN

NEUROLOGJIA 1. Mnyra e transmetimit hereditar n distrofin muskulare tipi Duchenne sht si m posht: A) autosomale dominante B) autosomale recisive C) e lidhur me X-in D) e tipit mitokondrial E) recesive sporadike

2. Episodet e migrens pa aura kan nj kohzgj atje prej: A) 2-10 minutash B) 10-60 minutash C) deri n 4 or D) mbi 4 or deri n 7 or E) m shum se nj jav

3. Nj lezion ishemik n territorin e arteries cerebri media sht prgjegjs pr: A) hemiplegji homolaterale me predonim n ansin e poshtme B) hemiplegji dhe hemianestezi kontrolaterale me paraliz t shikimit drejt ans s kundrt C) deficit t nervave kraniale homolaterale D) deficit t nervave kraniale homolaterale dhe shenja motore, sensitive e cerebelare kontrolaterale E) dizartri

4. N siringomieli historia zakonisht fillon me: A) fashikulacione B) dhimbje t lokalizuara n nj nga ansit e siprme C) crregullime sfinkteriane D) atrofi lokale, dobsi, sensacione djegse t padhimbsh me n nj ansi t siprme E) asnj nga prgj igjet e msiprme

5. Aksidenti ishemik tranzitor me prkufizim ka nj kohzgj atje prej: A) < 1 or B) < 12 or C) < 24 or D) < 1 jav E) < 1 muaj

6. Cili sht shkaku m i shpesht i AVC-ve? A) diabeti B) hipertensioni arterial C) trombo-embolia D) aneurisma E) ateroskleroza

7. Nj pacient pati nj dhimbje t fort mesi ndrkoh q po punonte me trupin n fleksion. Ditn tjetr, ai u zgj ua me dhimbje t fort dhe mpirje t gishtit t Ir t kmbs s majt. Ka shum mundsi q ai t ket hernie t diskut intervertebral: A) L1-L2 B) L4-L5 C) L5-S1 D) S1-S2 E) S2-S3

8. Cikli nga ekzaminimet e mposhtme sht m i indikuar n hetimin e nj pacienti me skleroze multiple: A) Tomografia aksiale e kompjuterizuar B) rezonanca magnetike C) angiografia e arterieve karotide D) shintigrafia cerebrale E) angiografia digitale

9. T gjitha nga pohimet e mposhtme n lidhje me smundjen e Parkinson jan t vrteta prvec: A) tremori ka nj frekuenc t shpejt rreth 10-12Hz B) smundja shkaktohet nga nj deficit i dopamins C) instabiliteti postural sht nje shkak i rndsishm i invaliditetit D) smundja mund t trajtohet me amantadin E) smundja sht m pak e shpesht se demenca Alzheimer

1 0. N nj pacient skaneri (TAK) vuri n dukje nj lezion ekspansiv cerebral (neoplazi, abces, lezion inflamator apo malformacion vaskular). Studimi i mtej shm diagnostik i lezionit (natyra, shtrirja, vaskularizimi) do t bhet me cilin nga ekza minimet e mposhtme: A) skaner (TAK) me kontrast intra-venoz B) rezonanc magnetike me kontrast intra-venoz C) angiografia digitale arteriale D) radiografi crani E) rezonanc magnetike me kontrast intra-venoz dhe angio-rezonance

1 1. N ciln nga gj endjet e mposhtme EEG-ja sht m e dobishme? A) koma B) demenca C) ictus D) vertigo E) cefale

1 2. Lobet temporale cerebrale prpunojn: A) njohjen vizive B) perceptimin auditiv C) memorj en D) afeksionin E) t gj itha t msiprmet

1 3. N kt list me crregullime neurologj ike me instalim akut dhe kohzgjatje prej pak minutash apo orsh vetm nj ra paraqet nj T IA (AIT ): A) sinkopi B) cefalea C) hypoestesia dhe parestezia n nj hemikorp D) deficit i sensibilitetit qe instalohet dhe zhduket me karakteristikat e ecjes E) inkontinenca urinare

1 4. Sindromi Guillain-Barre prek: A) pa dallim fmij apo adulte, n t dy sekset B) vetm adultt, pavarsisht seksit C) n mnyr predominnate subjektet e moshuar D) m tepr meshkujt E) m tepr femrat

1 5. Hidrocefalia qe nuk provokon hipertension intrakranial sht hidrocefali: A) nga tumor i nervit akustik B) nga atrofia cerebrale C) nga obstruksioni i ventrikulit t tret D) nga obstruksioni i foraminave t atis s ventrikulit t katrt E) nga malformacioni i Arnold-Chiari

1 6. MMSE (Mini-Mental State Examination) sht: A) nj ekzaminim klinik me rndsi t pakt klinike B) nj ekzaminim i nevoj shm pr t gjith pacientt me shenja t alterimit konj itiv-komportamental, t dizorientimit n koh e hapsir C) nj test, t cilin pacienti mund ta kryej vet D) nj tregues pr t vlersuar gravitetin e nj traume kraniale

1 7. Nga cili burim infeksioni rrjedh varianti anglez i semundjes se CreutzfeldtJacob? A) nga lopa B) nga dhia C) nga delja D ) nga njeri u E) nga maj muni

1 8. Nj grua n muaj in e tet t shtatzanis ka paraqitur n j avn e fundit nj shtim diskret n pesh t lidhur pj esrisht me nj retension t likideve: Gj at nats, ajo ankohej pr dhimbje n dorn e djatht si pickime gjilprash n pj esn palmare dhe n 2-3 gishtat e par t dors. Bhet fjal pr: A) skleroz multiple B) kompresion i nervit ulnar n berryl C) kompresion i nervit radial n 1/3 e mesme t humerusit D) sindrom i tunelit karpal E) rheumatalgia (dhimbje rheumatologjike)

1 9. Hemianopsia bitemporale vrehet: A) n tumoret e gj endrs pineale B) n kraniofaringeomat C) n aneurizmat e arteries ko munikante posteriore (ACoP) D) n gliomat e nervit optik E) n t gjitha rrethanat e msiprme

2 0. Tremori i qetesise: A) shtohet gjat gj umit B) shtohet gjat lvizjeve t vullnetshme C) takohet n disa patologji t sistemit ekstrapiramidal D) takohet n disa patologji t cerebelumit E) takohet n disa patologji t sistemit piramidal

2 1. Cfare nenkuptohet me spasticitet: A) nj dobsi muskulare, sidomos distale, me origjin nervore centrale B) nj rritje e tonusit muskular me hiperrefleksi C) nj difekt n koordinimin muskular D) nj gj endje dekontraktimi muskular tonik me spasma E) nj crregullim t ecjes q rndohet nga mbyllja e syve

2 2. Cili nga prshkrimet e mposhtme e prkufizon m mir dhimbj en migrenoze? A) dhimbje pulsatile e tipit migrant dhe karakter paroksistik B) dhimbje e vazhdueshme, e tipit rndues q rndohet me aktivitetin fizik C) dhimbje me intensitet t ult-t moderuar, vetm bilaterale, q zgj at > 12 or D) sht pulsatile, nga 4-72 ore, intensitet e moderuar-e rnd, unilaterale, ndonjher e shoqruar me nauzea e t vjella E) dhimbje e tipit t vazhdueshm, vetm t paraprir nga aura, shpesh induktuar nga stimuj odorant, viziv, auditiv

2 3. N nj pacient q paraqet status epileptik t gjeneralizuar, pr t garantuar funksionet vitale cila terapi do t praktikohet? A) barbiturik me rrug orale B) benzodiazepin me rrug endovenoze C) benzodiazepin me rrug orale D) neuroleptik me rrug endovenoze E) anti-depresiv tre-ciklike me rrug endovenoze

2 4. Shenja m e qndrueshme dhe m e rndsishme e disfunksionit cerebelar s ht: A) instabiliteti n ecje B) oshilacione t mdha me mbylljen e syve C) nistagmus D) hypostenia E) tremori

2 5. Paraliza periodike familj are sht nj dobsi muskulare e rnd pseudoparalitike e lidhur me: A) paksimin e K+ serik B) rregullim t transmetimit t impulsit nervor n mus kul C) alterimin e pllaks motore D) furnizimin e pakt me O2 n muskuj E) hypofunksionin kortiko-surenalien

2 6. Nj lezion komplet i palcs spinale mund t prfshij t gj itha prvec nj rs: A) ataksi n ecje B) paraplegjia C) anestezi posht nivelit t lezionit D) atrofi dhe fashikulacione t muskujve n ansit e poshtme E) rregullime sfinkteriane

2 7. Esht nj form idiopatike e cefale-s: A) cefalea nga monoksidi i carbonit B) cluster headache C) cefalea nga hiperpireksia (febrile) D) cefalea nga abuzimi me medika mente E) asnj nga prgj igjet e dhna

2 8. Prkufizohet TIK nj lvizje q ka karakteristikat e mposhtme prvec: A) zgj at pak B) e vullnetshme C) stereotipe D) e thjesht apo komplekse E ) p r s rit s e

2 9. Ne sindromin e kembeve pa pushim gjate nates jane te shpeshta: A) zgj ime te zgjatura nga dispnea B) zgjime konfuzionale nga gj ume i thelle C) levizje periodike te anesive te poshtme D) miokloni hipnoike ne fazen e dremitj es E) ushqyerje kompulsive

3 0. Cili nga simptomat e mposhtme sht tregues pr nj polineuropati n faz fillestare? A) cefalea B) krampet C) konvulsionet D) retensioni urinar E) dizartria

3 1. Cila nga shenjat e mposhtme nuk sht e pranishme n poliradikuloneuritin Gui llain-Barre? A) hypoestezia B) deficit motor C) dhimbje muskulare D) reduktim i reflekseve osteo-tendinoze E) hypertonia

3 2. N paralizn e nervit t gj asht kranial, syri sht: A) i rrotulluar nga jasht B) i rrotulluar nga brenda C) aksi okular nuk sht i ndryshuar D) i rrotulluar lart E) i rrotulluar posht

3 3. Neuralgj ia trigeminale sht m e shpesht: A) n fmij dhe n gra

B) n meshkuj dhe n t moshuar C) n gra dhe n subjekte t moshuar D) n fmij dhe n meshkuj E) n gra dhe n adoleshenc

3 4. Ndrprerja e plot e traktit optik t djatht do t provokoj: A) hemianopsi bitemporale B) hemianopsi homonime dextra C) hemianopsi homonime sinistra D) kuadrantanopsia dextra E) kuadrantanopsia sinistra

3 5. N ishemin cerebrale t territorit karotidien t djatht, trajtimi kirurgjikal (trombendarterektomia) sht i indikuar: A) n munges t pllakave karotidiene B) n prani t pllaks karotidiene sin pavarsisht nga grada e stenozs C) n prani t pllaks karotidiene dex pavarsisht nga grada e stenozs D) n prani t pllaks karotidiene dex me stenoz < 30% E) n prani t pllaks karotidiene dex me stenoz > 70%

3 6. A sht e mundur t ndrpriten medikamentet anti-epileptike? A) po, pas t paktn 2 vjet pa kriza B) j o, n asnj rast C) j o, nqse EEG mbetet e alteruar edhe pse pa kriza D) po, pas gjasht muaj sh pa kriza E) jo nqse epilepsia sht sekondare nga nj traume kraniale

3 7. Shkaku m i shpesht i hemorragjis subaraknoidale prfaqsohet nga: A) aneurizma cerebrale B) malformacioni arterio-venoz cerebral C) tumori cerebral D) trauma kraniale E) koagulopatit

3 8. Menj her pas nj seksioni (prerjej e) transversal t palcs spinale pritet t gj endet m shpesh: A) inkontinenca urinare B) retensioni urinar C) paraplegjia spastike

D) shenja e Babinskit E) rritje e reflekseve osteo-tendinoze

3 9. Manifestimet inaugurale m t shpeshta t miastenia gravis jan: A) disfagia dhe disfonia B) lodhja kronike C) ptoza palpebrale dhe diplopia D) rregullimet respiratore E) hypostenia proksimale

4 0. Cili nga ekzaminimet e mposhtme duhet pr t konfirmuar nj patologji hipof izare? A) TAK (skaner) cerebral B) TAK (skaner) i hipofizs C) Radiografia e sella turcica D) Kateterizimi i sinuseve petroze E) Rezonanca cerebrale

4 1. Regj istrimi elektroencefalografik bazal i nj neonati 3 muajsh: A) sht i njj t me at t nj adulti B) sht praktikisht i parregj istrueshm pr shkak t ko mpliancs s dobt t pacientit C) sht i ndryshm nga ai i adultit D) asnj nga prgjigj et e msiprme E) nuk paraqet fazat e gjumit REM nokturn

4 2. Mielografia: A) sht nj ekzaminim bioptik i aksoneve nervore B) lej on evidentimin e anomalive t konduksionit t fibrave nervore C) sht nj ekzaminim q prek pjest kaudale t cerebelumit D) lejon evidentimin e anomalive t kanalit vertebral E) sht nj ekzaminim q nuk parashikon prdorimin e mjeteve t kontrastit

4 3. Neuralgjia trigeminale lokalizohet n mnyr preferenciale: A) n nivel t degs s I-r t trigeminalit B) n nivel t degs s II-t dhe t III-t t trigeminalit C) n nivel fronto-okular D) n nivel oksipital E) n nivel temporo-parietal

4 4. Cila nga shenj at e mposhtme nuk i referohet nj lezioni t neuronit t par motor ? A) fashikulacionet B) shtimi i reflekseve osteo-tendinoze C) shenja e Babinskit D) hypertonia muskulare E) defiiti motor

4 5. Smundja e Alzheimer sht familj are n : A) 0-5% t rasteve B) 15-20% t rasteve C) 56-60% t rasteve D) 100% t rasteve E) nuk sht asnjher e tipit familjar

4 6. Polineuriti infeksioz i tipit t Guillain-Barre ka t gjitha karakteristikat e mposhtme prve njrs. Cils? A) sht shpesh pasoj e nj infeksioni viral t aparatit respirator B) sht e lidhur probabilisht me Cytomegalovirus C) proteinat likuorale jan veanrisht t shtuara D) n likuor jan t pranishme IgG oligoklonale E) shoqrohet shpesh me rregullime sensitive n shtrirje t madhe

4 7. Territori muskular m shpesh i prekur nga miastenia gravis sht: A) muskulatura proksimale e ansive t siprme B) muskulatura okulare ekstrinseke C) muskulutura faringo-laringeale D) muskulatura distale e ansive t siprme E) muskulatura proksimale e ansive t poshtme

4 8. Nj burr 68 vj ear me nj histori hipertensioni arterial paraqitet pr instalimin akut t nj defiiti motor e sensitiv n ansit e poshtme t shoqruara me inkontinenc t dyfisht. Ekzaminimi neurologj ik evidenton nj paraplegji flaske, tonusi sfinkterian i reduktuar, anestezia termo-algjike nga nj e treta e poshtme e toraksit e posht pa prekje t sensibilitetit vibrativ dhe t sensacionit t pozicionit dhe lvizj es. Etiologj ia m e mundshme sht: A) infarkti medular B) infarkti bulbar C) abcesi spinal

D) mieliti transvers E) sindroma Guillain-Barre

4 9. Nj rnie e menjhershme e forcs n ansit e poshtme me arefleksi osteotendinoze dhe me munges t shenj ave piramidale ka si shkaqe t mundshme : A) lezione demielinizante spinale B) tumor medular C) sindromn Guillain-Barre D) siringomielin E) araknoiditin spinal

5 0. Prekja e sistemit nervor periferik n terren t AIDS (SIDA) karakterizohet nga shfaqja e: A) neuropatis demielinizante akute, e tipit Guillain-Barre B) neuropatis sensitive distale simetrike, me disestezi doloroze C) neuropatis autonome D) multineuropatis kraniale, radikulopatis progresive doloroze me rregullime sfinkteriane E) t gj ithave prgj igjeve t msiprme

5 1. Smundja neurologjike e karakterizuar nga zona t prhapura demielinizimi me integritet relativ t cilindrakseve sht: A) sifilizi B) poliomieliti C) skleroza multiple D) anemia pernicioze E) skleroza laterale amiotrofike

5 2. Cila nga karakteristikat e mposhtme klinike nuk prkufizoj n nj aksident ishemik tranzitor? A) fillim i papritur B) simptomatologji neurologjike fokale C) prhapje progresive e simptomave n nj ansi apo hemikorp D) zhdukje e shpejt brenda 24 orve E) mungesa e shenjave neurologjike pas 24 orsh

5 3. Cili nga medikamentet e mposhtm prdoret n trajtimin e epilepsis temporale? A) carbamazepina B) levodopa

C) butirofenoni D) triciklikt E) fenotiazin

5 4. N malformacionin e Arnold-Chiari: A) klinika mund t simuloj nj tumor t fosa crani posterior, por jo nj skleroz multiple B) manifestohen shenja dhe simptoma si t lezioneve cerebelare, bulbare dhe t nervave kraniale C) klinika mund t simuloj siringomielin, por jo platibazine D) simptomat n prgjithsi nuk manifestohen deri n mosh adulte E) shfaqja e hidrocefalis sht e pazakonshme

5 5. T gjitha pohimet e mposhtme n lidhje me nj episod migrene jan t gabuara prve: A) zakonisht fillojn papritur B) mund t zgj as me jav C) shpesh paraprihet nga skotoma D) shoqrohet me spazma t masseterit E) asnj nga prgj igjet e msiprme

5 6. Mekanizmi i veprimit t toksins botulinike sht: A) inhibimi i konduksionit t kaliumit B) inhibimi i lidhjes post-sinaptike acetilkolin-receptor C) inhibimi i lirimit t acetilkolins nga elementi pre-sinaptik D) shtim i degradimit t acetilkolins E) inhibim i konduksionit t natriumit

5 7. Cila nga kto situata nuk sht faktor deklanshant (nxits) i nj kriz epilepti ke? A) abuzimi i alkoolit B) deprivimi i gj umit t nats C) ethe (temperatur) e zgj atur D) dehidratim apo hyponatremi e thell E) pirja e duhanit

5 8. N afazin Wernicke: A) t folurit nuk sht fluent B) pacienti sht i pavetdijshm pr at q e rrethon C) t folurit sht i pakuptueshm

D) sht i pranishm fenomeni i agramatizmit E) t gj itha prgjigj et e msiprme

5 9. Episodet e cefaleve t tipit cluster kan nj kohzgjatje prej: A) pak sekondash B) 2-3 minutash C) 15-180 minutash D) 4-24 orsh E) 24-72 orsh

6 0. Terapia anti-agregante sht e indikuar: A) n parandalimin sekondar t ishemis cerebrale B) n hemorragjin subaraknoidale C) n t gj ith subjektet mbi 50 vje, edhe pse n munges t faktorve t riskut vaskular D) n parandalimin sekondar t hemorragjis cerebrale intraparenkimale E) n asnj nga prgjigj et e dhna

6 1. Tremori parkinsonian sht i tipit: A) postural B) kinetik C) t qetsis D) ortostatik E) sinkron

6 2. Shenja e par m e shpesht e nj patologjie t nervit t VII-t kranial sht: A) ptoza palpebrale kontrolaterale B) paksimi i refleksit orbital t ans s prekur C) paksim i refleksit orbital t ans kontrolaterale D) devijimi kontrolateral i gj uhs E) fashikulacione t rima labiale

6 3. Me miotoni nnkuptohet: A) nj gj endje persistente e kontrakturs muskulare me krampe B) nj rritje persistente e tonusit muskular e tipit elastik, me rritje te reflekseve tendinoze C) nj rritje e tonusit muskular e tipit elastik, me spazma muskulare D) nj defiit i relaksimit muskular nga kontraksioni persistent edhe pse stimuli ka mbaruar

E) nj rregullim motor central me lvizje prsritse t vazhdueshme dhe postur anormale

6 4. Nj burr 50 vj ear paraqet prej rreth 15 muaj sh nj simptomatologji me rndim progresiv t shpejt t karakterizuar nga parapareza spastike, hiperrefleksia osteo-tendinoze difuze, shenja Babinski bilaterale, fashikulacione, atrofi t moderuar n muskujt e duarve, munges e defiiteve t sensibilitetit apo t rregullimeve sfinkteriane, cili sht argumenti diagnostik m i m undshm: A) skleroza multiple B) siringomielia C) smundja e Friedreich D) skleroza laterale amiotrofike E) smundja e Tay-Sachs

6 5. Pr trajtimin e atakut t cefales migrenoze rekurente preferohet t prdoret: A) ergotamina ose dihidroergotamina per os B) acidi acetilsalicilik me doz mbi 1000mg per os C) AIJS D) triptant E) nuk ka diferenca sinjifikative ndrmj et medikamenteve t prmendura

6 6. rregullimet neurogene t kontrollit t veziks urinare t lidhura me lezione t meduls spinale provokojn shpesh: A) nekroz tubulare akute B) hidronefroz bilaterale C) polipoz vezikale D) divertikuloz vezikale E) hidrocel

6 7. N ciln nga kto patologji dyshohet pr nj fizio-patologji autoimune? A) meningiti aseptik B) skleroz multiple C) tuberkuloma cerebrale D) hemorragj ia subaraknoidale E) panencefaliti subakut sklerotizant

6 8. Dhimbja e migrens sht veanrisht: A) migruese B) therse C) konstriktive D) pulsatile E) shpuese

6 9. Hemtomielia ndodh m shpesh n ecurin e nj: A) diskrazie hematologjike B) rupture t nj malformacioni arterio-venoz C) ruptura e nj arterieje arteriosklerotike D) traume E) asnj prej prgjigj eve t prmendura

7 0. Numri normal i limfociteve n lngun cerebro-spinal normal sht: A) deri n 2-3 celula/mm3 B) deri n 10 celula/mm3 C) deri n 100 celula/mm3 D) nuk duhet t jen celula t asnj tipi E) varet nga formula e gj akut periferik

7 1. Cila nga etiologjit e mposhtme t meningitit ka prognozn m t keqe? A) meningiti nga meningokoku B) meningiti nga pneumokoku C) meningiti nga Hemofilus influenze D) meningiti nga Coxsakiae E) meningiti nga Listeria monocitogenes

7 2. Cili sistem apo struktur preket n sklerozn laterale amiotrofike? A) cerebelum B) medula spinale C) motoneuroni i poshtm D) motoneuroni i I-r dhe i II-t E) sistemi sensitiv

7 3. Kujt i korrespondojn krizat e gjeneralizuara t nj ohura si grand mal? A) krizave epileptike q zgjasin m shum se 2 minuta

B) krizave konvulsive toniko-klonike C) vetm krizave q kan nj gjeneralizim sekondar D) vetm krizave t njpasnjshme q paraprijn statusin epileptik E) vetm krizave q shoqrohen me inkontinenc sfinkteriane

7 4. Nj hemiparez brakio-faciale q regredon n pak or mund t konstatohet me shpesh n: A) hemorragjin subaraknoidale B) krizn miastenike C) TIA (AIT) karotidiene D) pus-n e nj smundjeje demielinizante E) trombozn e arteries cerebrale media

7 5. Encefaliti akut nekrotizant dyshohet n: A) prhapjen e nj neurosifilizi B) prhapjen e nj meningiti tuberkular C) infeksionin nga neiseria meningitis D) infeksionin nga herpes simplex virus E) infeksionin nga bacili Coli

7 6. Cilat j an shkaqet m t shpeshta t shfaqj es s epilepsis tek t moshuarit? A) traumat kraniale B) encefalitet C) rregullimet metabolike D) tumoret cerebrale E) smundjet cerebro-vaskulare

7 7. N miastenia gravis, dobsia muskulare sht rezultat i nj insuficience t transmetimit kolinergj ik n junksionin neuro-muskular; dobsia muskulare mund t shfaqet edhe pr shkak t nj mbidozimi t medika menteve kolinomi metike t prdorur n traj timin e miastenis. Cila nga shenjat/simptomat e mposhtme mund t ndihmoj n diferencimin midis nj kriz miastenike nga nj krize kolinergj ike? A) fashikulacionet muskulare B) ptoza palpebrale bilaterale C) diplopia D) insuficienca respiratore akute E) reflekset osteo-tendinoze normale

7 8. N trajtimin e smundjes s Parkinson n fazn fillestare t saj, zgjedhja e preferuar aktuale sht: A) administrimi i dozave t larta t Levodopa-s B) tentimi i medikamenteve dopamino-agoniste C) dhnia e nj bashkshoqrimi t dopaminergjikve dhe levodopas D) administrimi i domperidon E) administrimi antikolinergjik

7 9. Faktort e riskut pr zhvillimin e smundj es s Alzheimer prfshij n t gj itha t mposhtmet prve njrs: A) trauma kraniale B) duhani C) mosha e avancuar D) nj vlla i prekur nga smundja e Alzheimer E) sindromi Down

8 0. Lokalizimi m i shpesht i hemorragjive cerebrale sht: A) cerebelum B) cauda C) truncus cerebri D) kapsula interna E) nucleus ruber

8 1. Nj komplikacion i rnd i korrigjimit t shpejt t nj hipernatremie sht: A) edema cerebrale B) mielinoliza pontine centrale C) hemorragj ia subaraknoidale D) hemorragj ia subdurale E) hemorragj ia intracerebrale

8 2. Koma q instalohet shpejt me ndj enjn e uris, dobsis ekstreme, dj ersitj es profuze dhe modifikimeve t respiracionit sht n prgj ithsi nj : A) kom hepatike B) kom diabetike C) kom hypoglicemike D) kom uremike E) kom cerebrale

8 3. Nj pacienti me kriza epileptike toniko-klonike duhet: A) ti bllokohen lvizj et duke e imobilizuar at

gj uhs sigurt

B) ti vendoset dika midis dhmbve pr t parandaluar dmtime t C) t pritet prfundimi i krizs dhe m pas t vendoset n pozicion t

D) ti inspektohet kaviteti i goj s me gishta, nga e djathta n t majt, pr t eleminuar trupa t huaj eventuale E) t gj itha prgjigj et e msiprme

8 4. Pacient 32 vje, traum nga aksident automobilistik. Nuk lviz mir ansit e poshtme dhe nuk kontrollon sfinkterat. Dyshohet nj lezion traumatik i meduls spinale. Cfar ekzaminimi krkon t parin? A) TAK (skaner) i tr kolons spinale B) radiografi e tr kolons C) Rezonanc spinale D) elektromiografi dhe potenciale t evokuara t palcs E) stratigrafia e kolons vertebrale

8 5. Terapia antikoagulante sht zgj edhje e par: A) n t gj itha ictus-et ishemi ke B) n ictus-et ishemike sekondare nga fibrilacioni arterial C) n ictus-et ishemi ke sekondare nga ateromatozat karotide D) n ictus-et hemorragjike E) n ictus-et kriptogenike

8 6. Arteriti temporal (ose arteriti Horton) sht i shoqruar n mnyr tipike me: A) nj rritje t sedimentit B) nj rritje t hematokritit C) nj rritje t hemoglobins D) nj rritje t trombociteve E) nj reduktim t glukozs

8 7. Cila nga smundjes e mposhtme kardio-vaskulare konsiderohet si m shum risk pr emboli cerebrale? A) persistenca e foramen ovale apertum B) ateromat e harkut t aorts < 4 mm C) prolapsi i valvuls mitrale D) fibroza e anulum t valvs mitrale E) fibrilacioni atrial rheumatik apo valvular

8 8. Cefalea e tipit tensiv sht veanrisht: A) bilaterale B) unilaterale C) vetm frontale D) vetm oksipitale E) gjithnj okulare

8 9. Halucinacionet j an prkufizuar si: A) bindje false q mund t eliminohen me kritikn B) bindje false q nuk mund t eleminohen me kritikn C) perceptime sensoriale t gabuara t bazuara n stimulimin natyral t r e cepto r ve D) perceptime sensoriale t gabuara t pabazuara n stimulimin natyral t receptorve E) sensacione anormale fardo t lidhura me nj lezion neurologjik

9 0. Shkaku m i shpesht i paralizs akute bilaterale t nervit t VII-t kraniale s ht: A) paraliza e Bell B) infeksioni nga HIV C) sindromi i Guillan-Barre D) neuroborrelioza E) A dhe C

9 1. N sfern cerebro-vaskulare, ekzaminimi i lngut cerebro-spinal mund t prbj nj mjet mbshtets diagnostike: A) n trombozn e bazilares B) n hemorragjin sub-araknoidale C) n disekimin karotien ose vertebral D) n asnj nga prgjigjet e prmenduara E) n t gj itha prgjigjet e prmendura

9 2. Nj hemianopsi homonime e djatht mund t j et e shkaktuar nga nj lezion: A) i nervit optik B) i kiazms optike C) n retinn e djatht D) n korteksin oksipital t dj atht E) n traktin optik te majt

9 3. Nj vertigo rotatore pozicionale sht shum probabilisht e lidhur me: A) atakun ishemik vertebro-bazilar B) intoksikimin nga opioidet C) disfunksionimit labirintik D) lezion demielinizant E) epilepsin

9 4. Cili nga ekzaminimet e mposhtme instrumentale konsiderohet zgjedhja e par n rastin e nj humbjeje t papritur dhe tranzitore t vetdij es me rrzim n tok n nj subjekt t moshuar, t pashoqruar me manifestime epileptike? A) elektroencefalograma B) elektrokardiograma C) TAK (skaner) i koks D) echo doppler i karotideve E) radiografia e craniumit

9 5. Pacient 75 vje, kardiopat, nn terapi me antikoagulante, me nj traume t leht kraniale disa dit prpara konsults klinike. Jan prezente nj instabilitet postural dhe hemiparez e leht sinistra. Cilin ekzaminim do t krkonit dhe pse? A) TAK (skaner) cerebral pr dyshim t hematoms subdurale dhe dyshim ishemie B) TAK (skaner) cerebral pr dyshim stenoze t arteries silviane C) TAK (skaner) cerebral pr dyshim neoplazie D) Rezonanc cerebrale pr dyshim smundjeje degj enerative E) Angiografi pr dyshim hemato m cerebrale

9 6. Statusi (etat de mal) epileptik: A) sht nj gj endje kronike mbi t ciln duhet t ndrhyhet me modifikime t vogla n terapi B) krkon nj vlersim anesteziologjik si qndrimi i par terapeutik C) mund t shprehet vetm me kriza konvulsive t gjeneralizuara D) sht nj urgj enc mj eksore E) ka fenitoinn si terapin e vetme t mundshme

9 7. Nj lezion i ganglioneve bazale mund t j ap kto shenja prve: A) lvizje t pavullnetshme B) rigiditet C) tremor D) paraliz flaccida E) humbje e lvizjeve automatike shoqruese

9 8. T dhnat e lngut cerebro-spinal n sklerozn multiple: A) jan prgjithsisht patognomonike B) nuk jan prgj ithsisht patognomonike C) jo rrall prfshij n nj pleocitoz t shprehur D) prfshijn vetm nj hypoglukoraki E) prfshij n vetm nj rritje t proteinave totale

9 9. Cili nga elementet e mposhtme klinike NUK bn pjes n karakteristikat universalisht t pranuara t atakeve ishemike cerebrale tranzitore (T IA/AIT)? A) fillim akut B) zgj atje e simptomatologjis jo m shum se 24 or C) shenja neurologjike t tipit fokal D) tendenca pr t recidivuar E) prekje neurologjike objektive akoma pozitive pas 24 orsh

100. Cila e dhn kontribuon n mnyr vendimtare diagnozn e epilepsis? A) EKG B) ekzaminimi obj ektiv i pacientit C) TAK (skaner) i koks D) anamneza E) hetimi gj enetik

101. Cili sht shkaku kryesor i vdekjes n SLA? A) infarkti i miokardit B) bronkopneumonia C) pneumonia respirative D) demenca E) insuficienca respiratore

102. Quhet ndryshe edhe cefalea e vetvrasj es: A) migrena me aura B) cefalea cluster C) neuroalgjia odontogjene D) neuralgjia e n. Occipitalis major E) hemikrania paroksistike kronike

103. N ciln nga kto situata vrehet rritja m e madhe e proteinave n likuor? A) bllokun e plot medular

B) hemorragjin subaraknoidale C) skleroz multiple D) meningo-encefaliti akut viral E) paralizn progresive

104. Migrena pa aura sht m e shpesht: A) n moshn infantile B) n t moshuarit C) gjat shtatzanis D) n adultt e rinj E) gj at menopauzs

105. Refleksi i akomodimit apo i afrimit drejt syve konsiston n: A) midriaz, konvergj enc dhe rritje e kurbaturs s kristalinit B) midriaz, rrotullim intern dhe rritje e kurbaturs s kristalinit C) mioz, divergjenc dhe rritje e kurbaturs s kristalinit D) mioz, konvergj enc dhe rritje e kurbaturs s kristalinit E) mioz, konvergjenc dhe ptoz

106. Cefalea e tipit tensiv: A) prkeqsohet nga aktiviteti fizik B) deklanshohet (nxitet) nga sforcimet fizike C) nuk modifikohet me aktivitetin fizik D) prkeqsohet gj at gj umit E) asnj nga prgigj et e msiprme

107. Me epilepsi petit mal nnkuptohet: A) nj form e dobsuar e grand mal B) nj alterim i gj endjes s koshiencs jo mbi baz organike C) nj kuadr epileptik me aspekte klinike dhe elektroencefalografike specifike D) epilepsia e fmijris s hershme E) epilepsia psikomotore

108. Midis formave t cefaleve t cituara, takohet m shpesh n burrat se n grat: A) migrena pa aura B) migrena me aura C) cefalea e tipit tensiv

D) cefalea cluster E) hemikrania kronike paroksistike

109. Nj kriz epileptike parciale motore e djatht mund t shkaktohet nga nj fokus epileptik i lokalizuar n: A) lobin frontal t djatht B) lobin parietal t majt C) lobin oksipital t maj t D) talamusin e djatht E) lobin frontal t majt

110. N nj pacient me rregullime t ecj es, rregullime sfinkteriane dhe humbje t sensibilitetit nga umbilikusi e posht, ekzaminimi q kryhet n rend t par sht: A) rezonanca magnetike e koks B) angiografia medulare C) rezonanca magnetike e kolons torakale D) rakicenteze E) TAK (skaner) spinal

111. Nj grua 32 vj ee sheh dysh prej nj muaji. Ky rregullim sht intermitent, por duket t jet m evident n mbrmje ; prej disa ditsh ka vn re t ket lodhje n prtypj en e mishit; n ekzaminim obj ektiv evidentohet nj ptoz palpebrale sinistra q rritet me zgj atjen e shikimit lart. Cila sht diagnoza m e mundshme ? A) TIA/AIT (atak ishemik tranzitor) n territorin vertebro-bazilar B) tumor mesenfalik C) tumor pontin D) histeri E) miastenia gravis

112. Diseminimi (metastazimi) cerebro-spinal vrehet me probabilitet t lart n : A) meduloblastoma B) adenoma hypofizare C) astrocitoma grada IV (glioblastoma) D) oligodendroglioma E) kraniofaringeoma

113. Cila nga terapit e mposhtme sht ajo e zgjedhura n trajtimin e trombozs venoze cerebrale? A) an tibio tikt B) antiedematozt C) antiepileptikt D) heparina E) steroidet

114. Paraliza periodike sht e karakterizuar nga nivele intermitente: A) t rritura t fosfatazes acide B) t ulta t kaliumit C) t ulta t kreatinins D) t rritura t fosforemis E) t rritura t aminoacideve

115. Cili nga alterimet e mposhtme t studimit elektrofiziologjik t konduksionit nervor jan karakteristike pr neuropatit demielinizante? A) amplitud e ult e potencialit sensitiv B) dekrement n stimulimin e prsritur C) reduktim n shpejtesin e konduksionit motor e sensitiv D) prezenca e fibrilacioneve n ekzaminimin me gjilpr t muskulit E) rritje e shpej tsis s konduksionit

116. N SLA, alterimet e para motore q takohen jan: A) fashikulacionet B) tetania C) mioklonus D) paraliza flakside E) paraliza spastike

117. Cilat val karakterizojn ritmin baz t elektroencefalograms? A) alfa B) beta C) teta D) delta E) ritmi Mu

118. Simptomat neurologj ike t hiperparatiroidizmit mund t jen: A) hiperaktiviteti B) dizorientimi temporo-spacial

C) letargjia gj itha t msiprmet

D) amnezia

E) t

OBSTETRIKE-GJINEKOLOGJI 1. A) B) C) D) E) Pas menopauzes, dispareunia shkaktohet me shpesh nga: thatesia vaginale inkontinenca infeksionet nga kandida problemet erektive te partnerit te tera pergj igjet jane te sakta

2. Zgj idhni testin me te mire te screening diagnostik per zbulimin e diagnozes se anemise falciforme te fetusit: A) kultura e qelizave te likidit amniotik B) kampionet e gjakut fetal C) alfafetoproteina e likidit amniotik D) perqendrimi kimik i urines materne E) ekografia

3. Nje grua 32-vj eare me anamneze per infertilitet prej 4 vitesh eshte induktuar qe te ovuloje me klomifen. Pas 16 j avesh gestacioni eshte vene re se fundi i uterusit ndodhet ne nivelin e umbilikusit. Nje ekografi ka treguar nje gravidance binjakesh ne dy sakuse me dy placenta (njera anteriore, ndersa tjetra posteriore). Kjo grua ndodhet nen risk per te zhvilluar nje prej patologjive te meposhtme: A) infeksion te rrugeve urinare B) hipertension te induktuar nga gestacioni C) diabet gestacional D) nivele materne te uleta te alfa-fetoproteines serike E) moniliaze vaginale

4. Incidenca e ciles prej patologjive te meposhtme behet me e shpeshta pas menopauzes? A) tumoret fibroide B) adenom ioza C) endometrioza D) relaksimi pelvik E) kistet e korpusit luteal

5. A) B) C) D) E)

Kundraindikacion per perdorimin e pilules anticoncepsion eshte: endo metrioza mastopatia fibrokistike kisti ovarian seroz karcinoma ma mare e operuar prej me shume se 3 vitesh nulipariteti

6. Ne ekzaminimin mikroskopik te nje sekrecioni uretral, neisseria gonorreae paraqitet si: A) streptokok grampozitiv j ashteqelizor B) diplokok grampozitiv jashteqelizor C) diplokok gramnegativ j ashteqelizor D) diplokok grampozitiv brendaqelizor E) diplokok gramnegativ brendaqelizor

7. Cila eshte e sakte prej pohimeve te meposhtme ne lidhje me sindromen vul v o-vestib ulare? A) perfaqeson formen prevalente te vulvodinise B) shoqerohet pothuajse gj ithmone me dispareuni C) te tera pergjigj et jane te sakta D) pergjithesisht mungojne lezionet vulvare ose infeksionet e dukshme vulvu-vaginale E) ndonjehere mund te verehet eriteme makulare pergjate sulkusit imenal dhe ne nivelin e hapjes se glandulave vestibulare major dhe minor

8. Cili prej tumoreve malinje te meposhtem j ep me shpesh metastaza ne placente dhe ne fetus? A) m elanom a B) leucemia C) karcinoma mamare D) karcinoma e cerviksit E) karcinoma uterine

9. A) B) C) D) E)

Rritja fiziologjike e VES gj ate gravidances shkaktohet nga: reduktimi i gamagl obulinave rritja e betaglobulinave rritja e fibrinogjenit plazmatik rritja e transferrines rritja e prolaktines

1 0. Gj ate procesit te lindj es thuhet se dilatacioni i qafes se uterusit eshte i plote kur arrin permasen: A) 2-3 cm B) 6 cm C) 5 cm D) 10 cm E) 12 cm

1 1. Ne cilen dite arrin LH nivelin me te larte te nje grua qe normalisht ka menstruacione do 30 dite? A) diten e 12-te B) diten e 14-te C) diten e 16-te D) diten e 18-te E) diten e 27-te

1 2. Cila prej patologjive te meposhtme eshte shkaku me i mundshem i gj akrrjedhjes uterine disfunksionale te nje grua 28-vj eare me ekzaminim obj ektiv negativ dhe pa asnje problem tj eter? A) kondiloma akuminata B) kondiloma lato C) karcinoma vaginale me qeliza skuamoze D) kankroidi E) herpesi gj enital

1 3. A) B) C) D) E)

Me termin ektropion kuptohet: prania e endometrit ne lokalizim ektopik nje lezion inflamator i cerviksit nje prolaps i uterusit nje eversion i mukozes endocervikale nje fistule rekto-vaginale

1 4. A) B) C) D) E)

Cila klase e Ig mbizoteron ne kulloshter: IgM IgG IgA IgD IgE

1 5. Cila prej te meposhtmeve nuk eshte shkak i lubrifikimit vaginal te reduktuar: A) m enopauza B) sindroma Sj gren C) sindroma Cushing D) skleroza multiple E) hipotiroidizmi

1 6. A) B) C) D) E)

Fistula ileo-vaginale kerkon kolostomi vilet korionike penetrojne ne miometer vilet korionike arrijne serozen vilet korionike ndodhen ne kontakt direkt me deciduan vilet korionike penetrojne ne cerviks

1 7. Cili eshte medikamenti me i pershtatshem per terapine e haemophilus vaginalis: A) metronidazoli B) mikonazoli C) d i enes tro li D) sulfisoksazoli E) deosiciklina

1 8. A) B) C) D) E)

Placente previa quhet: placenta e inseruar pj eserisht ose totalisht ne segmentin uterin inferior placenta e primipareve placenta qe nxirret perpara fetusit shkolitja e placentes ne muajin e gjashte eshte nje term obsolet qe nuk ka domethenie klinike

1 9. Traj timi elektiv i prezantimit me shpatull eshte: A) oksitocine me doze te larte B) ventuza obstetrike C) forcepsi D) sectio cesareo E) nuk eshte i nevoj shem asnje intervent sepse eshte nje prezantim fiziologj ik

2 0.

Frekuenca e infeksionit fetal si pasoje e rozolise materne:

A) eshte me e larte nese rozolia materne ndodh ne trimestrin e pare te g r a v ida n c es B) eshte me e larte nese rozolia materne ndodh ne trimestrin e dyte te g r a v ida n c es C) eshte me e larte nese rozolia materne ndodh ne trimestrin e trete te g r a v ida n c es D) eshte me e larte ne rast reinfektimi matern E) asnje prej pergjigj eve te mesiperme

2 1. Cila prej patologj ive te meposhtme shkakton nivele te larta materne te alfafetoproteines ne javen e 17-te te gravidances: A) trizomia e kromozomit 18 B) sindroma Down C) vonesa e rritjes intrauterine D) anencefalia E) data e gabuar, data e sakte e shtatzenise eshte java e 14-te

2 2. Nje grua e prekur nga endometrioza mund te kete te tera ankesat e meposhtme, perve se: A) dispareuni B) sterilitet C) dhembje gj ate defekimit D) dhembje abdominale periodike C) mastodini

2 3. Cili eshte termi me i pershtatshme per te treguar humbjen fillimisht hematike, e me pas seroze, qe eshte karakteristike per puerperen: A) m enorragjia B) m eno m etrorragjia C) lokiacioni D) m etrorragjia E) e kla m p sia

2 4. A) B) C) D) E)

Endrometrioza mund te provokoje: inkontinence amenorre leukorre dispareuni anemi

2 5. Shkaku organik me i shpeshte qe ndodhet ne baze te rregullimit te deshires hipoaktive femerore eshte: A) diabeti mellitus B) sindroma depresive C) hipotiroidizmi D) te tera pergj igjet e mesiperme j ane te sakta E) asnje pergjigje nuk eshte e sakte

2 6. Cili prej prezantimeve fetale te meposhtme nuk lej on lindjen spontane nese fetusi eshte zhvilluar normalisht ne terme: A) prezantimi me bregme B) prezantimi podalik C) prezantimi frontal D) prezantimi me fytyre E) prezantimi me verteks

2 7. A) B) C) D) E)

Hormoni folikulostimulues prodhohet nga: qelizat kromofobe te adenohipofizes qelizat gonadotrope te adenohipofizes qelizat acidofile te adenohipofizes qelizat e tekes interne te gj itha pergj igj et jane te sakta

2 8. Anomalia kromozomike qe ndeshet me shpesh gjate aborteve ne trimestrin e pare eshte: A) sindroma Turner B) p oliploidi a C) monozomia autozomale D) trizomia autozomike E) translokimi i pabilancuar

2 9. Per cilen prej semundjeve kromozomi ke te meposhtme eshte karakteristik pterigium coll i: A) sindroma Down B) sindroma Klinefelter C) sindroma Turner D) sindroma Patau E) sindroma cri du chat

3 0.

Shkaku me i shpeshte i ambiguitetit gj enital te femijeve eshte:

A) B) C) D) E)

mosndarja kromozomike disgenezia gonadike sindroma andrenogjenitale mozaicizmi feminilizimi testikular

3 1. Te tera simptomat e meposhte tregojne vdekje fetale intrauterine, me perjashtim te nj eres: A) mungesa e ndjerjes se levizj eve fetale aktive nga nena B) mungesa e rritjes se vellimit te uterusit C) rritja e peshes materne D) regresioni i simptomave te gravidances E) mungesa e rrahjes kardiake fetale gjate ekzaminimit ekografik

3 2. A) B) C) D) E)

Cila eshte simptome klinike e shpeshte e karcinomes endometriale: vaginiti rekurrent dispareunia metrorragjite asnje prej pergj igjeve amenorrea

3 3. A) B) C) D) E)

Lokalizimi me i shpesht i endometriozes eshte: siperfaqja peritoneale e zorres se holle omentumi apendiksi vezika urinare peritoneumit i zones Douglas

3 4. A) B) C) D) E)

Cili eshte shkaku me i shpeshte i sterilitetit femeror: hormonal (anovulacioni) tubarik uterin kongj enital vaginal

3 5. Cili prej pohimeve te meposhtme eshte i sakte: A) kriteri diagnostik kryesor i DSM-IV per vaginizmin eshte prania e spazmes te pavullnetshme rekurrente ose persistente te muskulatures se nje te tretes se jashtme te vagines, qe nderhyn ne marredheniet seksuale

B) ne rastin e sindromes vulvo-vestibulare, vestibulektomia ka si qellim rezekimin e indit qe shkakton shfaqjen e dhembjes se vestibulit vaginal C) dispareunia e thelle eshte karakteristike e endometriozes, e P ID, e varikoceles pelvike, si edhe e aderencave fibrotike D) te tera pergjigjet jane te sakta E) ndonjehere spazma e vaginizmit pershkruhet si nje spazme aq e gjate dhe e rende sa qe shkakton shfaqjen e dhembjes

3 6. Te tera patologj ite e meposhtme shoqerohen me rritje te nivelit te prolaktines ne qarkullim, me perjashtim te: A) stresit B) hipotiroidiz m it C) anoreksise nervore D) akromegalise E) shta tz en ise

3 7. A) B) C) D) E)

Ndarja e dyte maturuese e ovocitit ndodh: ne uterus ne vezore ne peritoneum ne vagine ne tube

3 8. HCG: A) B) C) D) E)

Cili eshte tumori me i mundshem ne prani te niveleve te larta te beta tumori i sinusit endodermal tumori i granulozes-tekes se vezores kistadenokarcinoma seroze tumori i qelizave Sertoli-Leydi g koriokarcinoma

3 9. Gj ate shtatzenise fiziologj ike ndodhin te tera modifiki met e meposhtme te funksionit renal dhe te rrugeve urinare, me perjashtim te: A) dilatacionit te uretereve B) rritjes se fluksit plazmatik renal C) rritjes se fluksit glomerular D) reduktimit te klirensit te kreatinines E) rritjes se eliminimit urinar te glukozes

4 0. Hemopoieza ekstramedulare eshte shume e shpeshte ne fetusin human. Cili eshte lokalizimi me i rendesishem i hemopoiezes ne muajin e tete te s htatz e nise: A) sakusi vitelin B) shpretka C) palca e kockave D) melia E) limfonodujt

4 1. Cila eshte shenja dhe/ose simptoma me e shpeshte me te cilen paraqitet karcinoma endometriale: A) rritja vellimore e uterusit B) kolika abdominale C) hemorragj ia D) leukorrea E) te tera pergj igjet jane te sakta

4 2. A) B) C) D) E)

Shkaku/shkaqet endrokrin i trupit te shkurter te grave te reja perfshin: sindroma Turner hipotiroidizmi hiperplazia surrenaliene morbus Cushing te tera shkaqet e mesiperme

4 3. K a r d i o t o k o g r a f i a : A) regjistron njekohesisht me dy sensore frekuencen kardiake fetale dhe aktivitetin e kontraktimit te miometrit B) regjistron njekohesisht me dy elektroda frekuencen kardiake materne dhe aktivitetin e kontraktimit te miometrit C) ben te mundur vleresimin e morfologjise kardiake fetale D) regjistron njekohesisht me dy sensore frekuence kardiake fetale dhe aktivitetin e kontraktimit te miometrit dhe ben te mundur vleresimin e morfologjise kardiake fetale E) te tera pergj igjet jane te sakta

4 4. A) B) C) D) E)

Sindroma Klinefelter karakterizohet nga modeli kromozomik anormal: XO trizomia e kromozomit 15 delecioni i krahut te shkurter te kromozomit 18 XXY XYY

4 5. Cili eshte medikamenti me i pershtatshem per trajtimin e nje vaginiti te shkaktuar nga moniliaza: A) metronidazoli B) mikonazoli C) d i enes tro li D) sulfisoksazoli E) deoksiciklina

4 6. Shkaku me i shpeshte i deshtimit te kontraceptiveve orale, qe si pasoje shkakton nje shtatzeni te padeshiruar, eshte: A) ovulacioni qe fillon ne mes te ciklit B) frekuenca e marredhenieve seksuale C) perdorimi i pasakte i kontraceptiveve orale D) keqperthithja gastrointestinale E) krijimi i antitrupave

4 7. Te tera patologjite e meposhtme shoqerohen me defekte te septumit interventrikular, me perjashtim te: A) sindromes feto-alkoolike B) duhanpirj es gj ate shtatzenise C) infeksionet intrauterine TORCH D) sindromes Down E) tetralogj ise Fallot

4 8. Terapite zevendesuese hormonale vaginale per menopauzen mund te p e r m ire s o jn e: A) thatesine vaginale B) d i spa r e unin e C) deshiren seksuale D) te tera pergj igjet e mesiprme j ane te sakta E) raportin e iftit

4 9. Cili eshte risku i zhvillimit te nje rritjeje dhe zhvillimi j onormal te fetusit pas kryerjes se amniocentezes: A) 0% B) 3.5% C) 5% D) 10% E) 25%

5 0. Cili prej trajtimeve te meposhtme duhet te perdoret rutine per trajtimin e pe e k l a m p sise: r A) diuretiket B) kufizimi i perdorimit te natriumit C) p ropa n ololi D) sulfati i magnezit E) d e a m b u li m i

5 1. Shkaku me i shpeshte i inkontinences urinare te nje paciente ne menopauze eshte: A) inkontinenca urinare nga stresi B) prania e divertikuj ve uretrale C) inkontinenca nga grumbullimi i urines D) vezika e paqendrueshme E) fistula

5 2. A) B) C) D) E)

Shkaku me i shpeshte i abdomenit akut ne periudhen puerperale eshte: sepsisi puerperal apendisiti volvulusi divertikuli i Meckel kolecistiti

5 3. Cila prej substancave te marra nga nena gjate shtatzenise shoqerohet me anomali fetale dhe neonatale me demtime vaskulare: A) duhani B) alkooli C) substancat narkotike D) anfetaminat E) kokaina

5 4. Cila prej prezantimeve te meposhtme eshte indikacion absolut per sectio cesareo: A) prezantimi me fytyre B) prezantimi frontal C) prezantimi podalik D) te tera pergjigjet jane te sakta E) asnj era

5 5. A) B) C) D) E)

Korpusi albikant eshte: folikul atrezik folikul i Graaf folikul primordial nje rrejdhoje cikatriciale e korpusit luteum asnj era

5 6. A) B) C) D) E)

Defekti me i shpeshte ne sindrome adrenogj enitale eshte: idiopatik deficiti i 11-hidroksilazes deficiti i 17-hidroksilazes deficiti i 21-hidroksilazes deficiti i 3 01-dehidrogjenazes

5 7. Te tera medikamentet e meposhtme shoqerohen me anomali fetale kongj enitale, me perjashtim te njeres: A) tetraciklina B) talidomidi C) litiumi D) sulfa midet E) k u m arin a

5 8. Plakja gj enitale femerore: A) nuk modifikon kapacitetin orgazmik te gruas B) ekzalton kapacitetin orgazmik te gruas C) mund te shkaktojne involucion deri ne 50% te muskulatures se lemuar te trupave kavernoze D) perkeqesohet nga mungesa e hormoneve te tiroides E) pershpejtohet nga dietat drastike

5 9. Cila prej situatave te meposhtme perfaqeson nje avantazh te ekzaminimit te vileve korionike mbi amniocentezen, per percaktimin e anomalive fetale: A) mund te kryhet me perpara gj ate shtatzenise B) mund te percaktoje me sakte defektet e tubit neural C) ka nje mundesi me te ulet per te provokuar abort D) aborti kryhet njekohesisht me ekza minimin E) asnjera prej pergj igjeve

6 0. madh A) B) C) D) E)

Cili prej tumoreve te meposhtem eshte pergjegjes per numrin me te te vdekj eve ne vit: endo metrial mamar pulm onar intestinal (koloni) cervikal

6 1. Terapia e rrezikut te abortit bazohet normalisht mbi perdorimin e terapive te meposhtme, me perj ashtim te: A) a ntibio tikoterapise B) progesteronikeve C) beta-stimulueseve D) antispastikeve E) regj imit te shtratit

6 2. Cila prej ekzaminimeve diagnostike te meposhtme eshte me i besueshem per diagnostikimin e gravidances ektopike: A) kuldocenteza B) biospia endometriale C) l a paroskopia D) dozimet seriale te hCG E) roentgenografia direkte e rajonit pelvik

6 3. Nivelet materne j onormale te alfafetoproteines lidhen me te tera patologj ite e meposhtme, me perj ashtim te: A) semundjes hepatike materne B) gravidances multiple C) vdekj es fetale intrauterine D) nefrozes kongjenitale fetale E) hidrocefalise fetale

6 4. ndodh A) B) C) D) E)

Perforacioni i uterusit gj ate eksplorimit instrumental te kavitetit uterin shpesh ne: te tera rastet e meposhtme retroversofleksionin uterin antifleksionin rigid neoplazine malinje te uterusit nje rast me sectio cesareo gjate koheve te fundit

6 5. Te tera vlerat e meposhtme fiziologjike ose laboratorike reduktohen ne trimestrin e dyte te nje shtatzenie normale, me perjashtim te: A) frekuences kardiake B) he m atokritit C) shpejtesise se eritrosedimentimit D) natremise E) vellimit respirator te imet

6 6. Traj timi i hipertensionit gestacional kryhet me te tera terapite e meposhtme, me perjashtim te nj erit: A) antagonisteve te kalciumit B) antikonvulsivanteve C) sulfatit te magnezit D) diuretikeve E) induktimit te lindj es ose sectio cesareo

6 7. A) B) C) D) E)

Cila eshte simptoma specifike e periudhes klimaterike: djersitjet nokturne pagjumesia renia ne peshe frakturat rritja ne peshe

6 8. Te tera prezantimet e meposhtme j ane prezantime cefalike, me perjashtim te prezantimit me: A) verteks B) bregme C) balle D) akro m ion E) fytyre

6 9. Pasi te jete identifikuar nje fetus ne risk ne nje shtatzeni qe e ka kaluar termen, trajtimi me i sakte eshte: A) amniocenteza per vleresimin e pj ekurise pulmonare B) lindja, pavaresisht nga gj endja e cerviksit C) vleresimi i pH nga skalpi fetal D) matja e somatomamotropines korionike humane E) perseritja e vleresimit te mireqenies fetale pas nje jave

7 0. Te nje grua shtatzene, diagnoza e hepatitit mund te vertetohet nga prania e nivelit te rritur te: A) VES B) fosfatazes alkaline C) leukocitozes D) SGOT E) BUN

7 1. Cila prej pohimeve te meposht me mbi leiomimat uterine eshte e sakte: A) jane indikacioni me i shpeshte per kryerjen e histerektomise B) transformimi malinj ndodh ne rreth 20% te rasteve C) kane prirje te regredojne gj ate shtatzenise D) nuk shkaktoj ne infertilitet E) kontraceptivet orale perfaqesojne linj en e pare te trajtimit per reduktimin e madhesise se tyre

7 2. Cila prej te meposhtmeve nuk eshte karakteristike e hiperemesis gravidaru m : A) hipokale m ia B) renia ne peshe C) k eto n e m ia D) nauzeja dhe te vj ellat E) acidoza metabolike

7 3. Nje grua ka nauze dhe te vj ella te patrajtueshme gj ate trimestrit te pare te shtatzenise, qe shkakton renie te lehte ne peshe dhe kerkon nje shtrim ne spital per shkak te anomalive elektrolitike dhe te paaftesise per ta mbajtur ushqimin. Cili prej pohimeve te meposhtme ka me shume mundesi te jete i vertete: A) gruaja eshte mbi 35 vj e B) gruaja ka femije te tj ere C) gruaja ka shume probleme te tjera sociale D) gruaja nuk i pergjigjet kritereve te ngushta te hiperemesis gravidarum E) gruaja ka koriokarcinome malinje

7 4. Nje primigravide 24-vj eare, shtatzene me 2 binjake ne javen e 11-te te shtatzenise, ankohet per nauze dhe te vjella me intensitet modest, qe j ane me te forta ne mengj es. Ka tentuar terapi jofarmakologjike, te cilat kane pasur efikasitet te pakte. Ka tentuar te pije sasi te vogla lengjesh here pas here per te minimizuar te vjellat. Cila do te ishte gj endja karakteristike e saj: A) tro m bocitopeni B) hiperkale m i C) anemi

D) E)

alkaloze reaksion leukomoid

7 5. A) B) C) D) E)

Shtimi mesatar matern ne peshe gjate shtatzenise fiziologjike eshte: 8.0 kg 6 kg 12.5 kg 20 kg 16 kg

7 6. Per cilen prej semundj eve te meposhtme eshte karakteristik transformimi mikropolikistik i vezores: A) sindroma Morris B) semundja Turner C) skleroza tuberoze D) semundja Stein-Leventhal E) pseudohermafroditizmi femeror

7 7. Nje gruaje 27-vjeare i eshte marre gj ak ne j aven e 16-te te shtatzenise. Gjaku dergohet per kontroll te alfa-fetoproteines ne gjakun matern. Rezulton nje nivel i ulet pozitiv. Me cilen prej diagnozave te meposhtme lidhet ky rezultat: A) spina bifida B) anencefali C) 45X D) trimozia e kromozomit 21 E) o m falocela

7 8. Ndodhja e ciklit menstrual varet nga: A) sekretimi i prolaktines nga hipofiza anteriore B) sekretimi pulsues i hormonit stimulues te sekretimit te gonadotropinave (GnRH) C) nga kohezgjatja e luhatshme e fazes folikulare D) nga sinteza e progesteronit ne korpus luteum E) nga sekretimi ovarian i estrogjeneve

7 9. Cili eshte medikamenti me i pershtatshem per nje vaginit nga tri k o m o n a si: A) metronidazoli B) mikonazoli

C) D) E)

dienestroli sulfisoksazoli deoksiciklina

8 0. Nje grua 25-vj eare, primipare ne j aven e 13-te te shtatzenise, ankohet per hemorragji vaginale. Gjate ekzaminimit objektiv ka nje presion arterial 160/95 mmHg, proteinuri 3+ dhe nje fundus uteri ne nivelin e umbilikusit. Cila prej te meposhtmeve eshte diagnoza me e mundshme: A) diabet gestacional B) gravidanza gemelare C) anencefali fetale D) abort i paevitueshem E) gravidance molare

8 1. A) B) C) D) E)

Maturimi i spermatozoideve ndodh ne: epididim us tubujt seminifere kanalin vaginal duktusin deferent kontakt me ovocitin

8 2. A) B) C) D) E)

Rruga me e shpeshte e difuzionit te neoplazive ovariane eshte/jane: venat ovariane enet limfatike te venes ovariane enet limfatike pelvike ekstensioni lokal diseminimi endoperitoneal

8 3. A) B) C) D) E)

Cilat jane shenjat dhe/ose simptomat themelore te preeklampsise: hipertensioni dhe skotomat hipertensioni dhe edemat hipertensioni, cefalea, konvulsionet hipertensioni dhe proteinuria asnj era

8 4. A) B) C) D)

Pilula estro-progestinike vepron kryesisht (me qellim kontraceptiv) ne: hipofize tube collus uteri vezore

E)

mbiveshkore

8 5. A) B) C) D) E)

Prolapsi i kordonit umbilikal mund te shkaktohet nga : asnje prej pergj igjeve prezantimeve anormale disproporcioneve fetus-pelvis gj atesise se madhe (mbi 75 cm) te tera pergj igjet jane te sakta

8 6. sakta A) B) C) D) E)

Te tera pohimet e meposhtme mbi karcinomen in situ te cerviksit j ane te me perjashtim te njeres: ka prekje te te tere trashesise se epitelit skuamoz ka qeliza qe ngj asojne me ato te nje karcinome invazive ka shenje per invazion stromal ka humbje te plote te stratifikimit ka regresioni dhe zhdukje okazionale

8 7. A) B) C) D) E)

Metoda me e mire per te identifikuar placenten previa eshte: ekografia a mniograf ia ekzaminimi gjinekologjik arteriografia selektive e arterieve uterine radiografia e pelvisit

8 8. rregullimet neurologj ike, alterimet okulare, hidrocefalia dhe kalcifikimet cerebrale karakterizoj ne formen kongjenitale te: A) gonorrese B) t o ksop laz mo zes C) semundjes nga citomegalovirusi D) sifilizit E) leis h m a nio z e s

8 9. Cila prej karakteristikave te meposhtme nuk eshte element i treguesit te Apgar: A) ngj yra e lekures B) tonusi muskulor C) temperatura trupore D) frekuenca kardiake E) impenjimi respirator

9 0. A) B) C) D) E)

Hormoni hipokalcemiant eshte: kalcitonina parathormoni kortizoni T SH MSH

9 1. eshte: A) B) C) D) E)

Indikacioni me i rendesishem per kirurgjine rindertuese te uterusit dopio aborti abitual dismenorrea m eno m etrorragjia dispareunia lindja e parakohshme

9 2. Cila prej patologj ive te meposhtme shoqerohet me shpesh me oli g o i d r a m nio s: h A) agenezia renale B) atrezia duodenale C) hidropsi fetal D) anencefalia E) gravidanza multiple

9 3. Dismenorrea primare shkaktohet nga kontraktime uterine qe lindin nga sekretimi i prostaglandinave ne endometer. Masat farmakologj ike per kontrollin e dhembj es perfshijne te tere medika mentet e meposhtme, perve: A) ibuprofenit B) etenil estradiolit me noretindron C) b romk riptines D) indo m etacines E) acidit mefenamik

9 4. A) B) C) D) E)

Perqendrimi i fibrinogj enit ne gjakun matern gjate shtatzenise: ulet shume ulet ne menyre te moderuar mbetet i pandryshuar rritet shume rritet ne menyre te moderuar

9 5. fare kuptohet me impenjim te pjeses se prezantuar: A) koha gjate se ciles koka e fetusit kalon planin e ngushtices superiore B) kalimi i pj eses se prezantuar te ngushtices superiore C) koha gj ate se ciles ndodh kalimi i perimetrit me te madh te pj eses se prezantuar permes planit te ngushtices superiore D) kalimi i kokes pertej planit te ngushtices superiore E) kalimi i pjeses se prezantuar pertej ngushtices inferiore

9 6. Nje grua 24-vj eare ka amenorre dytesore dhe nje test shtatzenie negativ. Nese administrimi i progesteronit nuk provokon gj akosje uterine, cila prej te meposhtmeve mund te jete nje mundesi e logj ikshme diagnostike: A) sindroma e ovarit polikistik B) gravidanca ektopike C) sindroma Turner D) insuficienca ovariane premature E) hipertiroidizmi

9 7. fare eshte T esti Post Koital: A) testi i sieropozitivitetit postkoital B) testi i shtatzenise qe kryhet nepermjet vleresimit te niveleve te hCG C) percaktimi i perqendrimit te spermatozoideve me levizsh meri lineare normale ne mukusin cervikal postkoital D) elektromiografia postorgazmike E) nuk ekziston

9 8. A) B) C) D) E)

Cili hormon stimulon maturimin e folikulit ovarian: FSH LH estrogj eni progesteroni te stost er oni

9 9. A) B) C) D) E)

fare eshte onfalocela: hernie cerebrale aplazi e paretit abdominal te raj onit umbilikal hernie inguinale kongjenitale divertikul hernie krurale

100. A) B) C) D) E)

Per cilin tumor eshte me i pershtatshem markuesi serik CA 125: tumorin e sinusit endometrial tumorin e granulozes-tekes se ovarit kistadenokarcinomen seroze tumorin e qelizave Sertoli-Leydig koriokarcinomen

101. A) B) C) D) E)

Cili eshte diametri me i madh i kokes se fetusit: diametri biparietal diametri bitemporal diametri suboksipito-bregmatik diametri verteks-mjeker diametri oksipito-frontal

102. A) B) C) D) E)

Shkaku me i shpeshte i vdekjes se grave qe shfaqin eklampsi eshte: ruptura e heparit insuficienca renale akute hemorragj ia cerebrale embolia pulmonare shoku septik

103. Te nje grua 23-vj eare e martuar pak kohe me pare eshte gjetur ne menyre te perseritur nje presion arterial 145/95 mmHg. Anamneza tregon qe pin nje pakete cigare ne dite, por nuk ka anamneze familjare per hipertension. Ekzaminimi objektiv eshte negativ. Eshte duke perdorur kontraceptive trifazike orale. Cili eshte shkaku me i mundshem i hipertensionit te kjo grua: A) hipertensioni esencial B) sindroma Turner C) hiperplazia fibromuskulore e arteries renale D) feokrom ocito m a E) etiologjia jatrogj ene

104. uterin A) B) C) D) E)

Ne rast perforimi te uterusit gj ate ekzaminimit instrumental te kavitetit duhet: te vazhdohet manovra diagnostike te vazhdohet me histerektomi te menjehershme te kryhet nje laparotomi eksplorative te kryhet ekografia urgjente te nderpritet ekzaminimi dhe te fillohet terapia antibiotike

105. sakte: A) B) C) D) E)

Cila prej pohimeve te meposhtme mbi fibroadenomen e sinusit eshte e eshte e fortme dhe e levizshme kthehet ne malinje ne 5% te rasteve pergjithesisht eshte e lokalizuar ne pj esen inferiore te sinusit zakonisht paraqet kalcifikime ne mamografi eshte e dhembshme perpara menstruacioneve

106. Cili prej agjenteve infektive te meposhtem nuk shkakton keqformi me e m brio-fetale: A) virusi i rozolise B) h e rpesviru si C) toxoplasma gondii D) gardnerella vaginalis E) c it o m e g a l o v ir u si

107. Cili prej te meposhtmeve nuk eshte nje test i zakonshem kontrolli ne nje shtatzeni fillestare pa nderliki me: A) dozimet seriale te hCG B) he m oglobina C) testet imunologj ike D) Pap-testi E) grupi i gjakut dhe faktori RH

108. A) B) C) D) E)

Vena ovariane dextra derdhet ne venen: renale dextra mezenterike inferiore kava inferiore iliake komune dextra hipogastrike dextra

109. Cila prej te meposhtmeve nuk eshte nje pasoje e mundshme e diabetit ne shtatzeni: A) aborti i trimestrit te pare B) prapambetja e rritjes intrauterine C) p olihidra mni os i D) placenta previa E) makrosomia fetale

110. Te tera informacionet e meposhtme mbi endometriozen j ane te verteta me perj ashtim te nj eres: A) provokon insuficience renale B) transformimet malinje j ane te rralla C) eshte me e shpeshte ne moshen fertile D) grate e semura mund te paraqesin infertilitet E) organi me i prekur eshte vezorj a

111. A) B) C) D) E)

Gjate shtatzenise, fibromat uterine kane prirje qe: te zvogelohen te rriten te mos ndryshoj ne te kolikuojne te kryej ne transformim malinj

112. Cili prej hormoneve te meposhtem nuk derivon nga sinteza ose prodhimi placentar: A) gonadotropina korionike humane B) hormoni laktogj en placentar C) p r olak tina D) p r ogester oni E) e s t r i ol i

113. Cili eshte trajtimi me i mire per inkontinencen urinare nga paqendrueshmeria e detrusorit: A) kolporrafia anteriore B) nderhyrja fionde C) far makoterapia D) te tera pergjigjet E) rehabilitimi muskulor i dyshemese pelvike

114. A) B) C) D) E)

Diagnoza e ovulacionit mund te vendoset duke vezhguar: modifikimet ciklike te mukusit cervikal te tera pergjigjet modifikimet ciklike te citologjise vaginale modifikimet ciklike te temperatures bazale modifikimet sekretore te endometrit

115. A)

Qelizat intersticiale te Leydig prodhoj ne: spermatogjene

B) C) D) E)

estrogj ene testosteron mineralkortikoide glukokortikoide

116. A) B) C) D) E)

IVG brenda 90 diteve mund te kerkohet nga: gruaj a partneri mj eku kurues mjeku gj inekolog komiteti etik i struktures sanitare

117. Nese nje homozigot normal iftohet me nje homozigot per nje gj en recesiv, femij et do te jene: A) te tere normale B) te tere te semure C) 50% normale D) 75% normale E) 25% normale

118. Te tera patologjite e meposhtme kane nje incidence me te madhe gjate shtatzenise multiple, me perjashtim te njeres: A) preeklampsia materne B) mikrosomia fetale C) atonia uterine postpartum D) vasa previa E) morbiditeti perinatal

119. Me perjashtim te njerit, te tere faktoret e meposhtem jane prognostike per karcinomen e endometrit: A) citologj ia peritoneale B) thellesia e invazionit miometrial C) metastazat limfonodulare D) mosha e pacientes E) diferencimi histologj ik

120. A) B) C)

Cili eshte estrogjeni me i perdorur ne kontacepsionin hormonal: e tini lie stradio li estron sulfati estrioli

D) E)

estradioli raloksifeni

121. A) B) C) D) E)

Eklampsia karakterizohet nga: cefale te lehta dhe tranzitore hiperrefleksi konvulsione te tipit gran mal konvulsione te tipit te lehte skotoma

122. Cili prej te meposhtmeve nuk shkak a menorreje: A) sindroma Turner B) diabeti mellitus C) sindroma e ovarit polikistik D) hiperplazia surrenaliene kongj enitale E) h i p e r p r o la k ti n e m ia

123. Cila prej patologj ive te meposhtme nuk mund te diagnostikohet ne periudhen prenatale me teknikat e gjenetikes molekulare: A) f enilketonuria B) fibroza kistike C) fibroza muskulare Duchenne D) defektet e tubit neural E) beta-talase mi a

124. A) B) C) D) E)

Disfunksionet seksuale femerore prekin popullaten me prevalence: 45% 25% 10% 5% nuk ekzistoj ne studime klinike dhe si rrj edhoje prevalenca nuk njihet

125. Zhvillimi i klitorisit dhe i labra major te vagines mundesohet nga veprimi i: A) estrogjeneve B) progesteronikeve C) androgjeneve D) gonadotropinave E) prolaktines

126. Nje balerine 16-vj eare, shume e dobet, ankohet per 6 muaj amenorre dhe ndj esi fryerje abdominale. Cila prej te meposhtmeve eshte diagnoza me e m undshme: A) shtatzenia B) sindroma premenstruale C) d ep res ion i D) anoreksia nervore E) hipotiroidizmi

127. A) B) C) D) E)

Galaktocela eshte: nje kist me retension qumeshti nje granulome lipofagike nje ektazi e duktuseve nje forme mastiti kronik nje nekroze mamare

128. A) B) C) D) E)

Cili eshte rezervuari natyror i toksoplazmes: femijet ne moshe shkollore ekskrementet e zogj ve peshqit e kontaminuar macet gjaku

129. Te nje paciente me preeklampsi, cila prej anomalive te meposhtme ndeshet me shpesh gj ate ekzaminimit te urines: A) proteinuria B) h ematur ia C) glukozuria D) ketonuria E) asnj era

130. A) B) C) D) E)

Cili prej risqeve materne te meposhtme rritet gjate gravidances multiple: infeksionet e rrugeve urinare hipertensioni i induktuar nga shtatzenia diabeti gestacional nivelet e uleta te alfa-fetoproteines serike te nenes moniliaza vaginale

131. A) B) C) D) E)

Kisti Bartolini me absces duhet: te asportohet me rruge kirurgjikale te drenohet duket kryer nje marsupializim te kurohet me bursa te ngrohta e te lageshta te kurohet me antibiotike te kontrollohet shpesh, pa kryer nderhyrje kirurgjikale

132. A) B) C) D) E)

Arborizimi si fier i mu kusit endocervikal verehet: ne shtatzeni ne menopauze ne fazen ovulatore ne periudhen mestruale ne rastet me hiperestrinizem

133. Gj ate javes se pare te puerperiumit, ne cilen perqindjen uterusi e humbet peshen e tij ne raport me periudhen menj ehere postpartum: A) 5% B) 20% C) 50% D) 80% E) 90%

134. A) B) C) D) E)

Cilin aspekt histologj ik merr me shpesh karcinoma e cerviksit uterin: adenokarcinome karcinome mukoep idermoidale karcinome epidermoidale koriokarcinome kistosarkome filoide

135. Cili prej tumoreve ovariane te meposhtme, aktiv nga pikepamja endokrinologj ike, shoqerohet me shpesh me pubertet te parakohshem te grate: A) disgerminoma B) ginandoblastoma C) fibroteko m a D) tumori i qelizave te granulozes-te kes E) asnj era

136. Cili eshte lokalizimi me i shpeshte i implantimit te embrionit ne gravidancen ektopike: A) vezorj a

B) C) D) E)

fundus uteri peritoneumi tuba cerviksi uterin

137. Miomat shoqerohen me te te tera patologjite klinike te meposhtme, perve se me: A) anemi B) pielonefrit C) poliakiuri D) dismenorre E) amenorre

138. A) B) C) D) E)

Amniocenteza per vleresimin e kariotipit fetal mund te kryhet: ne j avet 8-10 ne javet 10-12 ne javet 16-18 ne j avet 22-24 ne do periudhe te shtatzenise

139. Te tera vlerat e meposhtme fiziologjike ose laboratorike pesojne luhatje ne trimestrin e dyte te nje shtatzenis normale, me perjashtim te: A) frekuences kardiake B) he m atokritit C) shpejtesise se eritrosedimentimit D) natremise E) vellimit respirator te imet

140. A) B) C) D) E)

Hiperplazia glandulare kistike e endometrit shkaktohet nga: nje stimulim estrogj enik i zgjatur nje stimulim progestinik i zgj atur nje endometrit nje endometrioze nje karcinome e portio uterina

141. Dismenorrea primare duket se shkaktohet nga kontraktimet uterine nga sekretimi i prostaglandinave nga ana e endometrit. Masat farmakologj ike per kontrollin e dhembjes perfshijne perdorimin e te tera substancave te meposhtme, me perj ashtim te: A) ibuprofenit

B) C) D) E)

etinil estraiolit me noretindron bromkriptines indometacines acidit mefanamik

142. Cili hormon korrelohet shume me trofizmin e organeve gjenitale femerore: A) estradioli B) testosteroni C) DHEAS D) delta4-androstenedioni E) hormoni i rritj es

143. Nje grua 24-vj eare ankohet per sekrecione vaginale te bezdisshme dhe me prurit. Nje strisho e sekrecioneve vaginale tregon nje protozoo shume te levizshem. Cilet prej agj enteve farmakologj ike te meposhtem do te ishte traj timi me i pershtatshem: A) metronidazoli B) klotrimazoli C) mikonazoli D) acikloviri E) spek tin omicin a

144. Ne cilen moshe gestacionale (te perllogaritur nga cikli i fundit menstrual) eshte me i ndjeshem ndaj efekteve teratogj ene fruti i koncepsionit: A) ditet 1-14 B) ditet 15-28 C) ditet 29-72 D) ditet 73-90 E) ditet 91-120

145. Cila prej pohimeve te meposhtme mbi hepatitin B dhe shtatzenine eshte i sakte: A) shtatzenia pershpejton dekursin e hepatitit B akut matern B) menyra e lindjes nuk ka asnje ndikim mbi transmetimin nene-neonat C) ushqyerja me gji nuk e rrit riskun neonatal per te marre hepatitin B D) te porsalindurit mund te mbrohen nga hepatiti B duke marre imunizim pasiv ne diten e pare te lindjes E) shpejtesia e perparimit te semundj es eshte e njej te te nena dhe te neonati

146. Perqindja e aborteve te shkaktuara nga amniocenteza e kryer ne trimestrin e dyte eshte rreth: A) 1% B) 3% C) 4% D) 5% E) >5%

147. A) B) C) D) E)

Nga cili hormon induktohet plasja e folikulit ovarian: prolaktina FSH GH estradioli LH

148. Cila eshte shenja me e shpeshte e hapjes iminente te nje plage kirurgjikale: A) drenimi serik B) drenimi i gjakut C) drenimi sero-hematik D) dhembja abdominale pa drenim E) prania e permbajtjeve abdominale ne plage

149. A) B) C) D) E)

Numri me i madh i vdekj eve nga karcinoma cervikale shkaktohet nga: ekstensioni lokal metastazat ne sistemin nervor qendror metastazat ne mushkeri shkaqet jatrogj ene asnj era

150. Nje grua 20-vj eare paraqet hemorragji te tepert vaginale 1 ore pas lindjes vaginale spontane te nje neonati mashkull (femija i trete) me peshe 4400 g. Lindja ka zgj atur 7 ore me administrim o ksitocine gjate 2 oreve te fundit. Cili eshte shpj egimi me i mundshem per hemorragjine postpartum te saj: A) retensioni i indit placentar B) trauma e traktit gj enital C) atonia uterine D) inversioni uterin E) rregullimi i koagulimit

151. A) B) C) D) E)

Humbja mesatare e gjakut nga menstruacioni eshte: 10-15 ml 25-50 ml 75-100 ml 101-125 ml 130-150 ml

152. A) B) C) D) E)

pH vaginal korrelon me: perberj en e ekosistemit vaginal nivelin e estrogjeneve ne indet vaginale cenueshmerine nga infeksionet nga bakteret e kolonit moshen e pacientes te tera pergj igjet jane te sakta

153. Trajtimi paresor i karcinomes endometriale te kufizuar ne korpus uterii konsiston ne: A) terapia rrezatimi ekstern B) radium intrakavitar C) histerektomi D) k emio terapi E) terapi progestinike

154. Cila prej simptomave te meposhtme nuk ndeshet ne rastin e gravidances tubarike: A) dhembje ne kuadrantet inferiore abdominale B) humbje hematike vaginale ose spotting C) amenorre D) proktorragji E) dhembje ne shpatull

155. Cili prej te meposhtme ve nuk eshte indikacion absolut per sectio cesareo: A) prezantimi frontal B) placenta previa centrale C) vuajtja fetale akute ne fillim te procesit te lindjes D) prolapsi i funikulit E) sectio cesareo i meparshem

156. A)

Cili eshte treguesi fetal i prezantimit podalik: sakrumi

B) C) D) E)

tibia peroneumi iskiumi vertebra e 5-te lumbare

157. Cila prej metodikave te meposhtme te diagnozes prenatale kryhet ne periudhen me te hershme gestacionale: A) fetoskopia B) a mniocenteza C) marrja e vileve korionike D) funikolocenteza E) biopsia fetale

158. Ciklofosfamidi mund te shkaktoje te tera efektet e meposhtme kolaterale, me perjashtim te: A) nauzese dhe te vj ellave B) inhibimit medular C) stomatitit D) cistitit hemorragj ik E) alopecise

159. A) B) C) D) E)

Vaginizmi eshte shkaku fe meror kryesor i: d ispar eu nise moskonsumimit te marteses ejakulacionit te parakohshem krizes bashkeshortore uljes se deshires seksuale te femres

160. Te nje grua me cikle te rregullta 28-ditore, periudha fertile presupozohet te jete: A) ditet 2-5 B) ditet 7-11 C) ditet 10-16 D) ditet 18-26 E) ditet 20-29

161. A) B) C)

Gjinekomastia e vertete shfaqet gjate: trizomise se kromozomit 21 sindromes Turner sindromes Klinefelter

D) E)

morbus Addison adenomes Plummer

162. Cila eshte diagnoza me e mundshme te nje grua 24-vjeare, qe paraqet ne gj irin e djathte nje mase te levizshme, te izoluar, te kufizuar mire, me konsistence si gome dhe te padhembshme: A) karcinome mamare B) semundje fibrokistike te mameles C) f ibr oadeno me D) absces mamar E) nekroze dhj amore te ma meles

163. Cila/cilat prej situatave te meposhtme eshte/j ane e lidhur me nje rritje te mundesise se hemorragj ise postpartum: A) procesi i shpejte i lindj es B) procesi i gjate i lindj es C) stimulimi i lindjes me oksitocine D) gravidanza multiple E) te tera pergj igjet jane te sakta

164. Nje vaj ze 19-vjeare shkon te mjeku sepse nuk ka pasur kurre menstruacione. Ekzaminimi objektiv ve ne dukje pranine e gj inj ve normale, por nuk arrihet te individualizohet uterusi as me palpim dhe as gjate ekzaminimit pelvik. Cili prej ekzaminimeve te meposhtme do te ishte me i dobishmi per identifikimin e shkakut te amenorrese: A) LH dhe FSH B) prolaktina C) estrogj eni D) progesteroni E) te stost er oni

165. Te nje paciente ne moshe pediatrike me rrj edhje vaginale te vazhdueshme fillimisht duhet te kerkohet: A) trupi i huaj B) oksuria C) abuzimi i medika menteve D) lacerimet vaginale E) ureteri ektopik

166. A nderhyn statusi i infertilitetit ne marredhenien seksuale te nje ifti te qendrueshe m : A) vetem nese eshte e prekur gruaja B) vetem nese eshte i prekur burri C) shume shpesh D) kurre E) ndonjeher e

167. A) B) C) D) E)

Shkaku me i shpeshte i dispareunise se thelle eshte: sindroma Master Allen semundja inflamatore pelvike (PID) endometrioza varikocela sindroma e bllokimit te nervave abdominale (ACNES)

168. A) B) C) D) E)

Placenta prodhon ne menyre fiziologjike: estrogj en dhe progesteron gonadotropina korionike substance somatotroponosimile kortikosteroide te tera pergj igjet jane te sakta

169. Nje vaj ze 15-vj eare paraqet dhimbje abdominale te papritur. Ekzamini mi objektiv ve ne dukje nje mase te dhembshme ne aneksin e majte. Testi i shtatzenise eshte negativ. Radiografia tregon nje lezion hapesirezenes ne vezoren e maj te me zona kalficikimi fokal. Cila eshte diagnoza me e m undshme: A) kist folikular B) kistadenome mucinoze C) teratome kistike D) tumor Brenner E) kistadenome seroze

170. Cila prej formacioneve te meposhtme anatomike e merr prejardhjen nga duktusi Wolff te gruaj a: A) tubat uterine B) uterusi C) vagina D) ligamenti i gj ere E) asnjera prej formacioneve te mesiperme

171. A) B) C) D) E)

Mastiti akut ndodh me shpesh gjate: pubertetit l akt ac ionit ciklit menstrual normal shtatzenise menopauzes

172. Te tera pohimet e meposhtme mbi kancerin e vezores j ane te sakta, me perjashtim te nj eres: A) eshte kanceri me i shpeshte i aparatit gjenital femeror B) ka perqindjen me te larte te vdekshmerire nder tumoret me te shpeshta te aparatit gj enital femeror C) ka prirje qe te jete asimpto matik deri sa arrin stadin e perparuar D) zhvillimi i tij mund te ndikohet nga faktore ambientale, kulturore, socioekonomike E) shtrishot e Papanicolau (Pap test) nuk jane te efektshme per kontrollin diagnostik rutine

173. Nje primigravide 18-vjeare eshte ne j aven e 26-te te shtatzenise. Kultura e cerviksit eshte pozitive per Chlamydia trachomatis. Cili prej antibiotikeve te meposhtme eshte me i pershtatshem per kete paciente: A) dikloksacilina B) ampicilina C) cefuroksa midi D) vanko micina E) eritro m icina

174. A) B) C) D) E)

Gj ate jetes fetale, duktusi arterioz i Botallo sherben: si qark i shkurter per qarkullimin pulmonar per te shkembyer gjakun ndermj et gj ymtyreve si qark i shkurter per heparin ne zhvillim per te parandaluar koarktacionin asnjera prej pergjigjeve te mesiperme

175. Kater prej patologj ive te meposhtme j ane kundraindikacione absolute per perdorimin e kontraceptiveve hormonale te ko mbinuar. Cila prej patologjive perben te vetmin kundraindikacion relativ: A) anamneza pozitive per tromboflebit B) diabeti mellitus me probleme vaskulare C) hipertrigliceridemia dhe hiperkolesterolemia D) hemikrania me aura

E)

duhanpirja mbi 35 vj et

176. Nje grua 33-vj eare (G1P1), qe ka lindur femijen e saj te pare 5 jave me pare, telefonon ne ambulator. Regeron se ndjehet shume e lodhur dhe anksioze. Ndjen se nuk eshte ne gj endje te beje asgje ne menyren e duhur dhe se ndihet shume e trishtuar gjate te tere dites. Tregon se qan ne menyre te vazhdueshme dhe nuk arrin te fleje gj ate ntes. Cila eshte diagnoza me e sakte: A) depresioni postpartum B) rregullimi i ankthit C) psikoza postpartum D) semundja bipolare E) p a gju m esia

177. Nje paciente diagnostikohet me karcinome mamare. Cili eshte faktori prognostik me i rendesishem per trajtimin e kesja patologj ie: A) mosha e pacientes B) madhesia e tumorit C) metastazat ne limfonodujt ascelare D) prania ne tumor e receptoreve per estrogj enet E) prania ne tumor e receptorve per progesteronin

178. Nje grua 38-vj eare (G3P3) fillon te ushqeje me gji femij en e saj 5ditesh. Femija kapet mire pas gj irit dhe fillon te thithe. Cila prej pergj igj eve ndaj thithjes ndodh te gruaja: A) reduktimi i oksitocines B) rritja e PIF (faktori inhibues i prolaktines) C) rritja e prolaktines hipotalamike D) rritja e dopamines hipotalamike E) rritja e faktorit te lirimit te LH dhe te FSH

179. Nje grua 33-vj eare (G2P1) sapo ka lindur me lindje vaginale spontane. Nuk eshte ndjekur mire ne periudhen prenatale dhe, duke qene se nuk ka kryer asnje ekografi gj ate shtatzenise, eshte anksioze per te ditur seksin e femijes. Ne pamje te pare verehen organe gjenitale femerore, por duke i ekzaminuar me kuj des organet gjenitale verehet se ato duhet te klasifikohen si ambigue. Cili eshte hapi pasues me i mire per vleresimin e ketij te porsalinduri: A) analiza kromozomike B) rivleresimi ne moshen 1-muj ore C) ekografia pelvike D) ekzaminimi pelvik i plote D) laparotomia per kryerjen e gonadektomise

180. Gj ate vizites se nje gruaje 30-vjeare per infertilitet, vendoset diagnoza e uterusit bikorn. Gruas i shpjegohet se nevojiten ekzaminime te metej shme, pasi ekziston nje risk me i madh per anomali kongj enital ne njerin prej sistemeve organike te meposhtme: A) skeletik B) he m atopoietik C) urinar D) sistemin nervor qendror E) g a str o i n te sti n a l

181. Nje grua 74-vj eare paraqitet ne ambulator per nje vizite kontrolli. Pap testi dhe mamografia e fundit e saj jane te 3 viteve me pare. Ajo ka hipertension, nivele te larta te kolesterolit dhe osteoartrit. Ka nderprere pirjen e duhanit 15 vite me pare dhe nuk referon perdorim te alkoolit. Duke u bazuar mbi anamnezen e kesaj pacienteje, sila prej problemeve klinike te meposhtme duhet te jete shqetesimi kryesor i saj: A) semundja Alzheimer B) neoplazia ma mare C) semundja cerebrovaskulare D) semundja kardiake E) tumori pulmonar

182. Nje grua 50-vj eare diagnostikohet me kancer te cerviksit. Cili prej grupeve te limfonoduj ve eshte i pari qe perfshihet pas diseminimit metastazik te kesaj semundjeje, perve cerviksit dhe uterusit: A) limfonodujt iliake komune B) limfonodujt parametriale C) limfonodujt iliake eksterne D) limfonodujt paracervikale ose ureterale E) limfonodujt paraaortale

183. Nje grua 24-vj eare i kerkon mj ekut te saj mundesine e nje kontrolli gj enetik per nje mutacion te BRCA1. Nena e saj ka vdekur si pasoje e karcinomes mamare ne moshen 44-vj eare dhe nje moter e saj eshte diagnostikuar me karcinome duktale in situ ne moshen 38-vjeare. Cila eshte keshilla me e pershtashme per kete grua: A) shpjegohet se mutacionet e BRCA1 nuk j ane te lidhura me nje risk me te larte per karcinome ma mare B) rekomandohet kryerja e kontrollit vetem nese eshte pasardhese e hebrenjve Ashkenazi C) rekomandohet nje keshillim gjenetik perpara se te kryhet kontrolli gj enetik

D) sugj erohet qe t nenshtrohet nje interventi profilaktik mastektomie bilaterale ne vend te kontrollit gjenetik E) rekomandohet kryerja e nje biopsie mamare bilaterale

184. Nje grua 24-vj eare sapo eshte diagnostikuar me toksoplasmoze. Mundoheni te percaktoni se cilet mund te kene qene faktoret e saj te riskut. Cili prej tyre perben riskun me te larte: A) konsumimi i mishit te pagatuar B) konsumimi i peshkut te pagatuar C) prania e nje qeni ne shtepi D) kombesia angleze E) fakti qe ka pasur nje infeksion viral te parakohshem gjate shtatzenise

185. Nje shtatzene 35-vj eare ndodhet nen risk shume te larte per te zhvilluar njekohesisht neoplazi malinje te: A) cerviksit B) vezores C) gjirit D) vagines E) kolonit

186. Nje grua 23-vj eare (G1P0) sapo ka pasur nje lindje vaginale spontane. Nderkohe qe nxirrej placenta, verehet nje mase e mishte e kuqe, qe protrudon nga prapa placentes. Cili eshte hapi pasues me i mire per tu hedhur te kj o paciente: A) fillimi i infuzionit endovenoz te oksitocines B) kerkohet me shume personel mjekesor per nje asistence te m enjehershme C) vazhdohet heqja manuale e placentes D) i indikohet anestezistit qe te administroje sulfat magnezi E) shtyhet placenta brenda uterusit 187. Pas nje shtatzenie te pare, qe rezultoi ne nje humbje spontane ne trimestrin e pare, pacientja eshte e shqetesuar nga mu ndesia qe kjo gje te ndodhe serish. Cila eshte pergj igja me e pershtatshme ne lidhje me mundesine e nje rekurrence: A) mundesia varet nga perberja gjenetike e frutit te abortit te meparshem B) mundesia nuk ndryshon nga mundesia qe kishte aborti i pare C) mundesia eshte rritur rreth 50% D) mundesia eshte rritur mbi 50% E) mundesia varet nga seksi i frutit te abortit te pare

188. Nje grua 33-vj eare (G3P2) ne javen e 30-te te shtatzenise merr nje infeksion. Cili prej te meposhtmeve eshte nje riaktivizim dhe si rrjedhoje nuk perben risk per fetusin: A) coxsackiervirus grupi B B) virusi i rozolise C) virusi i variceles D) zj arri i Shen Andonit E) herpesvirusi tip 2

189. Nje grua 30-vj eare me nje patologji gj enetike te karakterizuar nga nje mungese e fenilalanine hidroksilazes, po mendon nje shtatzeni per here te pare. Mj eku i saj i shpjegon riskun me te larte te femij es se saj per prapambetje mendore dhe per semundje kardiake kongjenitale. Cili rekomandim duhet bere: A) perpara koncepsionit duhet te fillohet nje diete me ni vele te uleta te f e n ila la n i n e s B) rekomandohet nje suplement me glicine ne diete C) rekomandohet nje suplmenet me L-karnitine ne diete D) nuk ka nevoje per kontroll dietetik nese nivelet e fenilalanines jane lehtesisht te rritura E) te porsalindurit duhet ti jepet vitamine B12 menjehere pas lindj es

190. Diten pas nje marredhenieje seksuale rastesore me nje burre heteroseksual, i cili sapo eshte diagnostikuar me antikorpe pozitive ndaj HIV, pacientja j uaj eshte e shqetesuar dhe do te dije nese mund te jete e infektuar. Rezulton nje titer antikorpal negativ. Per te testuar mundesine e sierokonversionit, cila eshte periudha me e hershme per rivleresimin e pranise se antitrupave pas nje takimi seksual: A) 1-2 j ave B) 3-4 jave C) 4-10 jave D) 12-15 j ave E) 26-52 j ave

191. Nje grua 27-vj eare sapo ka pasur nje gravidance ektopike. Cila prej ngj arjeve te meposhtme eshte me e mundshmja qe te predispozoje per nje gravidance ektopike: A) nje konizim cervikal i meparshem B) semundja inflamatore pelvike (PID) C) perdorimi i mj eteve kontraceptive intrauterine (DIU) D) induktimi i ovulacionit E) ekspozimi in utero ndaj dietilstilbisterolit (DES)

192. Nje vaj ze 18-vjeare (G2P1), qe diten e pare te ciklit te fundit menstrual e ka pasur me date 7 maj, vj en per viziten e saj te pare obstetrike pas 10 javesh. Cila mund te jete data e mundshme e lindjes: A) 28 janari i vitit tjeter B) 7 shkurti i vitit tjeter C) 14 shkurti i vitit tjeter D) 21 shkurti i vitit tjeter E) 28 shkurti i vitit tjeter

193. Nje grua 30-vj eare shkon te mjeku pasi ka frike se mos zhvillon nje neoplazi ovariane. Gj yshja e saj 70-vj eare ka vdekur gjate koheve te fundit si pasoje e nje karcinome ovariane. Diskutohen faktoret e riskut dhe parandalimin e neoplazise se ovarit. fare mund ta reduktoje riskun e shfaqjes se neoplazise se ovarit: A) perdorimi i nje terapie te kombinuar me kontraceptive orale B) menopauza pas moshes 55-vjeare C) perdorimi i medikamenteve antiinflamatore j osteroide D) nullipariteti E) perdorimi i medikamenteve qe induktoj ne ovulacionin

194. Gj ate vleresimit te nje amenorreje dytesore te nje grua 24-vjeare diagnostikohet hiperprolaktinemia. Cila prej problemeve te meposhtme mund te rrise perqendrimin e prolaktines ne qarkullim dhe ame norrene te kjo paciente: A) s t r es i B) hipertiroidizmi paresor C) anoreksia nervore D) hiperplazia kongj enitale e surrenaleve E) semundja e ovarit polikistik 195. Nje vaj ze 18-vjeare ne javen e 8-te te shtatzenise, ankon per nauze dhe te vjella te perditshme gjate j aves se fundit. Nauzeja dhe te vj ellat jane simptoma te shpeshta te fazes fillestare te shtatzenise. Cili prej problemeve te meposhtme do te tregonte mundesine e nje diagnoze me serioze te hiperemesis gravidaru m : A) hipotiroidizmi B) hipokalem ia C) shtimi ne peshe D) proteinuria E) diarrea

196. Gj ate vleresimit per infertilitet te nje gruaje 25-vjeare, kryhet nge histerosalpingograme qe tregon shenja te sindromes Asherman. Cilen prej simptomave te meposhtme priten te verehen te kjo paciente: A) dismenorre

B) C) D) E)

h i po menorre m e n o rr a g ji metrorragji oligomenorre

PEDIATRIA 1. T gj itha pohimet e mposhtme j ane tregues prognostike pozitiv te Leuemise Limfoblastike akute n moshn pediatrike prve: A) Mosha 3-7 vj e. B) Seksi mashkull C) Limfocitoz fillestare <10 000/ mm3 D) Vlera e hemoglobins < 7g/dL E) Trombocitet >100 000 mm3

2. Kur dyshoj m pr nj invaginacion intestinal tek fmij a, cili sht egza minimi diagnostik q prjashton diagnozn? A) Klizma opake B) Radiografi e drejtprdrejt e abdomenit C) Radiografi me kontrast bariumi D) Stratigrafi E) Invertogram

3. Cili sht simptomi m i shpesht me t cilin fillon kraniofaringioma te fmijt? A) Strabizm B) Dhimbje koke C) Ngadalsim n rritje D) Turbullime t shikimit

E)

Marrje mendsh

4. A) B) C) D) E)

Divertikuli i Meckel-it mund t jet: Simptomatik vetm n seksin mashkull A sim ptomatik Gjithnj simptomatik Prcaktohet vetm me autopsi Simptomatik vetm n seksin femr

5. Cili nga komponentt e mposhtm gjendet m tepr n qumshtin e gj irit n krahasim me qumshtin e lops? A) Acidet yndyrore t pangopura B) Vitaminat C) Kazeina D) L aktoalbu m ina E) Kripra minerale

6. A) B) C) D) E)

Si prcaktohet diagnoza refluks gastroezofageal: Me an t shinitgrafis ezofageale Ph metria 24 orshe Manometria ezofageale Me an gastroskopis Asnjra nga t msiprmet

7. A) B) C) D) E)

Zhurma e pafaj shme sht zakonisht: Proto mezosistolike E vazhdueshme Mezotelesistolike Diastolike n pozicion t shtrir E pranishme vetm mbas ushtrimit fizik

8. Terapia e krizave astmatike t moderuara: A) Parashikon prdorimin e a2 stimulantve per os dhe n rast se nuk ka prmirsim pr 12 ort n vazhdim , prdoret prednisolon. B) Parashikon prdorimin e a2 stimulantit ipratropium bromid n rrug inhalatore dhe n.q.s nuk kemi prmirsim pr 4 or, prdoret prednisolon. C) Krkon shtrim urgjent n spital. D) Parashikon prdorimin e b2 stimulantt n rrug inhalatore dhe prednizolonit per os.

E)

Teofilina

9. Nj vaj z me mosh 4 vj e, po luante n nj fush t trajtuar me insekticid. Prindrit e sjellin n ndihmn e shpejt n gj ndje letargjie, me sekrecione nga goj a, mioz, lotim dhe inkontinenc urine dhe fekale, t vj ella, dhe dridhje. Mj eku dyshon pr helmim me fosfatoorganik. Pr t trajtuar efektin nikotinidik, cili nga mjekimet e mposhtme duhet prdorur? A) Atopina B) BAL (british antilewisite) C) Pralidoxim (2- Pam) D) Naloxon E) Ca- EDTA

1 0. A) B) C) D) E)

N rastin e nj fmije me gastroenterit akut, cili sht hapi prioritar? Prick- test alergjik. Vlersimi i gj endjes s hidratimi dhe ekuilibri acidobazik. Testi i absorbimit intestinal Fillimi i terapis antibiotike. Krkim i oksiureve n fee.

1 1. A) B) C) D) E)

Cili nuk sht komplikacion i parotitit? Adenit supurativ Pankreatiti Orkiti Gastriti Meningoencefaliti

1 2. A) B) C) D) E)

Cili simptom sht tipik pr sindromin nefrotik? Edema Poliuria Hematuria Glukozuria Hipertensioni

1 3. Infeksioni me Mycoplasma pneumoni mund t shkaktoj t gj itha pathologjit e mposhtme, prve: A) Bronkitit B) Bronkiolitit

C) D) E)

Konjuktivitit Faringotonsilitit Otitis media

1 4. Cili egzaminim prcakton gravitetin n diagnozn klinike t b r o n k i o litit? A) Ekzaminimi hemokromocitometrik. B) Rx e Toraksit C) T e m peratura D) Saturacioni O2 E) Presioni arterial sistemik

1 5. Pr ciln smundje hematologjike n pediatri rekomandohet heparina + plazma e freskt? A) Hemofilia A B) DIC (KID- koagulimi intravazal i diseminuar) C) Smundje von Willebrand D) Hemofilia B E) Difekt i koagulimit si pasoj e hepatopatis

1 6. Nj fmij 4 vj e q merr msime noti, shfaqet tek pediatri me dhimbje n veshin e djatht. Ekzaminimi objektiv i veshit tregon edem dhe eritem t kanalit auditiv me pranin e sekrecioneve me ngjyr t gjelbr. Dhimbja shtohet me prekje t aurikuls dhE)shtypjes mbi tragus. Cili sht shkaktari m i mundshm i ktij infeksioni? A) Hmophilus Influenz B) Pseudomonas aeruginosa C) Moraxella catarrhalis D) Streptococcus agalacti E) Escherichia coli

1 7. Nj neonat i gjat pr moshn gestionale paraqitet me lkur t rrudhur dhe t deskuamuar. Thonjt jan t gj ata dhe t gjelbra. Nuk ka prezenc pushi. Sa sht mosha gestionale e ktij fmije? A) 34 jav B) 36 jav C) 38 jav D) 40 jav E) 42 jav

1 8. A) B) C) D) E)

Cilat jan pasojat e rubeols kongenitale? A nencefalia Kardiopathia S hurdhim Verbim T gjitha t msiprmet

1 9. Cila ngA)ndrhyrjet e mposhtme sht e detyrueshme dhe prioritare tek nj fmij me arrest kardiocirkulator? A) Ekzaminimi biokimik dhe instrumental. B) Lirimi i rrugve t frym marrj es, ventilacion dhe masazh kardiak. C) Monitorim i presionit arterial dhe diurezs D) Vendosja e vigonit dhe fillimi i serumit endovenoz. E) Administrimi i medikamenteve jetike.

2 0. Intoleranca ndaj proteinave t qumshtit mund t manifestohet me t gj itha kuadret klinike t mposhtme, prve: A) Form akute me diarre dhe t vj ella B) Form kronike me kolit / ose anemi C) T vjella dhe acidoz metabolike. D) Dermatit atopik dhe urtikarie E) Diarre me gj ak okult n fee.

2 1. Cila nga pathologjit e mposhtme te sistemit genital, mund t jet shkaktar i abdomenit akut tek vajzat? A) Torsion i ovarit B) Hipotrofia e klitorisit C) Hipertrofia e klitorisit D) Hipertrofia e buzve t vogla E) Hipertrofia e buzve t mdha

2 2. Sasia e yndyrnave n dietn e fmijs s vitit t dyt t j ets,kshillohet n nj nga opsionet e mposhtme: A) Nn 10% t kalorive ditore. B) Nn 15% t kalorive ditore. C) Jo m tepr se 15% t kalorive ditore. D) Jo m tepr se 30% t kalorive ditore E) M tepr se 40% t kalorive ditore.

2 3. Cili nga trajtimet e mposhtme duhet t prdoret si medikament i linjs s par n rastin e nj ataku akut t astms te fmijt. A) 2 agonist per os B) Kortikosteroid per os C) 2 agonist n rrug inhalatore D) Antihistaminik per os E) 2 agonist per os dhe bikarbonat natriumi.

2 4. T gj itha pohimet e mposhtme jan tipike pr stenozn hipertrofike t pylorit, prve: A) Alkalozs metabolike B) Haset m shpesh tek meshkujt C) Shfaqet mbas moshs 2-3 j avshe. D) T vj ella me prmbajtje biliare. E) T vj ella pa prmbajtje biliare

2 5. A) B) C) kan D) E)

N cilin rast nuk indikohet profilaksia antimeningokoksike? Kontakte familjare me t smurin Kontakte me t smurin n erdhe gjat 7 ditve paraardhse. Subjekte n kontakt t drejtprdrejt me sekrecionet e pacientit, ose q prdorur objekte t prbashkta. Personeli sanitar q nuk ka pasur ekzpozim t drejtprdrejt me pacientin. Subj ekte q kan fjetur ose ngrn me pacientin.

2 6. me: A) B) C) D) E)

Hiperplazia gingivale mund t shfaqet tek nj pacient i cili merr mj ekim Meticilina A m picilina Eritromicina Tetraciklina Diphenylhydantoina

2 7. Cili nga faktort e mposhtm mendohet se lidhet me sindromin e vdekj es s papritur foshnjore (SIDS)? A) Pozicioni shtrir mbi bark B) Prdorimi i biberonit C) Toksikoza e shtatzanis D) K ardiopatia E) Luksacionet e lindura

2 8. A) B) C) D) E)

T gj itha sindromat e mposhtme j an uropati malformative, prve: H idroureteronefroza Ren polikistik Hipertrofia e klitorisit Megaureter obstruktiv primar Refluksi vezikoureteral

2 9. T gjitha komplikacionet e mposhtme mund t gjenden tek neonati i nj ma maje diabetike, prve: A) Ikter B) Policit e mi a C) Trauma t lindj es D) Hernia umbilikale E) Anomali kongj enitale

3 0. T gjitha t mposhtmet j an manifestime ekstraintestinale t alergjive nga ushqimet, prve: A) Dermatit atopik B) Urtika ries C) Dhimbj es s koks D) Bronkospazma, kolla e that e vazhduar. E) O nikomikoza

3 1. A) B) C) D) E)

Cila sht etiopatogjeneza e hernies inguinale te fmij a? Dobsi e paretit abdominal Moszbritja e testikulit Mosmbyllja e processus vaginalis peritonei Traum e lindjes Inflamacion e kanalit inguinal

3 2. Cili nga markert e mposhtm identifikon pacientin me HBV kronik me risk m t lart infektimi? A) HBsAg B) HBeAg C) IgM anti- HBc D) Antikorpe kundr HBc. E) Antikorpe kundr HBs.

3 3. A) B) C) D) E)

Cila nga pohimet e mposhtme sht i sakt, pr smundj en Lyme? Shkaktohet nga Borrelia vincenti Shkaktohet nga Adenovirus Lkura sht vendndodhja fillestare e infeksionit. Artriti sht nj nga manifestimet e para. Mjekimi kryhet me aminoglikozide.

3 4. A) B) C) D) E)

Hormoni Antidiuretik (Vazopresin) sintetizohet n: Qelizat acidofile t hipofizs anteriore Qelizat bazofilike t hipofizs anteriore Nga qelizat e n. supraopticus dhe n. paraventricularis t hipotalamusit. Nga n. Arcuatus e hipotalamusit. Asnj nga prgj igjet e msiperme.

3 5. thot: A) B) C) D) E)

N.q.s. nj fmij ka nj gjatsi m t shkurtr se percentili i tret do t M tepr se 3% e fmij ve t moshs s tij jan m t shkurtr se ai. M tepr se 97% e fmijve t moshs s tij jan m t gjat se ai. M tepr se 97% e fmijve t moshs s tij jan m t shkurtr se ai. M pak se 3% e fmijve t moshs s tij jan m t gjat se ai. M pak se 97% e fmijve t moshs s tij jan m t gj at se ai

3 6. Nj vaj z 14 vj eare, me nj histori pr hipotireoidizm ka temperatur, anoreksi, amenore dhe ikter q persiston prej 4 muaj sh. Diagnoza m e mundshme sht: A) Mononukleoz B) Kalkuloz biliare C) Defiit i 1- antitripsins D) Smundje e Wilsonit E) Hepatit autoimun

3 7. A) B) C) D) E)

Cila nga smundjet e mposhtme karakterizohet me pranin e petekieve? Purpura thrombocytopenike idiopathike Smundja von Willebrand Hemofilia klasike Munges e faktorve II, VII, IX, X Mungesa e faktorve III, VI, V III

3 8. Tek latantt me infeksione t rrugve urinare, manifestimi klinik m i shpesht sht: A) T e m peratur B) Dizuri C) P olakiuria D) Dhimbje t regjionit lumbar E) I n k o n ti n e n c a

3 9. T gjitha pohimet e mposhtme j an t pranishme tek anemia B12 deficitare, prve: A) Ulje e numrit t eritrociteve B) Ulje e Hemoglobins C) MCV e rritur D) MCV e ulur E) Retikulocite t rritura

4 0. Nj fmij e moshs 10 muaj she me diarre akute t rnd dhe dehidratim ka marr vetm qumsht t skremuar gjat dy ditve t para t diarres. Ciln sht vlera e pritshme e natriemis? A) 102 meq/L B) 124 meq/L C) 136 meq/L D) 142 meq/L E) 154 meq/L

4 1. Infeksioni i rrugve urinare tek fmij t sht shoqrues i refluksit vezikoureteral: A) N 100% t rasteve. B) 30- 50% t rasteve. C) Asnjher D) N 98% t rasteve. E) N 2-3 % t rasteve.

4 2. A) B) C) D) E)

Cili infeksion tek fmija nuk shkaktohet nga Streptococcus pneumoni? Otiti Pn eu moni a Meningiti Cistiti Faringiti

4 3. Cili nga pohimet e mposhtme sht manifestimi m i shpesht i defiitit t 1- antitripsins tek fmijt? A) B) C) D) E) Kistet pulmonare Miokard i ti Cirroza hepatike Insuficienca pankreatike Bronkiti Kronik Obstruktiv

4 4. Cila nga pohimet e mposhtme sht standarti i art pr lokalizimin e nj testikuli t papalpueshm n moshn pediatrike? A) Ekzaminimi objektiv B) Rx direkte e abdominit C) Rezonanca magnetike D) TAK (Tomografia Aksiale e kompjuterizuar) E) Eksplorimi laparoskopik

4 5. A) B) C) D) E)

Nj fmij mE)me sindromn Down, ka rrezik te rritur pr: Malformacione t aparatit urogj enital. Malformacione cerebrale. Malformacione t aparatit digjestiv. Malformacione kardiake Infeksione

4 6. T gj itha simptomat dhe shnjat e mposhtme bjn pjes tek kriteret e madhore te Johns it pr diagnozn e smundjes rheumatike, prve: A) Artralgj i B) Karditi C) Korea D) Eritema marginatum E) Nodule subkutane

4 7. A) B) C) D) E)

Apendiciti akut te fmija sht arsyeja m e shpesht e abdomenit akut: Vetm tek latantt Po Jo Vetm tek fmij t nn moshn 12 muajshe Vetm tek fmija n moshn e pubertetit.

4 8. Cili sht shkaktari m i shpesht i gastroenteritit akut n moshn pediatrike, n vendet e industrializuara, A) E. coli B) Salmonella spp. C) R otavirusi D) A d e n o v ir u si E) Lamblia intestinale

4 9. T gj ith t dhnat laboratorike t mposhtme jan karakteristike pr anemin feriprive, me prjashtim t: A) Ulje e nivelit t hekurit n serum B) Kapacitet i lidhjes s hekurit i rritur C) Ulje e nivele t feritins n serum. D) Ulje e protoporfirins eritrocitare t lir E) Eritrocite mikrocitike

5 0. A) B) C) D) E)

Cila nga thniet e mposhtme pr vaksinn kundr thruthit sht e sakt: Aplikohet te neonati. Aplikohet n moshn 15 muaj. Nuk aplikohet te prematurt. Nuk aplikohet te subj ektet alergjike. Nuk aplikohet te fmijt e nnave diabetike.

5 1. Nj nn q i jep gji foshnjes s saj 3 muajshe, diagnostikohet me mastit t gjirit t maj t. Gruaja nuk e di nse duhet t vazhdoj ti japi gji fmij s. Cila sht kshilla m e mir q mund ti j epni ksaj nne? A) T ndrpres menj her gj irin fmijs, sepse mund t infektohet. B) T ndrpres mnj her gjirin fmijs, sepse ajo do t filloj t marr antibiotik. C) T vazhdoj t japi gji vetm me gj irin e djatht. D) T vazhdoj t japi gji duke prdorur t dy gj injt. E) T kalohet n ushqim artificial.

5 2. A) B) C) D) E)

Cila nga sindromet e mposhtme nuk manifestohet me shtat t shkurtr: Sindromi T urner A kondroplazia Sindrom Noonan Sindrom De Soto Hypokondroplazia

5 3. Cila nga pohimet e mposhtme sht i vrtet pr vaksinimin antipneu m okoksik: A) Kryhet mbas moshs 2 vje B) Nuk rekomandohet. C) Kryhet duke filluar nga moshe 3 muajshe me cdo tip vaksine. D) Kryhet duke filluar nga mosha 2-3 muajshe me vaksin t konj uguar. E) Kryhet vetm tek femrat.

5 4. Nj fmij e moshs 12 vje shfaqet me lodhje, dhimbje gryke, dobsi, temperature, hepatosplenome gali, limfoadenopati t gj eneralizuar diagnostikohet me nj mononukleoz t shkaktuar nga virusi Epstein- Barr. T gj itha komplikacionet e mposhtme mund t shfaqen, me prjashtim t: A) Anemi aplastike B) Encefaliti C) Mieliti transvers D) Ruptur e lienit E) Hepatit kronik aktiv.

5 5. rende A) B) C) D) E)

Nje femije 4 vje paraqet fecE)ne sasi te shtuar (abondante), me ere te dhe me permbajtje yndyrore. Diagnoza me e mundshme eshte: Semundja celiaki Giardiasis Fibroze kistike Deficit i laktazes H ipotiroidizem

5 6. Cila nga karakteristikat e meposhteme eshtE)tipike per konvulsionet febrile te thj eshta: A) Kriza te gjeneralizuara ne rast rritje te temperatures trupore ne femijet e cdo moshe. B) Kriza parciale pa humbje te ndergj egjes. C) Kriza te gj eneralizuara ne femijet me deficite neurologjike te evidentuara gj ate temperatures D) Kriza toniko-klonike te gj eneralizuara ne rastet e episodeve febrilE)ne femij et nga mosha 6 muaj ne 5 vj e E) Risk i larte per te zhvilluar epilepsi ne te ardhmen.

5 7. Te gj itha pohimet e meposhtemE)qe lidhen me infeksionin HBV gjate shtatezanise dhe periudhes neonatale jane te verteta, pervec:

A) Infeksioni paralindjes me HBV mund te coje ne te ardhmen ne zhvillimin e nje carcinome hepatoqelizore B) Rreth 70-90% e te porsalindurve te infektuar me HBV do te behen bartes kronik te HBsAg C) Skema e rekomanduar per vaksinimin dhe profilaksine passive per HBV te te porsalindurit ndryshon ne varesi te gj endjes serologjike e nenes. D) Vaksina kunder hepatitit B eshte universalisht e rekomanduar per te gjithe te porsalindurit ne terme E) Vaksina anti HBV eshte e kunder indikuar ne shtatezani,pavaresisht gj endjes serologj ike te nenes

5 8. dhenat 10^6? A) B) C) D) E)

Cila nga diagnozat eshte me e mundshme ne nje femije 3 vje me te hematologj ike te meposhteme:HB 11.5 g/dl,MCV 60, eritrocite 5,5x Semundje kronike Profil talasemik Mungese hekuri Mungese ne acid folik Mungese ne vit B12

5 9. Cili test eshte gjeresisht i perdorur per te monitoruar terapine me h e p arin e ? A) Numrimi i trombociteve B) Koha e trombines (TT) C) Dozimi i nje faktori specifik D) Koha e pjesshme e tromboplastines(aPTT) E) Koha e protrombines(PT)

6 0. Cila nga gjendrat limfatike eshte me shpesh e prekur ne limfomen Hodgkin? A) Limfonodujt aksilar B) Limfonodujt inguinal C) Limfonodujt laterocervikal D) Limfonodujt ilo-mediastinal E) Limfonodujt lumboaortal

6 1. Cila nga shenjat apo simptomat e meposhtmE)mund te mungoje ne nje te porsalindur me meningit? A) T e m perature B) Te vj ella C) Kriza convulsive D) Rigiditeti nukal

E)

PCR e rritur

6 2. mucus A) B) C) D) E)

Nje latant 7 muajsh qe paraqet te qara intermitente, te vjella dhe fece me dhe gj ak ka nevoje per: Koprokulture dhe solucione gluko-elektrolitike Rx direkte abdomeni dhe barium enema Trajtim me prokinetike Rx me contrast(barium test) Pozicion antirefluks

6 3. A) B) C) D) E)

Parvovirusi B19 shkakton: Semundj en VI Semundjen V Mononukleozen infektive Ekzantema subitum Semundja e IV

6 4. A) B) C) D) E)

Vlerat e hematokritit ne lindje jane : 25-35% 45-65% 70-85% 65-85% mbi 85%

6 5. A) B) C) D) E)

Hematuria asimptomatike ne moshat shkollore eshte prezente ne: 0-0.1% te femij eve 0,5-2% te femij eve 3-5 % te femij eve 5-10% te femijeve 8-15% te femijeve

6 6. Cila nga alternativat e meposhtme mund te j ete nderlikim i interventit kirurgjikal te hernies inguinale te lindur tek femij et? A) Atrofi testikulare jatrogj ene B) Gastroektazi jatrogjene C) Retension urinar D) Nje kist ne duktusin tireo-gloss E) Nje infeksion i rrugeve urinare

6 7. Per te arritur te ashtequajturen Imunitetin ne ko munitet ( herD)immunity ) eshte e nevoj shme te vaksinohet: A) 50% e popullates receptive B) 70% e popullates receptive C) >90% e popullates receptive D) >80 % e popullates receptive E) >60 % e popullates receptive

68 . Nga cila prej deficiteve ushqimore te meposhteme jane te kercenuar latantet e ushqyer vetem me qumesht dhie ? A) Deficit/mungese ne vit A B) Deficit ne vit B6 C) Deficit ne vit E D) Deficit ne folate E) Deficit ne thiamine

6 9. Ne diagnozen definitive te sepsis neonatal, cila nga alternativat e meposhteme eshte vendimtare per vendosjen e diagnozes? A) T e m peratura B) Simptomatologj ia e organit C) Er itrosed ime nti D) Pozitiviteti i hemokultures E) Pergjigjja klinike gjate antibiotiko-terapise

7 0. Cila nga semundj et e trashegueshme ka nje kuader klinik qe karakterizohet nga infeksione te shpeshta dhE)deficit te aftesisE)se neutrofileve per te shkaterruar bakteret e fagocituara? A) Aga maglobu lin emi a B) Semundja Wilson C) Fibroza kistike D) Semundja granulomatoze kronike E) Neutropenia kongenitale

7 1. Nje te porsalinduri i behet nje ekzaminim obj ektiv. Cila nga shenjat nuk sugj eron per ndryshime pathologjike? A) A krocianoze B) Hepar 2cm nen harkun brinj or C) Sufel sistolik I lehte 1-2/VI D) Frekuenca respiratore=70/min E) Zona te pigmentuarA)ne nivelin lumbosakral

7 2. Te gj itha pohimet rreth infeksionit te Hemofilus influenza tipi b jane te verteta pervec: A) I identifikueshem ne mikroskop pas ngj yrimit me Gra m B) Eshte pergjej ges per epiglotitin C) Mund te shkaktoje celulit D) Toksoidi tetanik sherben si proteine conj uguese per vaksinen. E) Godet kryesisht femij et mbi 7 vje

7 3. Te gj itha pohimet e meposhteme jane te verteta ,pervec: A) Fontanela e perparme mbyllet ne vitin e pare te jetes B) Fontanela e perparme (anteriore) eshte e gjere ne rastet me h i p o tir o i d iz e m C) Fontanela posteriorembyllet ne moshen 3mujore D) Fontanela e perparme mbyllet me vonese ne rakitizem E) Fontanela e perparme mbyllet me shpejt ne hipertiroidizem

7 4. Nje femije 15 muajsh paraqet kolle paroksizmale e shoqeruar me te vjella. Trajtimi me cilen nga medikamentet e meposhteme mund te jape rezultatet terapeutike me te mira? A) A m p ic ili n e B) Cefaclor C) K or tikos teroid D) Eritro micine E) R i m fa picin e

7 5. Nje femije 10 vj e ka kaluar nje infeksion te rrugeve te siperme respiratore para 5 ditesh,eshte trajtuar me aspirine si antipiretik dhe gj endja dukej e permiresuar. Tani shtrohet ne urgjence me temperature,te vj ella profuze dhe letargji. Ne ekzaminimin objektiv vihet re hepatosplenomegali e lehte. Ekzamini met laboratorike tregojne per nje rritje te bilirubines totale, te transaminazave dhe amonemise. Cila nga opsionet e meposhteme perben diagnozen me te mundshme? A) Hepatit A (HAV) B) Hepatit I shkaktuar nga barnat C) Sindromi Reye D) Mononukleoza infective E) Sindromi Gilbert

7 6. Te gj ithe shenjat dhe simptomat e meposhteme j ane tregues te dehidrimit akut ne femije ,pervec?

A) B) C) D) E)

Tharje e mukozave dhe lekures Oliguri Renie ne peshe >10% H ipertension Somnolence, asteni

7 7. Cilat nga psindromat e meposhtme eshte shkaku kryesor i insuficiences respiratore te femijet obeze ? A) P neu m otoraksi B) Refluksi gastroezofageal. C) Insuficienca cardiake congjestive. D) Astma E) Apnea e gj umit

7 8. Cilat nga etapat e meposhteme te zhvillimit psikomotor karakterizojne nje femije 10 muaj sh? A) Te ecurit B) Te folurit C) Te shkarraviturit me nje laps D) Te ulet vetem dhe te ngrihet ne kembe duke e ndihmuar E) Te ndertoje nje kulle me 2-3 kuba

7 9. hasur A) B) C) D) E)

Cila nga anomalite e meposhteme kardiovaskulare eshte me shpesh e ne sindromin Williams? Difekt ne septumin atrio-ventrikular Prolaps I valvules mitrale Stenoze aortale supravalvulare Koarktacion aorte Stenoze e a.pulmonare

8 0. Ne traumat e heparit, cila nga te dhenat e meposhtme diagnostike mund te perdoret? A) Cistografia B) Urografia C) Ekografia abdominale D) TAC cerebrale E) Klizem me barium.

8 1. Te gjitha pohimet e meposhteme shkojne me diagnozen e sindromit Turner ne nje paciente adoleshente,pervec?

A) B) C) D) E)

Kariotipi 45,X Stature e shkurter Amenorre primare LH e rritur dhe FSH e ulur Koarktacion I aortes

8 2. Nje adoleshent ve re nje masE)ne forme qese jo te dhimbshme ne anen e majte te skrotumit. Cila nga diagnozat e meposhtme eshte me e mundshme? A) Hidrocele B) V ar ikocel e C) Epidid i m it D) Torsion I testikulit E) Torsion I appendix testis (hydatid Morgagni)

8 3. Cili eshte standarti I arte , per diagnostikimin se semundjes se Hir sc hprung? A) Ph-Metria B) Klizma opake C) Biopsia rektale D) Echografi abdominale E) Manometria anorektale.

8 4. Nje nene sjell femijen 4 vjear tek pediatri sepse ka hemorragji nga hundet cdo dite, nje gjakederdhje e vazhdueshme ne sasi te paket nga nares dexter.Cila nga mundesite e meposhtme eshte shkaku me i shpeshte i epistaksit ne popullaten pediatrike? A) Rinit alergj ik B) Zakoni I te futurit te gishtave ne hunde C) Semundja Von Willembrand D) Purpura trombocitopenike idiopatike E) Angiofibrome nasale

8 5. Parandalimi i transmetimit vertikal tek femijet me HIV realizohet me ane te: A) Lindj es me rruge natyrale (via vaginale) B) Te ushqyerit me gj i C) Marrja e AZT ne shtatezani D) Te gjitha pergj igj et e dhena E) Asnje nga pergj igjet e dhena

8 6. Nena e nje latanti 2 muajsh eshte e shqetesuar nga nje artikull ne gazete qe flet per SIDS (Sindromi I vdekj es se papritur) dhe pyet cilet jane faktoret e riskut per kete patologji.Te gj itha indikimet qe i jep nenes jane te verteta ,pervec? A) Hipertermia ambientale B) Ushqyerja artificial C) Duhanpirja e nenes ne shtatezani D) Anamneze familj are positive per raste te meparshme me S IDS E) Qendrim permbys ose anash jo i sigurte gjate nates

8 7. A) B) C) D) E)

Cili eshte shkaku me I shpeshte I hipoglicemise ne nje femije<1vj e? H iperinsulinizem Hipopitu ita ri zem Hipoglicemia ketotike Deficiti enzimatik I neoglukogenezes M alabsorbim i

8 8. Te gjitha pohimet e meposhtme mbi renien ne peshe ne ditet e para te jetes jane korrekte ,pervec? A) Mund te jete deri ne 15% ne prematuret. B) Vjen nga humbja e shprehur e mekoniumit dhe lengj eve C) Vjen nga mungesat relative kalorike ne 2-3 ditet e para D) Pesha e lindjes duhet te rekuperohet brenda dites se 5 te j etes E) Konsiderohet fiziologj ike nje humbje rreth 10% te peshes se lindjes

8 9. Diagnoza e semundjes Werding-Hoff man eshte shume e mundshme ne nje femije me hipotoni te rende qe manifestohet me cilen prej shenjave te m eposhteme? A) Reflekset tendinoze te thella normale B) K on vulsi one C) Fashikulacion (dridhje) e gj uhes D) Temperatura te herepashereshme E) Atrofi e nervit optik

9 0. Gj aku komplet i nje pacienti tregon ulje te vellimit korpuskular dhe ulje te numrit te retikulociteve. Cila nga diagnozat e meposhteme eshte me e m undshme? A) Talasemia major B) Anemia ferriprive. C) Anemia ne semundjet kronike D) Profil talasemik E) Anemia nga deficit I acidit folik

9 1. Nje femije 3vj e paraqet temperature 39 C ,rigiditet nukal prej 3 ditesh.Ne ekzaminimin objektiv vihet re hiperemi e farinksit, shtim te konsistences se limfonodujve cervical anterior dhe posterior si dhe rritje te rezistences gjate fleksionit te qafes. Masa me e shpejte qe duhet ndermarre eshte: A) Punksioni lumbar B) Test per anticorpet heterofile C) Tampon faringeal dhe peniciline per 7 dite nga goj a D) Tampon faringeal dhe peniciline per 10 dite nga goj a E) Tampon faringeal, formule leukocitare dhe rivleresim pas 24 oresh

9 2. Te gjithe faktoret e meposhtem jane te perfshire ne patogenezen e bronkodisplazise, pervec: A) Infeksioni B) Pre m aturitet C) Ikteri i parakohshem. D) O ksigjenoterapia E) Ventilimi mekanik

9 3. Nje vaj ze 9 vj ee ishte ne gjendje shendetesore shume te mire para 10 oresh, kur filloi te shfaqte letargji, te vjella, te qara intermitente me retraktim te gj ymtyreve te poshtme ne nivel te barkut.Ne qendren ambulatore defekon fecE)me pak gjak dhe xhelatinoze. Diagnoza me e mundshme eshte: A) Stenoze pilori B) A pend isi ti s C) Infeksion I rrugeve urinare D) Invaginim intestinal E) Ulcer duodenale

9 4. A) B) C) D) E)

Vleresimi klinik per te percaktuar pikezimin Apgar behet ne : 1min,3min ,6min 1min,6min ,8min 1min,5min,10min 3min,6min,9min 3min,10min,20min

9 5. A) B)

Traumat abdominale mund te jene shkak i abdomenit akut: Po, gj ithmone Po, ndonjehere

C) D) E)

Vetem nese shoqerohen me trauma kraniale Vetem nese shoqerohen me trauma torakale Vetem ne prani te frakturave te gj ymtyreve te poshtem

9 6. A) B) C) D) E)

Cila nga pohimet e meposhtme mbi sindromin Berger eshte korrekte: Proliferimi mezangial eshte I dokumentuar ne mikroskop optic Depozitimet mezangiale jane kryesisht te perbera nga IgG Eshte me e shpeshte ne femra Nivelet serike te C3 jane te ulura Trajtimi nuk mund te behet me imunosupresore

9 7. Cila nga shenj at e meposhteme eshte e nevojshme per te bere nje diagnoze te ADHD (crregullim i hiperaktivitetit dhe deficit i vemendjes): A) Shfaqje para moshes 10 vje B) Paaftesi per te mesuar C) I m pulsivitet D) Anamneze per trauma ne lindj e E) Moter/vella me diagnoze te ADHD

9 8. Nje femije me fibroze kistike ka pneumoni te shkaktuar nga Staphylococcus aureus. Femija pa pritur ben nje detres respirator, zbehje te zhurmave respiratore ne anen e majte te toraksit dhe devijim te trakese ne te djathte. Cila nga pohimet e meposhteme eshte diagnoza me e mundshme: A) Emfizeme B) Versament pleuritik C) Rupture e nje pneumatocele nen trysni D) Sindrom I detressit respirator tek I rrituri E) Infarkt pulmonar.

9 9. A) B) C) D) E)

Cili eshte shkaku me I shpeshte ne hipotiroidizmin j uvenile? Mungesa ne j od Tiroiditi subakut Struma e thjeshte Tiroiditi Hashimoto Karcinoma tiroidiene

100. Cili nga barnat e meposhtem mund te shkaktojE)rash morbiliform ne mononukleozen infective? A) Meticilina B) Ampicilina

C) D) E)

Eritromicina Tetraciklina Difenilhydantoina

101. Nje femije 3 vje paraqet temperature,rash kalimtar, artrit, hepatosplenomegali, versament perikardiak, eritrosedimentacion te rritur, leukocitoze, neutrofili. Cila nga alternativat e meposhteme eshte me e m undshme? A) Ethe reumatizmale acute B) Lupus eritematoz sistemik(LES) C) Artrit rheumatoid juvenile D) Semundja Lyme E) Spondiliti ankilozant

102. Cila nga karakteristikat klinikE)te meposhteme eshte tregues i nje urgj ence neurologjike-neurokirurgjike ne nje femije me cefale akute? A) Intensiteti I dhimbjes B) Lloji I dhimbj es C) Prania e fotofobise D) Prania e shenjave neurologjike shoqeruese E) Zgj atja e dhimbjes

103. A) B) C) D) E)

Cila nga keto crregullime elektrolitike eshte tipike per fibrozen kistike ? Acidoza metabolike Alkaloza hipokloremike Alkaloza hiperkloremike Alkaloze e kompensuar Acidoze respiratore

104. A) B) C) D) E)

Cila nga gjendiet pasuese karakterizon sindromin TURNER? H ir s u tiz m i Vonese mendore Rrisk I larte per obezitet Incidence e larte per hipotiroidizem congenital Incidence e larte per semundje autozomi ke

105. Nje latant tre javesh prezanton mikrocefali, kalcifikime cerebrale ne radiografine e kokes dhe verberi. Cila nga patologjite e meposhtme eshte me e mundshme per te shpjeguar keto simptoma? A) Hemoragji subdurale bilaterale

B) C) D) E)

Agenezi cerebrale Infeksion nga citomegalovirusi E rit roblastoze Mikrocefali primitive

106. A) B) C) D) E)

Cila nga anomalite kongenitale te meposhtme haset me shpesh? Mie l omeningo ce le H ipospadia F oko melia Palatoskizi Deformime te kembeve

107. Cila nga pohimet e meposhtme duhet ndjekur se pari ne ekzaminimet e femijes me traume te kraniumit? A) Anamneze mbi rrethanat e traumes B) Inspeksion dhe palpacion te kokes C) Kontroll i ekuilibrit acido-bazi k D) Vlersimi i funksioneve jetesore E) Radiografi e kafkes

108. Per nje rritje normale ne gjashte muajt e pare te jetes, cila nga vlerat e meposhtme eshte me afer marrjes ditore te rekomanduar te proteinave? A) 0.2 g/kg B) 1.0 g/kg C) 2.0 g/kg D) 5.0 g/kg E) 10.0 g/kg

109. Cila nga manifestimet e meposhtme do te pritej te gj endej ne nje femije gj ashte muaj sh me nje defekt te madh te septumit ventricular? A) Cianoze B) Zemer e zmadhuar C) Zhurme e vazhdueshme D) Ulje te vaskularizimit pulmonar ne radiografi E) Predominim te hipertrofise se ventrikulit te djathte ne EKG.

110. Cila eshte neoplazia malinje me e shpeshte tek femij et me te vegjel se pesembedhjete vjee? A) Morbus HODGKIN B) Neuroblastoma

C) D) E)

Leukemia Retinoblastoma Hepatoblastoma

111. Nje bebe mashkull lindi ne javen e tridhjete e nente te shtatzanise me nje peshe trupore 4500g.Te gj ithapohimet e meposhtme mund te jene shkak , pervec: A) Sindomi i Beckwith wiedman (hipoglicemia nga hiperinsulinemia, makroglosia etj) B) Pirja e duhanit nga nena C) Diabet mellitus i nenes D) Hidrops fetal E) Predispozite gjenetike

112. A) B) C) D) E)

Shkaku me i shpeshte i hipotiroidizmit congenital tek femijet eshte: Diskinezia glandular Deficit hipotalamik I TRH Defekt ne hormonogj eneze Hipotiroidizem sekondar Mungesa e j odit

113. Te gjitha shkaqet e meposhtme j ane pohime korrekte per anemine megaloblastike pervec: A) Malabsorbim intestinal B) Rezeksion i gj ysmes distale te ileumit C) Dieta vegjetariane integrale D) Hemoragji gastrointestinale E) Perdorim i zgj atur i trimetoprimes

114. A) B) C) D) E)

Si vihet diagnoza e Semundjes se Celiakise? Dozim i antiorpeve anti-Gliadine Dozimi i antikorpeve anti-muskulature te lemuar Dozimi i antikorpeve anti-Transglutaminaze Me vleresimin e HLA Me biopsi duodenale

115. Cila nga kombinimet e meposhtme eshte me i pershtashem per per nje femije shtate vje i diagnostikuar prej nje viti me diabet melitus tipi pare ? A) Hipoglicemiant oral dhe diete hipokalorike B) Insulinoterapi me diete te balancuar normokalorike

C) D) E)

Insulinoterapi dhe permbajtje e larte fibrave ne diete Insulinoterapi dhe perj ashtim i sheqernave nga dieta Insulinoterapi ose antidiabetik oral dhe diete hiperproteinike

116. A) B) C) D) E)

Femij et me obezitet te thjeshte prezantohen me shpesh: Pubertet te pershpejtuar Moshe kockore te vonuar Shtat me i shkurter sesa i moshatareve Shtat me i larte se i moshatareve Asnje nga pergjigjet e mesiperme

117. A) B) C) D) E)

Ne cilat nga patologjite e meposhtme indikohet perdorimi i naloksonit: Sindrome bipolare D ep re si o n E neurezis Skizofrenia Abuzimi nga droga

118. Cila nga manifestimet e meposhtmE)karakterizon sindromin e X-te t h y e s h em A) Fraktura multiple te kockave te gjata B) Plage qe sherohen me veshitiresi C) Vonese mentale D) Testikuj te vegjel E) Infeksione bakteriale rekurente

119. A) B) C) D) E)

Terapia kirurgj kale ne hernien inguinale tek femija konsiston ne . Mbyllje te kanalit umbilikal Mbyllje te kanalit pertoneo-vaginal Mbyllje te unazes inguinale te jashtme Vendosje te patchit ne zonen ehernies Vendosje te patchit ne skrotum

120. Prania e cianozes ne lindje mund te jete pasoje e te gj itha shkaqeve te meposhtme pervec: A) Tetrades Fallot B) Transpozicionit te eneve te medha te gjakut C) Duktus arterioz hapur D) Atrezi trikuspidale E) Hipertension pulmonar persistem

121. A) B) C) D) E)

Ne hernien inguinale te inkarceruar mund te pasohet mE)nje: Nje gonadoektomi Nje gastroektomi Nje nefroektomi Nje splenektomi Nje vezikostomi

122. Trajtimi ketoacidozes diabetike perfshin te gjitha masat e meposhtme pervec: A) Dhenies se bikarbonatit nese ph eshte me i vogel se 7.15 B) Dhenies se insulines ne fazat e para me 0.1 unite per kg ne ore C) Infuzionit te glukozes 5% ne rruge intravenoze nese glicemia eshte me e vogel se 250 mg/dl D) Dhenies se parakohshme te kaliumit nese ka acidoze E) Dhenia e insulines sukutan nese femija ushqehet

123. A) B) C) D) E)

Diarreja akute te latantet duhet te trajtohet me: A ntibiotike Probiotike Prebiotike Rehidrim oral Rehidrim endovenoz

124. Nje femije 5 vje vjen pasi eshte vizituar per nje faringit akut dhe pas marrj es se tamponi faringeal per mbjellj E)traj tohet me amoksiciline. Ne harkun kohor 24-48 ore,ai zhvillon nje egzanteme dhe tamponi faringal rrezulton negative. Cila nga dg e meposhtme eshte me e mundshme? A) Skarlatine B) M ononukleoze C) Semundje e gj ashte D) Semundja Kawasaki E) Purpura Schonlein-Henoch

125. Cila nga semundj et e meposhtme eshte shkaku me i shpeshte i malabsorbimit te femijet e moshes shkollore? A) Divertikuli meckelit B) Semundja e crohnit C) Celiakia D) Fibroza kistike

E)

Koloni irritable

126. Nje femije 4 javesh paraqitet qe prej 10 ditesh me perkeqesim te te vj ellave te cilat jane jobiliare. Ka uri, eshte aktiv dhe ka renie ne peshe.Cila nga dg e meposhtme eshte me e mundshmja? A) Obstruksion i zorres se holle B) Stenoze hipertrofike e pilorit C) G as troenet rit D) Invaginacion E) Neoplazi cerebrale

127. Komplikacioni me i shpeshte i nje etheje reumatizmale te patrajtuar ose te trajtuar jo mj aftueshem eshte: A) Artropati kronike B) Insuficensa renale C) Vonese ne stature D) Uveitis E) Kardiopati kronike

128. Te gjitha pohimet E)meposhtme j ane te verteta ne lidhje me pneumonine nga Mycoplazma pneuomonie, pervec: A) Eshte me e shpeshte ne moshen parashkollore B) Rezultat normal i rruazave te bardha te gjakut qarkulluese C) Prezence e aglutinit ne te ftohte te serumit D) Radiografi e toraksit me infiltracion te 1 ose 2 lobeve se bashku E) Manifestohet me temperature,dhimbje fyti dhe kolle te thate.

129. Ne 10 ml urine te mbledhura ne kushte te fresketa, te egzaminuara ne terren ushqyes te mire, hematuria mikroskopike perkufizohet si prezenca e te pakten: A) 0-1 rruaza te kuqe per fushe B) 3-5 rruaza te kuqe per fushe C) 3-10 rruaza te kuqe per fushe D) >10 rruaza te kuqe per fushe E) Asnje nga pergjigjet

130. Nje djale 14 vj e qe luan shpesh basketboll, sillet ne pediatri per calim dhe dhimbje ne gju. Dhimbja forcohet nga vrapi , kercimet, ngj itja e shkalleve. Tuberoziteti tibial eshte i enjtur por pa grumbullim te likidit sinovial apo

prezence te trashj es se kavitetit te gj urit. Cila nga dg e meposhtme eshte me e m undshme? A) Semundja e Pertes B) Renie e epifizes se kokes se femurit C) Semundja Osgood Schlatter D) Artriti septic i gjurit E) Kondromalacia e kupes se gjurit

131. Nje femije 4 vje ka temperature 40 grade celcius prej 4 ditesh. Egzaminimi objektiv paraqet konjuktivit, rash eritematoz, adenopati cervikale dhe edeme te duarve dhe kembeve. Egzaminimet laboratorike tregojne leukocitoze me predominim te leukociteve, trombocite ne norme, ERS i rritur . Cila eshte diagnoza me e mundshme? A) Skarlatina B) Ethe reumatizmale C) Periarterit nodoz D) Lupus eritematoz i diseminuar E) Sindromi Kawasaki

132. Nje djale 8 vj e me zona multiple alopecie. Ne ekzaminim me llampen e Wood-it shihet nje fluoroshence blu ne te verdhe. Cili nga patogj enet eshte me i m undshem? A) Epidermophyton floccosum B) Mikrosporom canis C) Candida albicans D) Tricophyton tonsurans E) A spergil lu s

133. A) B) C) D) E)

Si parndalohen kompikacionet reuamatizmalE)te Skarlatines? Me izolimin e femijeve te infektuar Me penicillin per 10 dite Me ampicillin per 6 dite Me vaksinimin masiv Asnje nga te mesipermet

134. A) B) C) D) E)

Femija 1 vj e peshon: 9-11 kg 13-14 kg 16-17 kg 19-20 kg 8-9 kg

135. Nje latant qe paraqitet ne gj endje te mire shendetsore, kohet e fundit ka vuaj tur nga nje infeksion nga rotavirusi. Pas rehidrimit oral, vazhdon riushqyerjen me formulen standart qe perdorte para fillimit te diarese. Sa here qe merr qumesht prezanton diare te uj shme. Diagnoza me e mundshme eshte A) Sindromi i zorres se shkurter B) Insuficienca pankreatike C) Deficit primari disaharidases D) Deficit sekondar i laktazes E) Alergji ndaj proteinave te qumeshtit te lopes

136. Nje femije me tumor te fosa krani posterior, ankohet per dyfishim te pamjes. Egzaminimi i levizjeve te syve tregon strabizmus. Paraliza e cilit muskul eshte me e mundshme? A) Paralize e m oblique superor B) Paralize e m rectus lateralis C) Paralize e m oblique inferior D) Paraliza e m rectus medialis E) Paralize e m rectus superior

137. A) B) C) D) E)

Zgj idh agj entin me te shpeshte pergj egj es per sepsisin neonatal: Streptokoku b-hemolitik I gr A Streptokoku b-hemolitik gr-B Stafilokoku aureus H. influence tipi B N.m eningitidis

138. Te gjitha teknikat e meposhtme perdoren per ekzaminimin e RGE pervec. A) Kalueshmerise me Ba B) Manometria ezofageale C) Ph metria ezofageale D) Shintigrafia me technecium radioaktiov E) Testi i urese ( Breath test )

139. A) B) C) D) E)

Cili eshte shkaku , me i shpeshte i hematurise mikroskopike? Sindromi nefritik Tu renal Infeksion i rrugeve urinare Nje koagulopati Traume renale

140. Cili nga defektet e traktit gastrintestnal shoqerohet me shpesh me malformacione kardiake? A) Omfalocele B) Volvulus congenital C) Semundja e Hirshprungut D) G a str o s k iz a E) Stenoza e pilorit

141. A) B) C) D) E)

Divertikuli I Meckelit eshte I vendosur. Ne rectum Ne ileum 20-40 cm nga valvula ileo cekale Ne ileum 120cm nga valvula ileo cekale Ne kolon Ne ezofag

142. Vleresimi I rritjes dhe zhvillimit eshte themelor tek femija .Te gjitha pohimet e meposhtme j ane te verteta, pervec. A) Cirkumferenca e kraniumit ne 2 vje ka arritur 90% te circumferences se te rriturit. B) Ne nje vje femija zakonisht trefishon peshen e lindjes C) Nje femije ne 7 muaj mund te qendroje ulur D) Permasat e peshes dhe gjatesise tek neonati jane te lidhura me potencialin gj enetik E) Ne 3 vj e femija mashkull ka arritur e gjatesise se moshes se rritur

143. kohes A) B) C) D) E)

Zgj idh semundj en me te pershtatshme e cila manifestohet me zgj atje te se hemorragjise dhe rritje te kohes se tromboplastines parciale. Purpura trombocitopenike idiopatike Semundja e von Willenbrandit Hemofilia klasike Mungesa e faktoreve II,VII, IX,X Mungesa e faktoreve III , VI,V III

144. Hidrocela ne dalje te kanalit peritoneo-vaginal nese nuk zgjidhet spontanisht duhet te operohet. A) Ne pubertet B) 15 ditet e para te j etes C) Rreth 12 muaj ve D) Pas 5 vjetesh

E)

Pas pubertetit

145. Cili nga agj entet e meposhtem eshte shkaku me i shpeshte i otitit te mesem pas periudhes neonatale? A) S. pneu moniae B) S.aureus C) Mycoplazma pnumonie D) E.coli E) Streptococu i gr A

146. Cila nga pohimet e meposhtme eshte shkaku me i shpeshte i hidronefrozes pielike tek femij a. A) Stenoze ne nivel te bashkimit pieloureterale B) anomali vazale C) K alkuloze D) Refluks veziko ureteral E) Valvules se uretres posteriore

147. Sa eshte treguesi i Apgarit tek nje neonat me FC=110/min, frymemarrj e te ngadalte dhe crregullt, te qara me gj alleri, ngjyra e lekures roze, ndersa ekstremitetet blu. A) 3 B) 4 C) 6 D) 8 E) 10

148. A) B) C) D) E)

Cila nga semundj et e meposhtme kerkon izolimin e femijes . Varicele Ekzanteme kritike Pozitivitet per Hiv Purpura Schonlein Henoch Mononukleoze infektive

149. Cila nga pohimet e meposhtme eshte e vertete per artritin rheumatoid juvenil? A) B) C) Forma me e shpeshte eshte ajo pauciartikulare e tipit te dyte. Semundje e Still eshte me e shpeshte te femrat Iridocikliti gjendet me shpesh ne formen poliartikulare.

D) E)

Forma pauciartikulare e tipit te dyte eshte elidhur me pranine e genotipi HLAB27 Sacro- ileiti gj endet me shpesh ne formen poliartikulare me faktoR rheumatoid pozitiv.

150. Te gj itha pohimet me poshte per sa i perket Luksacionit coxo-femoral kongenital jane te verteta, pervec: A) Si faktoret gj enetike dhe ata jo gj enetikE)jane te rendesishem ne patogenezen e semundjes B) Ne ekzaminimin objektiv mund te gjendet disproporcion ne gj atesine e gj ymtyreve dhe asimetri e plikes kutane gluteale. C) Nje shenje objektive mund te jete ajo e Ortolanit, qe eshte vleresimi i nje kliku i palpuar ne rotacionin ekstern dhe abduksionin e kycit. D) Diagnoza zakonisht mund te konfirmohet me ekzaminim ultrasonografik E) Ne shumicen e rasteve eshte e nevojshme korigjimi me nderhyrj e kirurgjikale.

151. Nje femije 3 vj e, gelltit te pakten 40 kompresa Fe-sulfat 300mg secila qe permbaj ne 60 mg Fe elementar. Cila nga shfaqjet e meposhtme eshteme pak e mundur? A) Vomitus dhe diarre B) Acidoza C) Kolapsi cirkulator D) Insuficienca hepatike E) Insuficienca renale

152. Cfare shenje ose simptome duhet kerkuar tek nje latant ne te cilin dyshohet per refluks gastro-ezofageal patologjik? A) Krizat e apnese B) Anemia ferodeficitare C) Hemoragjia gastro-intestinale D) Vonesa ne rritje dhe anoreksia E) Te gjitha

153. Nje femije 15 muaj ka anemi mikrocitare nga mun gesa e hekurit, rezistente ndaj trajtimit me hekur. Fecet e tij jane ne menyre te perseritur pozitive per gjak okult. Prinderit e sjellin femij en ne urgjence pasi kane vene re gjak nga rektumi te pashoqeruar me dhimbje. Cila eshte diagnoza me e m undshme? A) B) Invaginacioni Ulcera duodenale

C) D) E)

Volvulus Divertikuli Meckel Koliti ulceroz

154. A) B) C) D) E)

Ne traumat e shpretkes, cila nga metodat diagnostike mund te perdoret: Cistografia Urografia Ekografia abdominale TAC)i trurit Klizma me barium

155. Cila nga shenjat eshte shume e nevojshme ne diferencimin e M. Crohn nga Koliti ulceroz? A) B) C) D) E) Hemorragjia rektale Abseset e kripteve Perfshirja e ileumit Eritema nodoze Kolangiti

156. Nje djale 12 vje, paraqet veshtiresi progresive ne shkolle, me ikter kronik qe zgj at prej 6 muajsh. Te dhena laboratorike tregoj ne hipertransaminazemi modeste, anemi hemolitike, glukozuri. Diagnoza me e mundshme eshte: A) B) C) D) E) Hepatit kronik B Sindroma Reye Semundja Wilson G alaktozemia Glikogj enozat

157. Cilat prej infeksioneve , ka periudhen e inkubacionit me te gj ate te foshnjet? A) Rubeola B) Parotiti C) Fruthi D) Eritema infeksioze (semundja e 5-te) E) Varicela

158. Per te nxitur maturimin e pulmoneve ne rastin e lindj eve premature perdoren: A) K ortizoniket B) Antibi otiket C) Surfaktant ekzogj en D) Hormonet e tiroides E) Growth factor (GH)

159. A) B) C) D) E)

Limfadenopatia retro-nu kale eshte karakteristike e: Rubeola Semundja e gervishtj es se maces M ononukleoza Adeniti tuberkular Infeksion nga HIV

160. Cila nga terapite do te keshillohej per nje femije lindur ne term me nivel bilirubine mbas 36 oresh, prej 24mg/dl A) H idratim i B) Infuzion i albumines C) Fo totera pi a D) Terapia me barbiturate E) E ksan gu inotransfuz ion

161. A) B) C) D) E)

Cili nga tumoret takohet me shpesh ne testikulin kriptorkidik? Teratoma Seminoma Embrionalkarcinoma Tumor i qelizave Sertoli Tumor i qelizave Leydig

162. Aftesia per te kopjuar forma zhvillohet sipas nje rradhe te caktuar. Cila nga keto eshte rradha e duhur? A) Kopjon 1 kuadrat,1 kryq,1 rreth B) Kopj on 1kuadrat,1 rreth,1 kryq C) Kopj on 1kryq,1 rreth,1 kuadrat D) Kopjon 1 rreth,1kuadrat,1kryq E) Kopjon 1 rreth,1 kryq,1 kuadrat

163. A)

Hidrocela eshte: Nje grumbullim likidi seroz ne vaginen propria te testikulit

B) C) D) E)

Nje dilatacion i ventrikuj ve cerebral Nje grumbullim pusi ne nivelin skrotal Nje shfaqje ekzanteme Nje semundje infektive

164. A) B) C) D) E)

Mundesite e infektimit nga Toxoplasma per nj eriun j ane: Kontakti me macet Ngrenja e perimeve jo te lara mire Ngrenja e mishit te pa gatuar mire ose krudo Kontakti me delet Te gj itha te mesipermet

165. Zmadhimi i gj endrave limfatike nE)semundjen e gervishtj es se maces regredon menjehere mbas: A) 1-2 muaj B) 1-2jave C) 1-2 dite D) 6 muaj E) Te gjitha me siper jane te mundshme

166. A) B) C) D) E)

Cili nga tumoret solide eshte me i shpeshti tek foshnjet? Tumori Wilms Neuroblastoma Rabdomiosarkoma Sarkoma Ewing Teratoma

167. Nena e Anes nje vaj ze 3 vj eare, ju paraqet ekzaminimin rutine te gj akut komplet te vaj zes, si me poshte: Hb 12; MCV 75; ERS10; RB 10900; N 37%,L 48%, PLT 200000. Cfare mendoni? A) Ka nje infeksion probabilisht viral B) Ka nje anemi mikrocitare C) Ka nje leucemi limfoblastike akute (LLA) D) Ka parametra hematologjike brenda kufijve te normes per moshen E) Ka nje infeksion probabel bakterial

168. Te gjitha pohimet e meposhtme jane te verteta per sa i perket ekza minimit objektiv te neonatit, me perjashtim te: A) Frekuenca kardiake mesatare e neonatit eshte 100-110/min B) Retraksioni i toraksit anterior ne inspirim eshte i zakonshme

C) D) E)

50% e neonateve japin nje imazh iketrik ne pamje Refleksi i kapjes eshte normalisht prezent tek neonati Shenja e Babinski shpesh mund te evidentohet gjate vizites.

169. A) B) C) D) E)

Puberteti prekoks shoqerohet me: Rritje te nivelit te gonadotropinave Ulje te nivelit te gonadotropinave Vonese te moshes kockore Ritme te ulta rritje Te gj itha me siper

170. Te gj itha pohimet jane te vertetA)persa i perket hiperbilirubinemise neonatale, pervec: A) Te neonati ne term dhe te shendetshem mund te perdoret si fototerapia ashtu dhe eksanguinotransfuzioni B) Bilirubina eshte potencialisht toksike per SNQ C) Ka nje korelacion te drejte midis nivelit serik te bilirubines dhe ikterit nuklear. D) Rreth 1,1-2% te femij eve te ushqyer me gji zhvilloj ne ikter nga qumeshti i nenes E) Fototerapia mund te reduktoje nivelet e bilirubines serike te femijet me ikter jo hemolitik

171. Cili nga medikamentet eshte pergj egj es per ritjen e nivelit serik te Teofilines? A) Penicillina B) Eritromicina C) Cefalotina D) Sulfisoxazol E) Gentamicina

172. A) B) C) D) E)

Interventi per korigjimin e hernies inguinale duhet te kryhet: Kur femija arrin peshen trupore 5 kg Kur femija arin peshen trupore 10 kg Kur femija mbush moshen 6 muaj Ne varesi te rastit Si urgj ence ne momentin e paraqitjes

173. A)

Sa eshte cirkumferenca kraniale e neonatit? 60 cm

B) C) D) E)

55 cm 40 cm 30 cm 35 cm

174. Cila nga vitaminat keshillohet te shtohet ne muajt e pare te femija qe ushqehet me gji? A) Vitamina A B) Vitamina D C) Vitamina E D) Vitamina C E) Te gjitha

175. A) B) C) D) E)

Cili organ demtohet vecanerisht ne fenilketonuri? Veshka Mushkeria Zemra Melcia Truri

176. A) B) C) D) E)

Te cilat gra aplikohet immunoglobulina anti-D pas lindj es ? Te grate grup Rh+ me femij en RhTe grate grup Rh- me femij en Rh+ Te grate grup Rh- me femij en RhTe grate grup Rh+ me femij en Rh+ Te te gj itha grate mbas lindjes se pare

177. Pasojat nga infeksioni me T oxoplasma mund te jene A) Hidrocefalia B) K ar diopatia C) H epatopatia D) S hurdhim E) Keqformime te artikulacioneve

178. Ne S. Turner cila nga shenjat ose simptomat mungon? A) Gj atesia e shkurter B) Steriliteti C) Prapambetje mendore D) Pterigium koli E) Coxa vara

179. A) B) C) D) E)

Cila eshte gjatesia e femijes ne lindje 40 cm 55 cm 50 cm 60 cm 70 cm

180. A) B) C) D) E)

Cfare eshte semundja e Celiakise Intolerance permanente ndaj Glutenit Intolerance tranzitore ndaj Glutenit Hipersensibilitet ndaj Glutenit Intolerance sekondare ndaj Glutenit Intolerance permanente ndaj orizit

181. A) B) C) D) E)

Ne cilen moshe mund te perdoret vaksina anti-Papillomavirus? Ne moshen 6 vje Ne moshen 12 vje Ne moshen 8 vje Ne lindje Ne moshen 10 vj e

182. Cili serotip i meningokokut nuk eshte ende prezent ne vaksinat qe ndodhen ne treg? A) C B) A C) W-135 D) Y E) B

183. A) B) C) D) E)

Nje femije normal i shendetshem ne moshen 1 vj e peshon: 8-9 kg 7-8 kg 10-11 kg 11-12 kg 12-13 kg

184.

Kur ikteri konsiderohet fiziologjik?

A) B) C) D) E)

Kur shfaqet brenda 24 h Kur shfaqet mbas 24 h Kur bilirubina kalon vleren 20mg/dl Kur dominon bilirubina direkte Asnjera prej tyre

185. A) B) C) D) E)

Ne depistimin e hipotiroidizmit congenital cfare dozojme? GH T3 e T 4 TSH dhe/ose T4 FSH e LH 17OH Progesteron

186. Cila mund te jete nje nga pasojat e nje infeksioni invaziv nga Streptococcus Pneumoniae? A) Meningiti B) Encefaliti C) Karditi D) G lo merulonefriti E) Artriti

187. A) B) C) D) E)

Cili eshte shkaku i semundj es hemoragjike te te porsalindurit? Deficenca e vit D Deficenca e vit E Deficenca e vit K Deficenca e vit C Deficenca e Acidit folik

188. Kur nje neonat percaktohet si me peshe sheme te ulet ne lindje (VLBW)? A) < 2500gr B) <2000gr C) <1500gr D) <1000gr E) <500gr

189. Nje femije 5 vje vuan per her te dyte nga meningiti meningokoksik. Cili eshte ekzaminimi me i mire qe duhet bere? A) Percaktimi i nivelit te imunoglobulinave B) Ekzaminimi i subpopullatave limfocitareT.

C) D) E)

CH50 Matja sasiore e testit te NBT ( Clorur Nitroblu tetrazolit) Testet e hipersensibilitetit te vonuar kutan

190. Nje femije 8 vje shtrohet per shfaqj en e konvulsioneve. Cila nga shfaqjet e meposhteme do te mbeshteste me shume nje diagnoze te konvulsioneve parciale komplekse (psikomotore) A) Statusi mental normal,koshjent dhe pergj igj et gj ate konvulsioneve B) Nje faze e shpejte toniko-klonike C) A utomatizmat D) Modeli EEG karakterizuar nga trase maje- vale,3/sec E) Statusi mental normal,koshj ent dhe qe pergj igjet mbas konvulsioneve

191. Ne nje neonat diagnostikohet Aniridia. Cili nga egzaminimet e meposhtme duhet te kryhet tek ky femije? A) Radiografia torakale B) Matja e alfa fetoproteines C) Percaktimi i funksionit renal D) Egzaminimi i testikujve E) ECHO renale

192. Nje femije 2 vje eshte e plogesht dhe e zbehte. Ju kryeni nje gjak komplet dhe zbuloni se femija ka anemi megaloblastike. Cila e dhene ne anamneze shpjegon kete situate? A) Ha vetem produkte te ritura organikisht B) Pi vetem qumesht dhie C) Ka mare fototerapi ne periudhen neonatale D) Ka mare shume antibiotike per trajtimin e otitis media E) Eshte femije e lindur nga nje nene diabetike

193. Nje femije 5 vj e me Artrit Juvenil Idiopatik Pauciartikular ka ANA+(antikorpet anti-nukleare). Cila nga me poshte do te ishte gjetja me e mundeshme per kte rast? A) Perikarditi B) Nefriti C) Uveiti D) Splenomegalia E) Linfadenopatia

194. Nje adoleshente 16 vj e ka obezitet te shprehur. Cila nga situatat me poshte perfaqeson shkakun me te shpeshte te insuficiences respiratore ne adoleshentet obeze? A) Pneumotoraksi B) Refluksi gastro-ezofageal C) Insuficienca kardiake kongj estive D) Astma E) Apnea e gj umit

195. Nje femije 18 muajsh ka anemi mikrocitare. Cila nga te dhenat anamnestike shpjegon me mire shkakun e saj ? A) Pica B) Mungesa ne diete e perimeve te fresketa C) Futja e pamj aftueshme ne diete e lengjeve te frutave D) Marrja e sasive te medhe te vit. C E) Marrja e sasive te medha te qumeshtit te lopes te paholluar.

196. Nje neonat 2-j avesh paraqet apne. Femija kishte lindur ne term pas nj e shtatzanie te pakomplikuar. Nena e femijes kishte rinorhea dhe kolle,te cilat i kishin filluar para 3 javesh dhe tani ka nje kolle te shpeshte e persistente me te vj ella pas akseseve te kolles. Cili nga mj ekimet e meposhtme duhet tI fillohet femij es? A) C eftriaxone B) A m oxicilline C) A zitromicine D) V anco mic in e E) Am antadine

197. Nje foshnje 14-muajsh ka nje histori me diarre intermitente per 4 muaj. Shpesh ka defekime shkarkuesE)qe permbaj ne pjeseza ushqimore.Rritet mire dhe ekzaminimi objektiv eshte normal. Cili eshte hapi qe duhet ndjekur me tej ? A) Te qetesohen prinderit B) K oproku ltur e C) Vleresim cilesor i imunoglobulinave serike D) Vleresim cilesor I yndyrnave ne fece E) Te pershkruhen antidiarreike orale

198. Nje vaj ze 12 muaj she eshte diagnostikuar me eritroblastopeni tranzitore infantile. Cili nga pohimet eshte i sakte? A) Trajtimi me kortikosteroide zakonisht eshte i perzgj edhur B) Mund te jete i nevoj shem transfuzion eritrocitar C) Zakonisht eshte e pranishme hepatosplenomegalia

D) E)

Zakonisht nuk ka sherim spontan Semundja lidhet me infeksionin nga parvovirus

199. Ne nje latant me hipotoni dyshohet diagnoza Werdning-Hoffman.Cila nga shenjat e meposhtme mbeshtet diagnozen? A) Reflekse te thella tendinoze normale B) K on vulsi one C) Fashikulacione te gj uhes D) Episode febrile rekurente E) Atrofi e nervit optik

200. Nje vaj ze 12 vj e ka dobesi muskulare gj ate javes se fundit. Ka nje rash eritematoz ne fytyre, krahe, kofshe dhe nje rash retikular ne palpebrat superiore. Cili do te ishte ekzaminimi me i mire per tu kryer? A) Faktor reumatoid B) Er itrosed ime nti C) Analize urine D) Matja e kreatinine kinazes serike E) Antikorpet antinukleare

201. Cilen anomali hematologj ike duhet te dyshosh te nje neonat me mungese bilaterale te radiusit? A) Trombocitopenia B) A nemia C) N eu trop en ia D) Pancitopenia E) L i m fo p e n i a

202. Nje djale 14 muaj paraqet ekzeme grave,nje anamneze me otit rekurent dhe infeksion te gjinj ve, trombocitopeni.Cila eshte menyra e trashegimise se kesaj patologj ie? A) Lidhje me kromozomin X B) Autozomike dominante C) Autozomike recesive D) Mutacion i rastesishem E) Multifaktoriale

203. Nje neonat u diagnostikua me hipoparatiroidizem kongenital.Cila nga me poshte ka mundesi te gjendet me shpesh gjate vleresimit te tij? A) Mikrocefalia

B) C) D) E)

Hiponatremia Hiperkalemia G ogesima Kan di doz e

204. Nje vaj ze 16 muaj nga Nigeria paraqet nje fillim akut te ataksise se trungut me te vj ella, nistagmus dhe disartri. Nuk ka temperature,as rigiditet nukal. Cila nga te dhenat e anamnezes do te te ndihmonte per te gjetur shkakun e ketyre simptomave? A) Nivelet e larta te plumbit ne moshen 1 vje B) Konvulsione febrile ne moshen 13 muaj C) Faringodinia me vezikula ne palatum 3 jave me pare D) Semundje febrile me erupsione kutane 2 muaj me pare E) Vaksina e pare MMR (fruth, parotit, rubeole) 1 muaj me pare

205. Nje djale 12 vje vuan nga artrit migrant, me artikulacione te skuqur,te nxehte, te enjtur. Paraqet serologji per infeksion recent me streptokok gr A. Ne keto kushte artriti karakterizohet: A) Zakonisht jo i dhimbshem B) Sherohet pa shkaktuar deformime C) Shfaqet pasi zhduket temperatura D) Shfaqet vetem ne pacientet qe kane edhe kardit E) Perfshin ne te njejten menyre artikulacionet e medha dhe te vogla

206. Nje 2 vj ear me origj ine afrikane paraqitet me enjtje te dhimbshme te duarve dhe kembeve.Te dhenat laboratorike tregoj ne per hemoglobine 9 g/dl,leukocite 11500/mm3,trombocite 250000/mm3.Cili nga ekzaminimet e meposhtme do mbeshtese diagnozen tende? A) Ekzaminim radiologjik i plote B) VDRL C) M ielogra m e D) Elektroforeze e hemoglobines E) Kalcemia

207. Nje 4 vj ear nuk ka zhvillim fizik normal dhe ka te dhena per insuficence pankreatike ekzokrine.Cili eshte shkaku me i mundshem i ketyre shfaqjeve klinike? A) Pankreatit akut B) Atrezi biliare C) Sindroma e Swachman-Diamond D) Mungesa kongenitale e pankreasit E) Fibroza kistike

208. Ne nje vaj ze 2 vje dyshohet per tumor cerebral. Ajo ka ataksi, ligj erate te ruajtur, te vjella heret ne mengj es. Cili nga pohimet rreth tumoreve cerebrale te femijet eshte i sakte? A) Pj esa me e madhe e tyre lokalizohen ne linea mediana dhe/ose poshtE) tentoriumit. B) Tumoret cerebrale paraqesin nje forme te rralle te neoplazive te femijet. C) Hipertensioni endokranial eshte i rralle nder shenj at e shfaqjes se tumorit. D) Konvulsionet paraqesin shenj en me te shpeshte te shfaqjes se tumorit ne pj esen me te madhe te rasteve. E) Pj esa me e madhe shfaqen ne vitin e pare te jetes.

209. Nje 12 vj eare ka ikter,tremor progresiv dhe labilitet emocional.Cilen nga me poshte mund te gj esh me shpesh gj ate ekzaminimit objektiv? A) Cirkumferenca e kokes eshte mbi percentilin 95 B) Ngj yrim i kornese ne te verdhe te erret C) Shurdhim konduktor bilateral D) Limfadenopati e gjeneralizuar E) Floke ne fovea sakrale

210. Nje neonat 38 javesh lindi nga nje nene me diabet gestacional. Pesha e lindjes eshte 4255g. Cfare pritet te gj endet me shpesh te ky neonat? A) Difekt i tubit neural B) Sindroma e kolonit te shkurter te majte. C) K ardiomegali D) H idronefroze E) Displazi renale

211. Nje 2 vjear ka nje mase abdominale voluminoze dhe pancitopeni. Cila nga diagnozat e meposhtme mund te vertetohet me mielograme? A) H epatoblastome B) N euroblasto m e C) Karcinome me qeliza renale D) Rabdomiosarkome E) Tumori Wilms

212. Nje 6 vj eare ka konj uktivit unilateral jopurulent,jo i dhimbshem me limfadenit preaurikular.Cili eshte shaktari me i mundshem? A) Mycobacterium avium

B) C) D) E)

Bartonella henselae Adenovirus spp. Staphylococcus aureus Chlamydia trachomatis

213. Nje vaj ze me displazi fibrotike te kockave dhe pigmentim j onormal te lekures diagnostikohet me nje patologji e cila lidhet edhe me nje nga manifestimet e meposhtme: A) Anemia. B) Shurdhi m. C) Pubertet prekoks. D) Neurofibrome multiple. E) Glomerulonefrit kronik.

214. Nje femije normal 6 muaj sh paraqet nje zhurme kardiake te vazhdueshme me puls periferik te tensionuar. Cili do ishte hapi i rradhes? A) Vleresim i kariotipit. B) Korigjim i defektit permes kirurgj ise ose kateterit. C) Profilaksi kardiake gjate jetes per procedura te caktuara. D) Ekzaminim kontrolli ne moshen 12 muaj. E) Qetesimi i prinderve.

215. Ne qendren shendetsore ku punon, je duke ndjekur nje pacient me Kraniosinostoze dhe malformacion te kokes e fytyres. Cilin malformacion tjeter kongenital ka me shume mundesi te gjesh te ky pacient? A) Malformacion te trakese dhe ezofagut. B) Malformacion kardiak. C) Malformacion te traktit genito-urinar. D) Malformacion te ekstremiteteve. E) Malformacion te kolones vertebrale.

216. Nje 14 vj eare vuan nga cefale progresive.Ne ekza minim objektiv vihet re edeme papilare bilaterale.CT e kokes rezulton normale. Cili duhet te jete ekza minimi i rradhes? A) Punksion lumbar. B) Rezonance magnetike e kokes. C) CT e orbitave. D) Ekzaminim toksikologjik i urines. E) Matja e nivelit te -HCG.

217. Nje femije 8 muaj sh ka te vj ella dhe te qara per 12 ore.Ne ekzaminim obj ektiv vihet re nje mase ne forme salcice ne kuadratin superior dekster te abdomenit. Cila nga me poshte do ishte me e vlefshme? A) Inserimi i nje sonde nazo-gastrike. B) Ekzaminim i feeve per parazite. C) Hemokulture. D) Ekografi abdominale. E) Ekzaminim me klizme bariumi.

218. Cili ekzaminim eshte me i rendesishem per tu bere te nje 2 vjear me sindrom Beckwith-Wiedemann dhe me nje mase abdominale? A) shintigrafi hepatobiliare B) endoskopi gastrointestinale speriore C) niveli i katekolaminave ne urine D) niveli i alfa-fetoproteinave serike E) cistouretrografia miksionale

219. Nje i porsalindur ka nje vonese ne eleminimin e me koniumit.Radiografia me barium tregon dilatim te kolonit proksimal dhe obstriksion te kolonit distal.Cili do ishte ekzaminimi i rradhes? A) CT abdominale B) Ekzaminim i feeve C) Biopsi rektale D) Matja e Cl- ne dj erse E) Analize kromozomike

220. Nje femije 2 muaj sh ka dispne te rende dhe cianoze.Radiografia e toraksit paraqet kardiomegali minimale dhe pamje retikulare difuze te fushave pulmonare. Cila nga me poshte shpjegon me mire keto rezultate? A) Miokardit viral akut. B) Sindroma e zemres hipoplastike te majte. C) Atrezi e arteries pulmonare. D) Drenazh jonormal pulmonar me obstruksion venoz. E) Transpozicion i eneve te medha.

221. Nje 7 vj eare ka enureze nokturne sekondare.Cili eshte ekzaminimi me i mire per tu ndjekur? A) Ekografia renale. B) Cistouretrografia mi ksionale. C) Radiografia abdominale. D) Analize urine. E) Klirensi i kreatinines.

222. Prinderit e nje femije 8 vje j ane te shqetesuar per shtatin e tij te shkurter. Cili eshte hapi tjeter me i rendesishem? A) Percaktimi i indeksit te mases trupore. B) Matja e stadit te maturimit genital. C) Matja emoshes kockore. D) Matja e shpejtesise se rritjes. E) Matja e raportit peshe/gjatesi.

223. A) B) C) D) E)

Cili nga pohimet e meposhtme eshte tipik per nje konvulsion febril? Zgj at maksimalisht 15 min. Shfaqet ne nje femer me moshe me pak se 6 muaj. Paralize Todd post-konvulsive. Temperature jo >38 C ne 24 oret e ardhshme. Anamneze familj are pozitive per konvulsione febrile ne 40% te rasteve.

224. A) B) C) D) E)

Cila eshte karakteristika dalluese mes manise dhe depresionit? Alterim i aktivitetit. Prezenca e psikozes. Cilesia e humorit. Prezenca e pagjumesise. Prezenca e nje mekanizmi fizpatologjik te nj ohur.

225. Persistenca e reflekseve neonatale eshte tregues i zhvillimit te vone. Refleksi Moro duhet te zhduket ne moshen: A) 1-2 muaj B) 3-5 muaj C) 6-8 muaj D) 10-12 muaj E) 14-18 muaj

226. A) B) C) D) E)

Ne cilin prej kushteve te meposhtme gj endet mioze unilaterale? Sindroma Claude- Bernard- Horner. Paralize e nervit te trete. Tumor qe prek nervin optik. Marrje e barbiturateve. Retinopati e prematuritetit.

227. Nje 8 vje are ka hemipareze pas nje konvulsioni fokal. Brenda 24 oreve dobesia dhe deficiti neurologj ik zhduken. Cila eshte diagnoza me e mundshme? A) Hemiplegji nga hemikraniumi pergj egjes B) Hemiplegji spastike C) Encefalit D) Tumor nententorial E) Paraliza T odd

228. Nje femije dergohet te pediatri per nje vizite kontrolli. Femija kalon nje obj ekt nga nje dore ne tjetren,rri ulur,perserit tingujt e te folures,paraqet perdorim te mire te gishtit tregues. Cila eshte mosha e femijes? A) 2 muaj B) 4 muaj C) 6 muaj D) 8 muaj E) 10 muaj

229. Te folurit eshte nje mates i zhvillimit konj itiv dhe emcional. Me cilen nga situatat eshte e lidhur vonesa ne te folur? A) Sindroma Di George B) Sindroma Williams- Beuren C) Diabet D) Keqtrajtim E) Astma

230. Nje vaj ze e klases se dyte ka aftesi intelektuale te nje nxenesi te vitit te pare te shkolles se mesme. Cili eshte koeficenti i inteligjences se vajzes? A) 90 B) 105 C) 120 D) 135 E) 150

231. Cili nga komplikacionet e meposhtme gjendet te nje person me bulimi nervore pa shqetesime te tjera psikiatrike? A) Peshe trupore nen 75% te peshes ideale B) Abuzim me laksative C) Ushtrim fizik i rende dhe i zgjatur D) Periudha shendoshjeje gj ate te cilave pacienti humb kontrollin E) Seksi femer

232. A) B) C) D) E)

Te gjitha pohimet mbi rregullimin autike j ane te sakta, perve: Me i shpeshte te meshkujt se te femrat. Fillimi eshte para moshes 3 vj eare. Femij et e prekur kane sjellje motore stereotipike. Shumica kane prapambetje mendore. Thuhet se shkaku me i shpeshte eshte rubeola kongenitale.

233. Kolika te latanti eshte shume shqetesuese per prinderit.Cili nga pohimet eshte i sakte? A) Kolika zakonisht eshte e lidhur me ushqimin artificial. B) Kolika tipike fillon ne javen 41-42 gestacionale pavarsisht moshes gestacionale ne lindje. C) Kolika ka prevalence me te larte te femijet kaukaziane. D) Kolika shfaqet me shpesh te femrat. E) Ka studime qe parashikojne temperamentin e femij eve qe kane vuajtur nga kolika.

234. Nje nene sjell femijen 10 vje te mj eku sepse eshte e shqetesuar per zhurmat e perseritura qe ai ben. Nena zbulon se djali kishte patur episode te tilla disa vite me pare. Dj ali pershkruhet si hiperaktiv dhe nuk eshte ne gj endje te kontrolloje impulset e veta. Cilin nga manifestimet e meposhtme pret te shfaqet ne te ardhmen? A) Episode intermitente me fj ale te keqija. B) Zj arrvenie dhe keqtrajtim i kafsheve. C) Enureze nokturne. D) Refuzon te shkoje ne shkolle nga frika e te qenit i braktisur. E) Paralize.

235. Nje femije ka fituar kontrollin e sfinkterave,ben lojra interaktive,thote emrin dhe mbiemrin e vet,pedalon nje tricikel,kopj on nje kryq dhe nje rreth por jo nje katror apo trekendesh. Nuk kap dot nje top qe eshte duke u perplasur,nuk vishet pa ndihme. Cila eshte mosha e femij es? A) 1 vje B) 2 vje C) 3 vje D) 4 vje E) 5 vje

236. A) B) C)

Cili eshte kriteri i rregullimit te te mesuarit? Inteligj ence mbi mesatare. Ngaterrim i shkrimit te shkronjave. Mungese e problemeve e mocionale apo te sjellj es.

D) Mungese lidhjeje mes inteligjences dhe rendimentit ne nje apo me shume fusha. E) Defiit ne perceptimin vizual apo auditiv.

GASTROHEP ATOLOGJIA 1. Cili sht funksioni fiziologjik m i rndsishm i acideve biliare? A) Favorizon eskretimin e substancave toksike B) Favorizon absorbimin e vitamins B12 C) Ruan pH e duhur intestinal D) Favorizon absorbimin e yndyrnave E) Favorizon absorbimin e aminoacideve

2. N cilin prej rasteve t mposhtme sht transmural lezioni intestinal? A) B) C) D) E) Koliti uleroz Celiakia Morbus Crohn Asnjra prej ktyre Smundja e Whipple

3. Nj puntor 35 vjear bn nj kontroll ambulator pr t vlersuar gj ndjn me diarre dhe rni n pesh. Nprmjet ezofagogastroduodenoskopis, q u realizua pr marrjen e biopsis n zorrn e holl, u vu re eritem e ezofagut, me disa erozione, dy ulera gastrike si dhe uleracione t shumta duodenale. N biopsi u vu re rrafshim i vileve intestinale. Cili prej ekzaminimeve t mposhtm do t kryeje si hapin pas ardhs diagnostikues? A) B) C) D) E) ERCP (Kolangiopankreatolografi retrograde endoskopike) Kolonoskopi G astrine m ia Krkohen anti-trupat anti-endomisium dhe anti-gliadin Testin e Schilling

4. Kolangjitet shkaktohen m shpesh nga: A) Prdorimi i pijeve t ftohta nga pacient me litiaz multiple t rrugve biliare B) Bakteret Gram + q vijn m shpsh nga arteria hepatike C) Efekti detergj ent i acideve biliare D) Disfunksioni i sfinterit t Oddit dhe autodigjestioni nga enzimat pankreatike q kan br refluks n rrugt biliare E) Bakteret Gram + dhe Gram q vijn nga zorrt

5. T gj itha gj endjet e mposhtm jan shkaktare t pseudoobstruksionit intestinal kronik, prve: A) B) C) D) E) Skleroderm ia Prdorimi i antidepresivve triciklik Diabeti melitus H ipertiroidizmi Amiloidoza

6. Lezionet e Morbusit Celiak hasen n nivelin e: A) B) C) D) E) Sto m akut Vetm jejunum Duodenit dhe jej unumit Ileusit Vetm duodenit

7. Cila prej patologjive mund t shfaq ascit: A) B) C) D) E) Sindroma nefrozike Retokoliti uleroz Steatoza hepatike Defiiti i disakaridazs Tireotosikoza

8. Cilt jan parametrat laboratorik m t prdorshm pr t monitoruar proesin inflamator n hapatitin kronik: A) B) C) D) E) Kriprat biliare totale Bilirubinemia e fraksionuar Transaminazat dhe gamaglobulinat sierike Kohn e protrombins Fosfataza alkaline

9. T gj itha gjendjet e mposhtme j an faktor rreziku pr kolelitiazn kolesterinike, prve: A) B) C) D) E) Obeziteti Terapia me klofibrat Terapia me estrogj ene Rezeksioni kirurgj ikal i ileusit Retokoliti uleroz

10. Si mptoma e dj egies retrosternale sht tregues specifik pr: A) B) C) D) E) Uler gastrike Uler duodenale Gastriti autoimun Refluksi ezofageal Gatriti nga Helicobacter pylori

1 1. Nj murator 21 vj ear, ka bilirubinemin totale= 2.3 mg/dl dhe me fraksionin e drejt= 0.3mg/dl. Nuk ka histori familj are pr hepatopati, nga anamneza evidentohet se pi rreth 40 gr alkol n dit, nuk ka simptoma t tj era ose shenja hepat opatie. Hemokromi, numrimi i retikulociteve, ALT, AST, fosfataza alkaline, dhe GGT jan n norm, sikurse elektroforeza dhe INR. Gjithashtu, rezulton HBsAg negativ, HBsAb pozitiv, HBcAb negativ. Ju do t orientoheshit drejt:

A) B) C) D) E)

Hepatit B aktiv Hepatit B n shrim Hepatit C Hepatit alkolik Smundja e Gilbert

12. Mund t shfaqet hipoglicemi pas ushqyerjes (postprandiale): A) B) C) D) E) Morbusi Celiak Pas gastrorezeksionit subtotal Prej nj fistule gastro-kolike Prej lemzs s vazhdueshme N ulern duodenale

1 3. Cila prej smundj eve t mposhtme mund t jet shkak i ko ms hepatike? A) B) C) D) E) Hepatiti kronik aktiv me zhvillim cirrotik Ikteri obstruktiv nga kalkuloza cirrotike Cirroza hepatike e dekompensuar Smundja e Gilbert Cirroza septale jo e plot

14. Pr ciln arsye realizohet testi me ksilozium (xylosio)? A) B) C) D) E) Pr verifikimin e funksionalitetit t disakaridazs intestinale Pr t verifikuar integritetin e siprfaqes s zorres s holl Pr verifikimin e funksionalitetit t pankreasit Pr verifikimin e fosforilimit t karbohidrateve n zorr Pr vlersimin e absorbimit t glicideve

15. T gj ith medikamentet e mposhtm mund t prdoren n trajtimin e ulers peptike, prve: A) B) C) D) E) Salazopirina O m eprazol Sucralfat Ranitidin Pirenzepin

1 6. Zotria 52-vj ear ka rreth 2 vite me diarre dhe rnie t theksuar n pesh. Nuk ka histori pr gj akderdhje gastroenterike dhe gj aku ocult n fee n tre kampion. Rrfen se n t kaluarn ka qn prdorues i rregullt i alkolit, por prej disa vitesh referon se nuk e ka prdorur m. Esht diabetik dhe mjekohet me insulin. Para gj asht vitesh ka br kolecistektomi. Ekzaminimet laboratorike rezultojn normal, prve nj anemie t leht dhe nderkoh ekzaminimi radiologj ik i zorrve t holla e t trasha rezultoi normal. Cfar do t sugjeronit: A) B) C) D) E) ERCP ( kolanxhopankreatolografia retrograde endoskopike) Biopsia e jejunumit Dozimi i yndyrnave n fee Testi i frym marrj es pr disakaride Kolonoskopi me biopsi

1 7. Smundja e Crohn prfshin: A) B) C) D) E) Vetm ileusin Vetm ileusin dhe kolonin Vetm kolonin Ilesusin, kolonin dhe pjes t tjera t aparatit trets Ileus, kolon dhe rektum

1 8. Sindroma e Mallory-Weiss verifikohet n rastin e: A) B) C) D) E) Tumori endokrin i pankreasit Diarrea sekretore T vj ellat e vazhdueshme Tumori hipofizar Polipoza xhiovanile

1 9. N Morbusin Crohn m shpesh litiaza e kolecists sht e lidhur me: A) Rritje e absorbimint t kriprave biliare n ileus B) Reduktim i pool-it t kriprave biliare C) Rritja e absorbimit t kolesterolit nga ileusi D) Bashk shoqrimi me kolecistitin E) H ipovitaminoza

2 0. N ciln pjes t traktit intestinal realizohet absorbimi i hekurit?

A) B) C) D) E)

Duodeni Ileus Kolon Sto maku I gj ith trakti intestinal

2 1. Cili sht prbrsi kryesor i kalkujve biliar radiotrasparent: A) B) C) D) E) Bilirubina Kripra biliare Urobilina Kolesteroli Lecitina

2 2. Cili sht tumori beninj m i shpesht i heparit? A) B) C) D) E) Hemangioma kapilare Fibroma jaline Fibroadeno m a N eurofibrom a Adenoma e duktuseve biliare

2 3. Peritoniti bakterial spontan prfaqson nj komplikanc t cils prej patologjive t mposhtme: A) B) C) D) E) Apendisiti akut Divertikuloza e kolonit Cirroza hepatike Smundja e Crohn Kolecistiti

2 4. N cilt limfonoduj metastatizon m shpesh karcinoma e j unksionit rektosig m oid al? A) B) C) D) E) H ipogastrik Inguinal Mezenterikt e poshtm Mezenterikt e siprm Tripodit celiak

2 5. T gjith parametrat e mposhtm prdoren pr klasifikimin Child-Pugh t cirrozs hepatike, prve: A) B) C) D) E) A lbumine m i Asciti Biliru bina Kolinesteraza Encefalopatia

2 6. Cilat j an karakteristikat e Sindroms Plummer-Vinson? A) Esht prfshirja e ezofagut (dizfagi) e shoqruar me anemi mikrocitike dhe glosit (inflamacion i gj uhs) B) Odinofagi me anemi sideropenike C) Dizfagi me glosit e pa alternime hematike D) Dizfagi paradoksale me anemi sideropenike dhe glosit E) Akalazi ezofageale me anemi makrocitike

2 7. Sindroma e Carolit prkufizohet si: A) Patologji e rrugve biliare intrahepatike, karakterizuar nga zgjerime cistike multiple t duktuseve biliare segmentare B) Patologji biliare post-inflamatore C) Patologji neoplazike t rrugve biliare intrahepatike D) Litiaz t rrugve biliare intrahepatike E) Litiaz t rrugs principale biliare

2 8. Sindroma e Reye shoqrohet shpesh me ciln prej gj ndjeve? A) B) C) D) E) Diabeti B ruceloza Prdorimi i acidit acetilsalicilik H e m okro m atoza Prdorimi i heparins

2 9. Alternativat rreth sekretimit t acidit nga stomaku n kushtet e ulers peptike gastrike jan t gabuara, prve: A) Si n gj endjen normale B) Si n ulern duodenale

C) Mungon pas stimulimit maksimal D) M e rritur se normalisht n kushtet e stimulimit bazal E) M e ult se n gj endjen normale

3 0. Cili prej alternimeve t mposhtm sht karakteristik pr Sindromn DubinJohnson: A) B) C) D) E) Hiperbilirubinemi Hiperbilirubinemi Hiperbilirubinemi Hiperbilirubinemi Hiperbilirubinemi direkte me kolestaz t rritur direkte me kolestaz normale indirekte me rritje t kolestazs indirekte me shtim t citolizs direkte me shtim t citolizs dhe kolestazs

3 1. T gjith elementt e mposhtm j an karakteristik e kuadrit klinik t Morbusit t Wilson, prve: A) B) C) D) E) Simptoma dhe shenjat e prfshirjes hepatike Unaza e Kayser-Fleischer Manifestime neurologjike Ngj yra bronx e lkurs Ulje t nivelit sierik t ceruloplazmins

3 2. Cila prej shenjave ose simptoma ve t mposhtme mundsoj n diagnozn diferenciale midis ulers duodenale t preforuar dhe pankreatitit kronik: A) B) C) D) E) Dhimbje t forta abdominale Shok hipovolemik Zhdukja e otuzitetit hepatik Mas e palpueshme e me zogastrit T vjella t vazhdueshme

3 3. T gjitha gjendjet e mposhtme shoqrohen me rritje t incidencs s hepatokarinoms, prve: A) B) C) D) E) Infeksioni kronik nga virusi C Infeksion kronik nga virusi B H emokro m atoza Hepatopatia alkolike Angioma hepatike

3 4. Koloni absorbon n lidhje me masn q i vj en nga ileusi: A) B) C) D) M shum se 90% t ujit dhe elektrolitve Midis 50% dhe 90% t uj it dhe elektrolitve M pak se 50% t ujit dhe elektrolitve M shum se 90% t ujit dhe 50% t elektrolitve

E) 50% t ujit dhe t gj ith elektrolitet

3 5. Cila prej karakteristikave t mposhtme prcakton early gastric cancer A) B) C) D) E) Prek mukozn ose submukozn gastrike Prek tunikn mus kulare Prek tunikn sieroze Prek t gjith paretin gastrik Prek t gjith stacionet limfonodale satelitare

3 6. Cila prej gjendjeve shoqrohet me pranin e antitrupave anticitoplazmin t neutrofileve (ANCA)? A) B) C) D) E) Cirroza biliare primare Hepatiti autoimun tip I Hepatiti autoimun tip II Kolangiti sklerozant primitiv Hepatokarcino m a

3 7. Cili prej medikamenteve sht i prfshir n zhvillimin e insufiencs hepatike akute: A) B) C) D) E) Kontraceptivt oral FANS Paraceta m oli T etraciklina Rifa m picina

3 8. Qelizat oxintike (parietale) j an t lokalizuara n mukozn e: A) B) C) D) E) Fundusit dhe antrumit Antrumit dhe trupit Trupit dhe fundusit Antrumit dhe pj ess circumkardiale Zona prepilorike

3 9. Cila prej alternativave sht e vrtet: A) Encefalopatia hepatike sht rregullim cerebral me rritje t komponentit n e u r o i n i b it o r B) Aminoacidet aromatike jan: triptofani, fenilalanina, tirozina dhe metionina C) Era e nj pacienti me cirroz sht sekondare nga hiperamoniemia D) Merkaptanet derivoj n nga metabolizmi intestinal i glutamins E) Terapia diuretike prmirson encefalopatin hepatike

4 0. Si mptoma kryesore e pankreatitit akut sht: A) B) C) D) E) Ikter Nauze e t vj ella Dhimbj a Diarre Te mperatur

4 1. Pankreatiti akut shkaktohet m shpesh nga: A) B) C) D) E) Litiaza e kolecists Kalkuj intrahepatik H iperlipide m i Abuzimi me alkolin Litiaza e koledokut

4 2. T gj itha jan forma t diarres sekretore, prve: A) B) C) D) E) Diarrea nga infeksioni me vibrio cholerae Diarrea nga sindromi karcinoid Diarrea nga VIPoma Diarrea nga defiiti i disakaridazs Diarrea nga infeksioni me Escherichia coli

4 3. T gjith kta parametra mund t shprehin nj kolestaz, prve njrit: A) B) C) D) Fosfataza alkaline 5-n uk leotidaz a D iam ino-oksidaza Gamaglutamiltranspeptidaza

E) Bilirubina

4 4. Cili sht faktori etiologjik m i r ndsishm i hipertensionit portal nga cirroza hepatike? A) B) C) D) E) Reduktimi i qarkullimit hepatik Nekroz t qelizave hepatike Infiltrimi limfocitar Septe fibrotike Depozitimi biliar

4 5. Si mptomatologjia e dhimbj eve n ulern duodenale prmirsohet nga: A) B) C) D) E) Pas ushqimit Jashtqitja Gjumi T rrish shtrir n ann e majt Aktiviteti fizik

4 6. N insufiencn hepatike akute t rnd cili prej ekzaminimeve t mposhtm ka rndsi prognostike? A) B) C) D) E) Transam inazat Latikodeidrogj enaza Koha e Quick Albu min e mi a Fosfataza alkaline

4 7. Kalkujt biliar prej bilirubinat kaliumi shfaqen m shpesh tek pacientt e prekur nga: A) B) C) D) E) Morbusi Crohn n ileus Rezeksioni ileal Anemia hemolitike kronike Ulera peptike Retokoliti uleroz

4 8. Nj pensionist 72 vjear (q ka punuar si marangoz) me kardiopati iskemike dhe SPOK, shtrohet n spital pr bronkopneumoni dhe i prgj igjet mj ekimit empirik

me cefotaxime. N ditn e gjasht t shtrimit rishfaqet temperatura dhe nis dhimbja abdominale me diarre t ujshme. Cdo t bnit n kto rrethana: A) B) C) D) E) Zvendsimin e cefotaxime me clindamicine Krkoni ezofagogastroduodenoskopin me biopsi t jej unumit Klizmn e zorrve t holla Test imunologj ik duke krkuar toksinat e clostridium difficile K olonoskopi

4 9. Asciti i pa komplikuar tek nj pacient cirrotik sht gjithnj i karakterizuar nga: A) B) C) D) E) Mungesa totale e qelizave inflamatore A lbum inemia<3g/dl pH<7 Pamje t turbullt 250 polimorfonukleat/cc

5 0. T gjitha smundj et e mposhtme shkaktoj n cirroz, prve: A) B) C) D) E) Defiiti i alfa 1-antitripsins S kistozomiaza Smundja e Gaucher Hepatiti nga virusi C Hepatiti nga virusi A

5 1. Cila prej alternativave t mposhtme nuk sht shenj e apendisitit akut: A) B) C) D) E) A noreksi Stranguri Dhimbje n kuadratin iliak t djatht T vj ella Te mperatur

5 2. Cili prej alternimeve sht karakteristike e koms hepatike: A) B) C) D) E) Konfuzioni mental Flapping tremor H iperef leks i H iporefleksi Asnjra prej prgj igjeve

5 3. Gastrinemia > 1000pg/ml (norma <100 pg/dl) sugjeron m shum pr: A) B) C) D) E) Sindroma Sindroma Sindroma Sindroma Sindroma e Zollinger-Ellison Menetrier e Mallory-Wiss e Verner-Morrison e antriumit t ngecur

5 4. Kolestaza prej medikamenteve karakterizohet nga: A) B) C) D) E) Hiperbilirubinemi kryesisht indirekte Munges t rritjes s gamaGT Hiperbilirubinemi direkte q zgj at prej kohsh Mungesa e pruritit Eozinofili e theksuar (>20%)

5 5. T gjitha manifestimet j an pjes e kuadrit klinik t temperaturs mesdhetare familjare, prve: A) B) C) D) E) Te mperaturs Dhimbje abdominale Pleuriti Ikter Artralgji

5 6. N rastin e smundjes divertikulare t zorrs s trash, nuk ka asnj her lezione t tilla n: A) B) C) D) E) Ciekum Koloni ashendent Flesurn hepatike e t lienit R ektum Mund t formohen kudo

5 7. Nj pacient paraqet rritje t kohs s protrombins me 6 sek m shum se normalja dhe trombocitet 45,0000/cc. Cili prej ekzaminimeve sht i kundrindikuar? A) Paracenteza pr diagnostikim B) Biopsi hepatike perkutane C) Pozicionimin e sonds s Sengstaken-Blakemore

D) Kolonoskopia E) G astroskopi a

5 8. N ciln prej smundjeve gjenden m shpesh komplikanca si fistulat (enteroenterike, enterokutane etj.) A) B) C) D) E) Celiakia Morbusi Crohn Koliti uleroz Kancer i zorrs s trash Ulera gastrike

5 9. Stimulimi maksimal i sekretimit t acidit gastrik realizohet me an t: A) B) C) D) E) Pe ntagas trin a G lukagone Sekretina Kolecistokinina Vip

6 0. Alternativat e mposhtme pr sindromn e Zollinger-Ellison j an t sakta, prve: A) B) C) D) E) Shoqrohet me ulern peptike, q shpesh i prgj igj et shum pak terapis Shoqrohet me diarre kronike Shoqrohet me hipergastrinemi t theksuar Ulerat peptike mund t j en t shumta Shrohet pas zhdukjes s Helicobacter pylori

6 1. N retokolitin uleroz, cila prej gj endj eve rrit rrezikun pr kancer t kolonit? A) B) C) D) E) Prania e smundj es prej shum kohsh Histori pr megakolon toksik Prezenc t pseudopolipeve gjat ekzaminimit endoskopik Rezistenc ndaj terapis steroide Zhvillim t ashpr t smundjes

6 2. Nga cila prej gj endjeve t mposhtme shkaktohet m shpesh obstruksioni i kolonit?

A) B) C) D) E)

A derenca t Volvolusi Hernia Karcinoma Divertikuliti

6 3. Cila prej alternativave t mposhtme rreth sindroms s Gilbert sht e sakt? A) Ikteri spjegohet pjesrisht nga hemoliza B) Prdorimi i fenobarbitalit shkakton n 4-8 dit nj ulje t dukshme t bilirubins n gj ak C) Prdorimi i kortizonit rrit nivelin e bilirubins n gj ak D) Biopsia tregon pr alternime t kanalikulave biliare E) Prdorimi i fenobarbitalit rrit nivelin e bilirubins

6 4. T gjitha alternativat e mposhtme rreth smundj es s Menetrier j an t sakta, prve: A) B) C) D) E) Esht patologji e zorrs s holl Esht patologji e stomakut Mund t shoqrohet me hipoalbuminemi Mund t shkaktoj anemi nga gj akderdhja e fshehur Diagnoza bazohet tek ekzaminimi endoskopik ku merret biopsi

6 5. Sindroma e Crigler-Najjar karakterizohet kryesisht nga rritja n gj ak e: A) B) C) D) E) Bilirubins direkte Bilirubins indirekte Kriprave biliare Fosfatazs alkaline Gama-GT

6 6. Cili prej parametrave t gjetura n likidin e ascitit sht tregues pr diagnostikimin e peritonitit bakterial spontan: A) B) C) D) E) Pamja si qumsht Prania e 300 granulociteve neutrofile/mm3 Prania e 300 leukociteve totale/mm3 Prania e m shum se 3g% e proteinave Pamje hemorragjike

6 7. T gjitha alternativat e mposhtme rreth cirrozs biliare primare j an t sakta, prve: A) Shoqrohet shpesh me prurit B) Nj prqindje e lart e pacientve rezultojn pozitiv pr antitrupat a n ti m it o k o n d r C) Prdorimi i acideve biliare mund t j et i dobishm kontrollin e simptomave D) Esht m e shpesht tek femrat E) Trajtimi m efikas sht ai me D-penicilaminn

6 8. Virusi delta sht virus difektoz, q pr tu replikuar ka nevoj pr: A) B) C) D) E) Citomegalovirus Virusi i hepatitit B Virusi i hepatitit A Virusi i Epstein-Barr Hepatiti G

6 9. T gjitha shenjat e mposhtme gjenden n insufiencn hepatike si pasoj e hepaitit kronik C, prve: A) B) C) D) E) Purpura e palpueshme n ekstremitetet e poshtme Otuzitet q zhvendoset dhe sht dekliv gjat perkusionit t abdomenit Flapping tremor Unaza e Kayser-Fleischer Nishanet arakniform

7 0. Hemorragjia nga variet ezofageale sht komplikanc e: A) B) C) D) E) Hipertensionit arterial Hipertensionit portal Insufiencs kardiake t dj atht Kakezia nga neoplazia Insufienca hepatike

7 1. Cili sht efekti q jep gastrina gj at aktivitetit t saj fiziologjik: A) Stimulon sekretimin e acidit klorhidrik B) Stimulon rritjen e cck (kolecistokinina) C) Prshpejton zbrazj en gastrike

D) Stimulon sekretimin e bikarbonateve E) Stimulon e skretimin e lngut biliar

7 2. Cili prej alternimeve hematokimi ke sht m i shpesht n morbusin Celiak: A) B) C) D) E) H ipokalemi H ipolipemi Hipoalbuminemi Hiposidere mi Hipofosforem i

7 3. N kushtet e malabsorbimit t gjeneralizuar, cila sht alternativa e sakt? A) N pankreatitin kronik malabsorbimi i gjeneralizuar sht paraprakisht prezent B) Jashtqitja karakterizohet nga vllim fekal i ult dhe numr i lart i herve t jashtqitjes C) Steatorrea sht gjithnj prezent D) Steatorrea shoqrohet me steatoz hepatike E) Feet jan gj ithnj likide

7 4. Cila sht metodika m e prshtatshme pr t diagnostikuar refluksin gastroezofageal acid? A) B) C) D) E) Manometria ezofageale pH-metria ezofageale e 24 orve Ekzaminimi radiologjik i ezofagut Ezofagogastroduodenoskopia Testi i Bernstein

7 5. T gjitha alternativat e mposhtme rreth akalazis j an t sakta, prve nj rs: A) Mund t shkaktoj dizfagi, dhimbje toraksi, regurgitim B) Karakterizohet nga alternimet gj at relaksimit t sfinterit ezofageal t poshtm C) pH-metria ezofageale sht ekza minimi m i domosdoshm pr ta d ia g n o sti k u a r D) Nifedipina kontrollon simptomat tek shum pacient E) Mund t trajtohet me toksinn e botulins

7 6. T gjitha alternativat e mposhtme j an shkak i malabsorbimit tek nj pacient me smundj en e Crohn, prve nj rs: A) B) C) D) E) Lokalizimi i gj ere n jejunum-ileal Smundja rekto-sigmoide aktive Kontaminimi bakterial i zorrs s holl Fistula jejunum-kolike Mbetje nga rezeksioni intestinal i gj er

7 7. Sekretimi gastrik i theksuar dhe ulerat peptike prsritse vrehen n: A) B) C) D) E) H ipoparatiroidizm Sindroma e Zollinger-Ellison Sindroma e Mallory-Weiss Sindroma Dumping Sindroma e Verner Morrison

7 8. Litiaza pigmentare e kolecists ndodh m shpesh n: A) B) C) D) E) Anemin hemolitike Anemin hipokromike H iperkole sterole m in A teroskleroza Pankretiti kronik

7 9. Cili prej medikamenteve sht m i prshtatshmi pr trajtimin e kolitit pseudomembranoz? A) C eftazidim e B) V anco micine C) Streptomicine D) M esalazine E) Linco m icine

8 0. N urin nuk gj endet asnj her bilirubina indirekte sepse: A) B) C) D) E) Nuk sht hidrosolubil Pesha molekulare nuk e lejon filtrimin e saj glomerular Absorbohet trsisht nga tubujt Nga kontakti me ajrin oksidohet menj her Eliminohet nga veshka

8 1. Cila prej patologjive komplikohet m shpesh me megakolon toksik? A) B) C) D) E) Polopoza e prhapur e kolonit Divertikuliti i kolonit Megakoloni kongenital Retokoliti uleroz Koliti granulomatoz

8 2. Nuk shkakton hipertension portal? A) B) C) D) E) Cirroza biliare primitive Obstruksioni i fluksit venoz hepatik Tromboza e vens splenike Cirroza hepatike post-hepatike Steatoza hepatike

8 3. Nuk ka vler diagnostikuese realizimi i biopsis intestinale n ciln prej s m u n dje v e? A) B) C) D) E) Defiiti i folateve Smundja e Whipple A betalipoproteine m ia A ga maglobulinemia Kompleksi Mycobacterium avium

8 4. Cili prej komplikacioneve nuk i bashkangjitet smundjeve inflamatore intestinale kronike: A) Eritema nodoze B) Ragada (fisura) anale C) Uveiti D) Peri ko la ng i ti E) S pondiloartroza

8 5. Sa sht koha e inkubimit t nj toksiinfeksioni ushqimor nga toksina sta fil o k o k si k e : A) 3-6 or B) 1 dit

C) 20-30 minuta D) 2 dit E) 5-8 dit

8 6. Cili prej alternimeve metabolike favorizon neurotoksicitetin nga amoniaku? A) B) C) D) E) Hipersodiemia Hiperglicemia A lkaloza Acidoza Hiperkalemia

87.Cili sht komplikacioni m i shpesht i morbusit Crohn? A) B) C) D) E) Obstruksioni Hemorragjia Pe rforacion i Peritoniti Dermatiti herpetiform

8 8. N smundjen e refluksit gastroezofageal, dizfagia kur sht e pranishme mund t jet: A) B) C) D) Vetm funksionale Vetm organike Funksionale dhe organike As funksionale dhe as organike, duke qn se dizfagia nuk sht simptom e ksaj smundjeje E) Dizfagia nuk sht asnjher e pranishme

8 9. Cili prej ekzaminimeve laboratorike sht i vlefshm pr vlersimin e shkalls s aktivitetit inflamator tek hepatiti kronik: A) B) C) D) E) Dozimi i imunoglobulinave n gjak Dozimi i albuminemis Dozimi i bilirubinemis Dozimi i fasfatazemis alkaline Gama-GT

9 0. Vitamina B12 orale, absorbohet n:

A) B) C) D) E)

Stomak Jejunum Ileus Kolon Duoden

9 1. Cila prej gjendjeve shoqrohet m shpesh me adenokarcinomn e ezofagut? A) B) C) D) E) Akalazia Alkolizmi Tiloza Ezofagu i Barret Duhanpirja

9 2. Cila prej alternativave rreth hepatitit kronik autoimun t tipit I nukl sht e sakt: A) B) C) D) E) Prek kryesisht t moshuarit Esht m i shpesht tek femrat Shoqrohet me pranin e antitrupave antinukl Shoqrohet me hipergamaglobulinemi N prgj ithsi i prgj igjet terapis imunosupresive

9 3. Cfar do t thot dizfagi? A) B) C) D) E) Vshtirsi n deglutim Vshtirsi n artikulimin e fjals Pamundsi absolute n kalimin e ushqimit n ezofag Dhimbje krampiforme retrosternale pas deglutimit Pirroz

9 4. Kuadri i nj koleciste t zmadhuar t palpueshme tek nj pacient pa ikter, sugjeron pr: A) B) C) D) E) Karcinoma e ampul s Vater Karcinoma e pankreasit Kalkuli i koledokut Kalkul n duktusin cistik Kalkuloz e kolecists

9 5. Aktualisht cili sht ekza minimi m i dobishm pr diagnostikimin dhe trajtimin e kalkulozs s koledokut? A) B) C) D) E) K olangio-R M N Ecotomografia e abdomenit superior Kolangiografia e.v TAC ERCP (kolangio pankreatografia retrograde endoskopike)

9 6. Kush nuk e karakterizon smundjen e heparit t yndyrosur jo alkolik (NAFLD): A) B) C) D) E) Shoqrimi me obezitet, diabet e hipertrigliceridemi Prevalenc e lart n popullatn n prgjithsi Alternime t shpeshta t indekseve t metabolizmit t hekurit Parenkima hepatike bhet hiperecogj ene Zhvillim sistematik drejt insufiencs hepatike

9 7. Cila prej gjendjeve mund t ndrhyj duke maskuar rritj en e amilazs n pankretitin akut? A) B) C) D) E) Hipertrigliceridemia H iperglice mia H ipokale m ia H iperamoniem ia Hiperkolesterolemia

9 8. Cili prej medikamenteve nuk sugj erohet n zhdukjen e Helicobacter pylori: A) B) C) D) E) Omepra zo li Sucralfati A m oxicilina Claritromicina Bismuthi koloidal

9 9. Cili prej ekzaminimeve sht m i pranueshm pr t diferencuar ikterin kolestatik prej obstruksionit t rrugve biliare ekstra-hepatike nga ikteri kolestatik i n tr a h e p a ti k : A) B) C) D) Prcaktimi i bilirubins direkte Prcaktimi i fosfatazs alkaline Ecografia hepatike Prcaktimi i gama-GT

E) Dozimi i acideve biliare

100. T gjitha smundj et shkaktoj n ascit, prve njrs: A) B) C) D) E) Skompensi me staz M ixedema Cirroza hepatike H ipertiroidizmi Tuberkuloza intestinale

101. Sindroma e Zollinger-Ellison shoqrohet me t gjitha gj endjet e mposhtme, prve njrs: A) B) C) D) E) Smundja uleroze e rnd Diarrea Hipersekrecioni acid gastrik, n kushtet bazale Reduktimi i gastrinemis pas stimulimit me sekretin Hipergastrine m i

102. Kush nuk e shkakton cirrozn hepatike? A) B) C) D) E) Infeksioni nga virusi A Infeksioni nga virusi C Inflamacioni kronik i rrugve biliare H e m okro m atoza Infeksioni nga virusi B

103. Anemia makrocitike shpesh shkaktohet nga: A) B) C) D) E) Karcinoma e ezofagut Polipi rektal H ipertiroidizmi Sindroma me humbje t proteinave Gastriti atrofik i trupit gastrik

104. Tre student kan ngrn sbashku dy dit m par (sup, mish me zarzavate dhe mblsir me krem kan paraqitur episodin e gastroenteritit febril.

Analizat laboratorike tregojn pr hipopotasemi dhe qeliza inflamatore n fee. Ky kuadr t bn t mendosh pr: A) B) C) D) E) Kolera Salmoneloza Toksiinfeksion nga stafilokoku Toksiinfeksion nga clostridium Toksiinfeksion nga Bacillus cereus

105. Si konfermohet diagnoza e infeksionit nga HCV? A) B) C) D) E) Prania e HCV-ARN Transaminazat rriten t paktn 7 her m shum se vlera normale Prania e anti-HBc Prania e anti-HCV Prania e anti-HBs

106. Si mund t konsiderohet pacienti q paraqet kto vlera t markuesve pr hepatitin viral B: HBsAg pozitiv; antiHBs negativ; HBeAg negativ; anti HBe pozitiv; antiHBc pozitiv; HBV ADN (qarkulluese) negativ: A) B) C) D) E) Hepatit akut Hepatit kronik aktiv Mbarts kronik i HBsAg Infeksion i mparshm i shruar Asnjra prej prgj igjeve

107. Diarrea e udhtarit shkaktohet m shpesh nga: A) B) C) D) E) Staphylococcus aureus Herpes virus Mycobachterius bovis Escherichia coli enterotoksigjene Campylobacter jejuni

108. Infeksioni kronik nga virusi i hepatitit C shoqrohet m shpesh me: A) B) C) D) E) Purpura krioglobulinemike Smundja e Churg-Strauss Leuemia me qeliza ka pelute Panarteriti nodoz Polimialgjia reumatike

SEMUNDJE INFEKTIVE 1. Cila sht vatra infektive m e shpesht q shkakton sepsi tek pacientt jo t h o s p ita liz u a r: A) B) C) D) E) Infeksion q nis nga trakti gastroenterik Infeksion q nis nga rrugt e frymmarrjes Infeksion i endokardit Infeksion i aparatit urinar Infeksion/kolonizim i CVC

2. Cila prej smundj eve prkufizohet si zoonos: A) B) C) D) E) A skaridiaza Ankilostomiaza A mebiaza T oxoplazmoza Enterobiaza

3. Temperatur 38-39 celsius, prurit t trupit, rash vezikular me prurit t gj eneralizuar me makula, papula, vezikula, flluska e kore, anoreksi, frisone, cefale. Ky kuadr klinik t bn t mendosh pr: A) B) C) D) E) Lija e dhenve Fruthi Rubeola Skarlatina Mononukleoza

4. Toxoplazmoza: A) B) C) D) E) Zhvillohet n mnyr asimptomatike tek personi imunoko mpetent Nuk sht kurr infeksion oportunistik Rrall her j ep limfadenit T gj itha prgjigjet Asnjra prej prgj igjeve

5. Nprmjet cilit prej likideve trasmetohet HIV-i:

A) B) C) D) E)

Djersa Pshtyma Lot Urina Gjak

6. Polmoniti lobar i mirfillt shkaktohet kryesisht nga: A) B) C) D) E) Streptococcus pneumonie Listeria monocytogenes Stafilococcu aureus Streptococcus pyogenes Mycoplasma pneumonie

7. Botulizmi nga ushqimi shkaktohet kryesisht pas konsumimit t: A) B) C) D) E) Mish i pa br ose pak i br Ushqim kutie i konservuar n vaj ose i marinuar n kushte shtpie Ushqim i konservuar n uthull Uj i ndotur Midhje ose peshk t gjall

8. Hepatiti akut nga virusi A: A) Dyshohet pas prdorimit t ushqimeve t konservuara B) Diagnostikohet vetm nse ka pozitivitet pr antitrupat anti-HAV t klass IgG C) Trasmetohet me rrug oro-fekale D) Shkakton rritje t bilirubinemis por q nuk shkon m shum se 25mg/dl E) Nuk ka asnjher zhvillim fulminant

9. Hepatitet q transmetohet me rrug oro-fekale j an: A) B) C) D) E) Hepatiti A dhe D Hepatiti B Hepatiti C dhe D Hepatiti A dhe B Hepatiti A dhe C

1 0. N tripanozomiazn amerikane cilat jan manifestimet m t rnda dhe m t shpeshta t smundjes kronike? A) B) C) D) E) M egaezofagu M egakolon Insufienca hepatike Kardio miopa ti a Insufienca reanle

1 1. Cila prej patologj ive t mposhtme q lidhen me infeksionin nga T oxoplazma Gondii prej eskluzivisht pacientt imunodepres: A) B) C) D) E) Li n mf oadenopati a Em briopatia Encefaliti T gj itha prgjigjet Asnjra prej prgj igjeve

1 2. Sepsi karakterizohet nga: A) Prezenca tranzitore e baktereve n gj ak B) Hiporeaktiviteti ndaj nj infeksioni C) Lshimin apo hedhjen n gj akun qarkullues t baktereve nga nj vatr infektive e lokalizuar D) Lokalizim i infeksionit n nj ose dy organe E) Reaksion i ekzagj eruar i organizmit ndaj nj infeksioni

1 3. Oksiuraza shkaktohet nga: A) B) C) D) E) Echinococcus granulosus Taenia solium Enterobius vermicularis Necator americanus Hymenolepis nana

1 4. Cili prej agjentve etiologj ik sht shkaktari m i shpesht i osteomielitit akut? A) B) C) D) E) Salmonella tifo Haemophilus influentiae Stafilococco aureus Streptoc occo Gonococco

1 5. Cili sht vektori i Leishmania donovani? A) M ushkonja B) Miza C) Fleboto m i D) Pleshti E) T gj itha prgjigj et

1 6. Cili prej agjentve etiologj ik sht m shpesh shkaku i miokarditit akut? A) B) C) D) E) Stafilococco aureus Coxackie virus EBV Streptococcus viridans Lija e dhenve

1 7. Herpes Zoster: A) B) C) D) E) Shkaktohet nga i njjti virus i lis s dhenve Shkaktohet nga nj virus i ngjashm me at t lis s dhenve Shkaktohet nga nj rinovirus me ADN Shkaktohet nga nj enterovirus Shkaktohet nga coxackie

1 8. Cili prej ktyre baketereve shkakton leukopeni? A) B) C) D) E) Klensiella oxitoca Salmonella typhi Streptococco i grupit A Enterococco Ps udo m onas e

1 9. Neoplazia m e shpesht gjat AIDS sht: A) B) C) D) E) Sarkoma e Kaposit Adenokarcinoma e laringut Leucemia limfatike kronike Adenokarcinoma gastrike Adenokarcinoma e pulmonit

2 0. Gj at mononukleozs infektive, n ekzaminim hemo kromocitometrik vihet re: A) B) C) D) E) Lim fopeni Neutrofili relative E ozinofili Limfocitoz me prani t limfociteve aktive Policitemi

2 1. Meningiti purulent karakterizohet nga cila gj endje e likidit cerebrospinal: A) B) C) D) E) Hipoproteinorraki, pleiocitoz neutrofile, glikorraki normale Hiperproteinorraki, pleiocitoz limfocitare, hipoglikorraki Hiperproteionorraki, pleiocitoz nutrofile, hipoglikorraki Proteinorraki n norm, munges t pleiocitozs, hipoglikorraki Hiperproteinorraki, pleiocitoz limfocitare, glikorraki normale

2 2. Nj i ri toksikodipendent (per i.v) ka shfaqur prej 4 muaj sh hepatitin akut dhe koinfeksionin HBV+HDV. Mj keu i familjes i keshillon t kryej disa analiza laboratorike pr t vlersuar gj endjen e smundj es. Rezultatet jan: transaminazat normal; antitrupat anti HDV pozitiv; HBV-ADN plazmatike negative; antitrupat anti HBs pozitive; antitrupat anti HBe pozitive. Mbi bazn e ktyre t dhnave mjeku i thot t riut se sht shruar. Mbi cilat t dhna bazohet mj eku pr t dhn kt prgj igj e, se ai sht shruar: A) B) C) D) E) Transaminazat n norm Antitrupat anti HDV pozitiv HBV-ADN plazmatike negative Antitrupat anti HBs pozitive Antitrupat anti HBe pozitive

2 3. Cilat jan alternimet laboratorike q vihen re gjat mononukleozs infektive? A) B) C) D) E) Leuokopeni dhe piastrinopeni Leukocitoz neutrofile dhe rritje t transaminazave Leukocitoz limfocite dhe rritje t transaminazave Leukopeni dhe rritje t bilirubins Anemi dhe piastrinopeni

2 4. Mbledhja e sekrecioneve pr ekzaminimin n kultur mund t kryhet:

A) B) C) D) E)

Vetm n mngj es, edhe pse pacienti nuk ka koll Vetm n mngj es kur je pa ngrn Preferohet nse pacienti ka qndruar gjat i shtrir Cdo moment t dits, mj afton q pacienti t ket koll me sekrecione Disa her gj at dits n mo mentet kur ka pikun e temperaturs

2 5. Vdekja e zez (peste bubbonica) transmetohet nga: A) B) C) D) E) Miza Pleshti Mushkonja Morri Grenxa

2 6. Nj person q sht shpuar me agen q ka prdorur nj pacient me HCV-ARN pozitiv: A) B) C) D) E) I inj ektohen imunoglobulina standarte Aktualisht nuk ekziston asnj profilaksi pas ekspozimit I injektohen imunoglobulina standarte nse viremia sht >400.000/ml T gj itha prgjigjet Asnjra prej prgj igjeve

2 7. Shfaqja m e shpesht e infeksionit t par nga herpes simlex tip 1 sht: A) B) C) D) E) Herpes labial Gengivo-stomatit E zofagit Hepatit Encefalit

2 8. Kush ka rrezikun m t lart pr tu infektuar me bacilin e tuberkulozit? A) Nj erz q jan n kontakt t ngusht me persona t prekur nga tuberkulozi e q j an n faz aktive B) Persona q kan lindur n vende ku TB sht mjaft e prhapur si n Azi, Afrikn Jugore, Europn lindore C) T moshuarit D) Personat e varfr, pa prkuj desje mj eksore si t pastreht, toksikodipendentt, persona q jetojn n azile, personeli mjeksor i ekspozuar ndaj TB E) T gj itha prgjigj et

2 9. Rast klinik: Pacienti ka kto karakteristika n ekzaminimin e likidit cerebrospinal: pamje t qart; qeliza 600 mmc (N=60% L=40%) glikorraki t ulur; proteinorraki t rritur. Nga se mund t jet prekur ky pacient? A) B) C) D) E) Meningiti meningocoxik Meningiti viral Meningiti tuberkular n zhvillim Meningiti nga E. Coli Asnjra prej prgj igjeve

3 0. Sepsi karakterizohet nga: A) B) C) D) E) Prania n gjak e grimcave virale difektive Prania n gjak e antigjeneve bakteriale Alternimi i termorregullimit n nivelin qndror Prania n gj ak e baktereve n proces shumimi aktiv Mungesa e nj prgjigje inflamatore sistemike

3 1. Virusi Ebola ka shkaktuar epidemi n A) B) C) D) E) Europn veriore Europn lindore Azi Afrik Amerikn e jugut

3 2. Cfar shenj ash paraqet nj pacient i prekur nga botulizmi? A) B) C) D) E) Shenja neurologj ike me paraliz Shtim t pshtyms Crregullime sensitive Diarre persistente Shenja t nj polmoniti

3 3. Cila prej smundj eve shkaktohet nga chlamidie? A) B) C) D) Smundja e Weil Temperatura Q Myku Trakoma

E) Sporotrikoza

3 4. Cili prej viruseve sht shkaku m i mundshm pr Sarkomn e Kaposit? A) B) C) D) E) Virusi T-limfotrop Virus coxackie Herpesvirusi human 8 Citomegalovirus E cho-virus

3 5. Cili sht trajtimi i dizanteris nga ameba: A) B) C) D) E) Cefalosporina V anco micina M etronidazoli A lbendazoli Fluorokinoloni

3 6. N meningj it, cefalea sht shenj e: A) B) C) D) E) Hipertensionit endokranial Irritim i radiksit spinal H iperes tezi T gj itha prgjigjet Asnjra nga prgjigj et

3 7. Tek nj pacient t prekur nga kolera, ndrhyrja e par terapeutike bazohet n: A) B) C) D) E) Dhnien e antibiotikve Dhnien e dizinfektantve intestinal Pastrimin me an t kriprave Rehidrimin e pacientit me solucione elektrolitike dhe ruajtjen e hidratimit. Edhe pse nuk ka t vj ella pacienti duhet mbajtur pa ngrn

3 8. Polmoniti lobar shkaktohet n prgjithsi nga: A) B) C) D) Stafilococcu E nterococcu P neum ococcu Virusi respirator sincicial

E) Mycoplasma pneumonie

3 9. Pacienti me temperatur, eritem migruese, cefale dhe dhimbje artikulare me shum mundsi sht prekur nga: A) B) C) D) E) Reumatizmi artikular akut Ricketsioza Smundja e Lyme Fruthi Lija e dhenve

4 0. Cili ushqim shkakton m lehtsisht intoksikim nga Stafilococcu aureus? A) B) C) D) E) Kremi i pastieris Mishi i zier Frutat e freskta Fruta t thata Uji i ndotur

4 1. Meningiti sht proces inflamator: A) B) C) D) E) Prfshin parenkimn cerebrale Prfshin durn Prfshin leptomeninget Prfshin vetm bazn e kranit Prfshin vetm pjesn e siprme t kranit

4 2. Pr cilin prej hepatiteve ekziston vaksin? A) B) C) D) E) Hepatiti A dhe C Hepatiti C Hepatiti D Hepatiti A dhe B Hepatiti G, A dhe B

4 3. Strategj ia Dots (directly observed therapy) pr TBC prfshin: A) Pacienti ndiqet n mnyr konstante nga personeli, prgj at 6-8 muaj ve t terapis, pr prdorimin e rregullt t terapis, t efekteve, efikasitetit t tyre dhe ecurin e smundj es

B) C) D) E)

Pacienti merr mj ekimin nn vzhgi min e familjarve Pacienti v re efektet ansore Pacienti vet-kurohet Asnjra prej prgj igjeve

4 4. Temperatur 40 celsius, konxhuktivit, fotofobi, rash makulo-papular eritematoz (nga fytyra n trup, n ekstremitete, duar e kemb). Ky kuadr klinik ju bn t mendoni pr: A) B) C) D) E) Lija e dhenve Fruthi Rubeola Skarlatina Mononukleoza

4 5. Tek bruceloza, temperatura: A) B) C) D) E) Mungon Esht onduluese Esht periodike Rritet me shkall gjat javs s par t smundj es Kap maksimumin n ditn e tret t smundjes

4 6. N prani t temperaturs, takikardis, hipotensionit, takipnes dhe leukopenis mund t mendoj m pr: A) B) C) D) E) Sepsi nga Gram pozitiv L eptospiroza Sepsi nga Gram negativ T gj itha prgjigjet Asnjra prej prgj igjeve

4 7. Forma m e rnd e malaries sht ajo q shkaktohet nga: A) B) C) D) E) Plasmodium malaria Plasmodium falciparum Plasmodium oval Plasmodium vivax Plasmodium oval tek nj person q ka vuajtur dhe m par nga P. Malaria

4 8. Pr ciln nga patologj it sugjerohet terapi me antibiotik?

A) B) C) D) E)

Mononukleoza infektive Hepatiti B akut B ruceloza Ethet e verdha Ethet tropikale

4 9. Malaria sht e prhapur n: A) B) C) D) E) Vendet e Mesdheut Vetm n vende bregdetare Zona n lartsin mbi 1800 m Vetm n disa zona afrikane N zona tropikale dhe subtropikale

5 0. Nuk duhet tu bhet vaksinimi me MMR (fruth, parotid, rubeol): A) Fmij t me reaksion nga xhealtina dhe neomicina, fmij t me alterime t sistemit imunitar (tumore, AIDS) ose q marrin mj ekime imunosupresive ose radioterapi B) Fmij t me smundje febrile akute C) Grat gjat shtatzanis D) Personat q i nnshtrohen transfuzionit ose trajtimeve hemoderivuese E) T gj itha prgjigj et

5 1. Periudha dritare pr HIV-in sht: A) Periudha n t ciln j an t evidentueshm anti- HIV por viremia nuk sht ende e pranishme B) Periudha n t ciln nuk sht i mundur transmetimi i smundjes C) Periudha n t ciln sht i pranishm HIV-ARN por nuk j an shfaqur ende antitrupat D) T gj itha prgjigjet E) Asnjra prej prgj igjeve

5 2. Pr nj pacient dyshohet t ket tifon abdominale. Pr t konfirmuar kt diagnoz krkoj m: A) B) C) D) E) Reaksionin e Widal-it Sierodiagnozn e Wright Antitrupat jo t plot Reaksionin e Wasserman-it Reaksionin e Paul Bunnel

5 3. Smundjet me origjin virale, n shumicn e rasteve shoqrohen me: A) B) C) D) E) Leukocitoz neutrofile Leukocitoz limfocite Leukocitoz bazofile Lim focitoz Trombocitoz

5 4. Hemokultura konsiston n kulturn e gj akut pr t krkuar agj entt patogjen q shkaktoj n smundje q prhapen (si sepsi, endokarditi etj.): A) B) C) D) Esht i mj aftueshm marrja e nj kampioni gj aku kur pacienti ka temperatur Esht i mj aftuesh m marrja e kampionit t gj akut n mnyr rastsore Marrja e gj akut bhet vetm hert n mngjes dhe pa ngrn Esht e domosdoshme marrja e prsritur e disa kampionve t gj akut gj at 24 orve, pasi prania e baktereve n gjak nuk sht e prhershme E) Shpeshtsia e marrj es s kampionve t gj akut varet dhe nga agjenti patogj en pr t cilin dyshoj m

5 5. Cila patologji e lkurs ka rrezik m t lart pr t zhvilluar herpex simlex t rnd: A) B) C) D) E) Dermatiti atopik Psoriaza Ekzema nga kontakti Dermatiti Lytha

5 6. Cila nga alternativat rreth toxoplazms nuk sht e vrtet: A) B) C) D) E) Macja sht mbartsi i ndrmjtm dhe nj eriu ai prfundimtar Macja sht mbartsi prfundimtar Mund t ket encefalit tek pacientt imunodepres N format intrauterine ka shpesh prfshirje t trurit Shpesh sht asimptomatike

5 7. Cfar sht influenca aviare? A) Smundje e kafshve shkaktuar nga virust aviar, q normalisht infektoj n vetm zogj t

B) C) D) E)

Transmetohet nga insektet Esht smudje pandemike Trasmetohet nga flebotomi Esht smundje bakterial

5 8. Cili prej agj entve patogjen sht shkaktari m i shpesht i polmonitit tek t moshuari? A) B) C) D) E) Mycoplasma pneumonie Chlamydia pneumoniae Streptococco pneumonie Klebsiella pneumonie T gj itha prgjigj et

5 9. Si do t ishte m e sakt t veprohej kur duam t vaksinojm kundr rubeols nj grua n mosh fertile? A) Injektim t vaksins n mes t ciklit B) Inj ektim t vaksins n ditn 24-26 t ciklit C) Inj ektim t vaksins gjat ciklit mestrual dhe dhnia e kontraceptivve pr tre muaj me rradh D) Injektim i vaksins gjat do faze t ciklit dhe dhnia e kontraceptivve pr gjasht muaj me rradh E) Injektim t vaksins n fund t ciklit

6 0. Shfaqja kryesore klinike e brucelozs akute e sub-akute sht: A) B) C) D) E) Temperatura onduluese Temperatura me stade Temperatura anfibolike (me luhatje) Temperatura kuintane (e pes ditve) Temperatura terciane (e tre ditve)

6 1. Si quhen kushtet n t cilat ndalohet kalimi i mi krorganizmave n ambiente sterile? A) B) C) D) E) Antisepsi Asepsi B akte riostaz Zona filtr Bonifikimi i ambientit

6 2. Cili prej infeksioneve nuk mund t prkufizohet si nj infeksion oportunist tek pacienti me AIDS? A) B) C) D) E) Meningiti criptocoxik Pneumocistoza pulmonare Toxoplazmoza cerebrale Retokoliti uleroz Korioretiniti citomegalik

6 3. Cila sht simptoma m e shpesht e mononukleozs infektive: A) B) C) D) E) A noreksi Faringodini Mialgj i Nauze Koll

6 4. Cilat jan zonat q prek m shpesh smundja e Lyme? A) B) C) D) E) Sistemi nervor qndror Aparati muskulo-skeletrik Lkura T gj itha prgjigjet Asnjra prej prgj igjeve

6 5. Nj fmij 6 muaj sh, pas 3 ditsh me temperatur t lart shfaq ekzantem t kuqrremt n t gjith lkurn dhe ndrkoh temperatura ulet n mnyr t menjhershme. Nga se mund t jet prekur ky fmij? A) B) C) D) E) Fruthi Ekzantema kritike Rubeola Toxoplazmoza kongenitale Infeksion kongenital nga citomegalovirus

6 6. Profilaksia e rregullt kundr malarias me klorokin jepet: A) Niset trajtimi n momentin q mbrrihet n zonn malarike dhe prfundohet n fund t qndrimit B) Niset trajtimi nj jav para se t shkohet n zonn malarike dhe nrpritet n fund t qndrimit

C) Niset trajtimi nj j av para se t shkohet n zonn malarike dhe vazhdohet deri n dy jav pas kthimit D) Niset trajtimi dy j av para se t shkohet n zonn malarike dhe va zhdohet deri n 46 jav pas kthimit E) Asnjra prej prgj igjeve

6 7. N nj person t imunizuar kundrejt hepatitit B, infeksioni nga virusi delta (HDV) shfaqet: A) N 10% t rasteve B) M pak se 10% t rasteve C) Midis 10-50% t rasteve D) M shum se 50% t rasteve E) A snjher

6 8. Rast klinik: Rreth 5 or pas nj dasme, disa prej t ftuarve shfaqn dhimbje abdominale, t vj ella, diarre. Cila mund t jet diagnoza? A) B) C) D) E) Gastroenterit viral Enterit nga campylobacter jej umi Intoksikim nga stafilococcus aureus Infeksion nga salmonela Amebiaz

6 9. Cilin ekzaminim do t kryenit nse dyshoni pr tenian? A) B) C) D) E) U rinokultur Tampon rektal K oproku ltur Analiz urine Analiz parazitologjike t feeve

7 0. Cila nuk sht ndr simptomat e shpeshta t infeksionit primar nga HIV? A) B) C) D) E) Te mperatur Fa ringodini Artralgj i Rash kutan makulo-papular Alopeci

7 1. Patogj eneza e shfaqjes n lkur t smudjeve infektive sht e lidhur me:

A) B) C) D) E)

Shumimin e patogj enit n nivelin e lkurs Fenomene imunologjike Lshimi i toksinave Fenomene vaskulare T gj itha prgjigj et

7 2. Sterilizim do t thot: A) B) C) D) E) Shkatrrim t agjentve patogj en Shkatrrim t agjentve patogjen e jo patogjen Shkatrrim t agjentve jo patogj en Shkatrrim vetm t agjentve bakterial Shkatrrim t gjithckaje q mbart nj agjent mikrobial

7 3. N aspektin epidemiologj ik, hepatiti A lidhet m shum me: A) B) C) D) E) Marrdhniet heteroseksuale jo t mbrojtura Prdorimi i hemoderivuesve Prdorimi i ujit ose frutave t detit t kontaminuara Shkmbimi i lugve q jan prdorur nga persona t smur me HAV Kontakti i ngusht respirator me persona t smur

7 4. Manifestimi m i shpesht tek nj i rritur imunokompetent sht: A) B) C) D) E) Retiniti Hepatiti akut Hepatiti kronik Li m fo aden iti Encefaliti

7 5. Leukopenia sht e shpesht n rastin e: A) Kolera B) Enteriti nga Escherichia coli enterohemorragjik C) S higeloza D) Ethet tifoide E) L eptospiroza

7 6. Orkiti sht komplikim i munshm i njrs prej smundj eve infektive. Cils prej tyre?

A) B) C) D) E)

Rubeola Fruthi Lija e dhenve Parotiti (shytat) Ekzantema kritike

7 7. T sapolindurit nga nna t infekstuara me HIV jan: A) B) C) D) E) Gjithnj HIV-Ab pozitiv Gjithnj HIV-Ab negativ Gjithnj HIV-ARN negativ Gjithnj HIV-ARN pozitiv HIV-Ab pozitiv n 50% t rasteve

7 8. Cili prej ktyre agj entve shkakton m shpesh hepatit fulminant? A) B) C) D) E) Virusi i fruthit Pneumocysti carini HBV HIV Leishmania donovani

7 9. Cili prej virusve nuk shkakton sindromn e mononukleozs? A) Virusi i rubeols B) Virusi i Epstein-Barr C) Herpes virus human tipi 6 (HHV6) D) HIV E) K oronavirus

8 0. Hepatiti nga virusi delta verifikohet vetm n rast se ekziston nj kohsisht dhe infeksioni nga: A) B) C) D) E) HAV HBV Virusi Epstein-Barr HCV Citomegalovirus

8 1. Cila struktur preket m shpesh ngainfeksioni i citomegalovirusit? A) B) C) D) E) Gjndrat e pshtyms Gjndrat e pshtyms dhe veshkat Pulm oni Pulmoni dhe hepari Hepari

8 2. Terapia pr meningitin tuberkular prfshin 4 medikamente antituberkualre. Cili prej ktyre medikamenteve nuk prfshihet? A) B) C) D) E) Claritromicina I zoniazidi Pirazina m idi Eta m butoli Rifa m picina

8 3. Pneumonit e komunitetit jan: A) B) C) D) E) Merren pas nj episodi gripi (influence) Merren n komunitetet e toksikodipendentve Merren n spital Merren jasht spitali, n jetn e prditshme Tek t sapolindurit

8 4. Chikungunya e nj ohur dhe si smundja e njeriut t prkulur transmetohet nga: A) B) C) D) E) Pickimi i mushkonj s s infektuar, Aedes Albopictus dhe Aedes Aegypti Nga insektet Kontakti seksual U shqimi Inhalimi

8 5. Cili sht agjenti etiologjik i etheve Q? A) B) C) D) E) Clamidia Legionela St reptococc u H.influenca Coxiela burneti

8 6. Komplikimet kryesore q shkaktohen nga virusi aviar (shpendve) tek njeriu jan: A) B) C) D) E) Mioziti Pneumonia baketriale sekondare Sindroma e Reye sbashku me terapin me ASA M iokarditi T gj itha prgjigj et

8 7. Kondilomat (lythat) shkaktohen nga: A) B) C) D) E) HPV HSV Enterovirus HIV Baktere

8 8. Vaksinimi kundr kolers j ep: A) B) C) D) E) Imunitet t prhershm Imunitet pr 5 vjet Imunitet pr 10 vj et Imunitet pr 18 muaj Imunitet pr 6 muaj

8 9. Cili prej markuesve t gj akut mundson diagnozn etiologjike pr hepatitin akut tip B? A) B) C) D) E) HBsAg IgG antiHBc A ntiH B c IgM anti HBc HBeAg

9 0. Streptocoket viridant shkaktoj n m shpesh: A) B) C) D) E) Meningit Endokardit Cistopielit Kolecistit Enterit

9 1. Medikamentet parsore pr traj timin e brucelozs jan: A) B) C) D) E) T etraciklina Antibiotikt glikopeptidik Cefalosporina t gjenerats s II Cefalosporina t gj enerats s III Metronidazoli

9 2. Cila prej smundj eve nuk transmetohet nprmjet rrugve seksuale: A) B) C) D) E) Limfoma e Lennert HBV Limfogranuloma venere Ulera e but venere Herpes simplex tip 2

9 3. Shnja e Koplik sht e pranishme n: A) B) C) D) E) Lija e dhenve Fruthi Rubeola Skarlatina Tifo

9 4. Endokarditi infektiv tek toksikodipendentt q marrin drog nga vena shkaktohet m shum nga: A) B) C) D) E) Stafilococu aureus Streptococu viridans Escherichia coli Enterococ u Gonococu

9 5. T gjitha gjendjet e mposhtme mund t j apin fals pozitiv pr reaksionin ndaj sifilizit, prve njrs: A) B) C) D) E) Mononukleoza infektive Tripanozomiaza Lupus eritematoz sistemi k Malaria Bruceloza

9 6. Erisipela shkaktohet nga: A) B) C) D) E) Streptococu piogenes Stafilococu aureus Agj enti nuk sht identifikuar ende N shumicn e rasteve izolohet agjent i ngjashm e prionet Proteus vulgaris

9 7. Cili prej alternimeve t likidit cerebro-spinal diferencon meningitin bakterial nga ai viral? A) B) C) D) E) H iperproteinorraki Pleiocitoz likidi H ipoglikorraki Prevalenc limfocitare Rritje t presionit n likid

9 8. Flora bakteriale normale e pranishme n organizm: A) Esht e rndsishme, sepse ka funksion mbroj ts B) Prania e saj tregon se personi sht i smur C) Esht e pranishme vetm tek personat imunodepres (AIDS, neoplazi, trapiante etj.) D) Esht e pranishme vetm tek personat q jan shruar nga nj smundje infektive E) Esht e pranishme vetm tek personat me s mundje infektive kronike

9 9. Nj person i vaksinuar pr hepatitin B sht: A) B) C) D) E) HBsAg pozitiv AntiHBe pozitiv AntiHBe dhe Anti HBc pozitiv AntiHBe, antiHBc dhe anti HBs pozitiv Anti HBs pozitiv

100. Cestodet : A) Nuk jan patogjen pr njeriun B) Jan protozoar

C) Jan krimba cilindrik D) Jan krimba t shesht t segmentuar E) Jan krimba t shesht jo t segmentuar

101. Cila prej vaksinave prmban virus t gjall t dobsuar? A) B) C) D) E) Anti hepatiti A A ntidifteriku Antipolio tip Salk Anti ethet e verdha Antitifo

102. Perforimi intestinal sht pjes e komplikimeve t: A) B) C) D) E) Kolitit pseudomembranoz Ethet tifoide Kolera Enteriti nga Campylobacter jej unum S higeloza

103. Imuniteti pasiv gjat nj infeksioni sht ai i fituar nga: A) B) C) D) Mamaj a Pasi e ka hasur infeksionin por pa klinik Pasi e hasur infeksionin dhe m pas sht shruar Pasi e ka hasur infeksionin dhe m pas sht shruar, pavarsisht nga kuadri klinik E) Pasi sht vaksinuar pr kt infeksion

104. Nj e re 25 vj eare shfaqi temperatur, faringodini dhe asteni. Pas 24 orve ku temperatura rritej, ajo vuri re pranin e disa limfonodujve n qaf. K onsultohet me mj ekun dhe vihet re: limfonoduj t zmadhuar n t gj itha stacionet limfatike siprfasore, subikter t sklerave, skuqje t faringut dhe disa pllaka t bardha tek tonsilat. Nga analiza e gj akut vihet re: Hb 13.5g/dl, eritrocite 4.260.000/mmc, leukocite 25.000/mmc me formul: N 21%; E 2,5%; B 0.5%; L 39%; M 28%; qeliza t mdhaja limfocitare 9%. Trombocitet 350.000/mmc; AST 250UI/L; ALT 900UI/L; Bilirubin tot 3.5mg/dl; Bilirubin direkt 1.8mg/dl. Cili mund t jet agjenti patogjen m i mundshm pr kte gjendj e: A) Virusi i Epstein-Barr B) Streptococu gr A

C) Mycoplazma hominis D) Neisseria gonorrhaea E) Escherichia coli

105. Profilaksia ndaj meningitit meningokoksik pr ata q jan kontakt t ngusht me t bhet me: A) B) C) D) E) Gamaglobulina Rifa mpicin e Peniciline Ertro micine Cloramfenikol

106. Spiroketet e llojit Borrelia mund t shkaktoj n: A) B) C) D) E) Stomatit Temperaturn periodike Tifon murine Gripin Gastritin

107. Cila sht terapia pr Morbusin e Hansen, n formn tuberkulare: A) B) C) D) E) Dapsoni Rifa mpicin a Dapsoni+Rifampicina Clofazim ina Claritro m icina

108. Skarlatina shkaktohet nga: A) B) C) D) E) Streptokoku i gr A Stafilokoku Treponema pallidum Nuk njihet etiologjia Pneumokoku

109. Idatidoza shkaktohet nga: A) Echinokoku

B) C) D) E)

Tenia Hymonelepis nana L eptospira Agj ent t ngj ashm me prionet

110. N botulizmin nga ushqimi, smundja shkaktohet nga: A) B) C) D) E) Toksina e prodhuar n zorr Toksina e formuar paraprakisht tek ushqimi Nga produktet q lirohen gj at lizs s bakterit n stomak Bakteri n faz replikuese Toksina citotonike

111. N infeksionin nga HIV, niveli i viremis sht i lart dhe keshtu q rreziku pr transmetim i smundj es sht m i madh: A) B) C) D) E) Kur infeksioni merret me rrug seksuale N 24 ort e para pas kontaktit Gjat fazs s latncs klinike Si gj at fazs akute dhe m von, ku niveli i CD4 sht shum i ult Pas trajtimit me zidovudine, lamivudine dhe ndinavir

112. Nj pensionist 76 vj ear, ish minator, me silikoz pulmonare, pas 2 ditsh me temperatur t lart e otalgji t djatht, me delir e i hutuar bie n koma. N ekza minimin objektiv evidentohen shenja meningeale,herpes labial dhe me mbrana timpanike e djatht e turbullt. N analizat e gjakut vihet re leukocitoz neutrofile. Analiza e likidit cerebro-spinal tregon pr pleiocitoz neutrofile, hipiglikorraki, hiperprotidorraki. Cili mund t jet agjenti patogj en m i mundshm? A) B) C) D) E) Meningit nga gram negativt Meningit meningokoksik Meningit pneumokoksik Meningit tuberkular Meningit nga echo virus

113. Epidemit q prhapen npr kontinente quhen: A) B) C) D) Universale Pandemike E ndem ike Sporadike

E) Asnjra prej prgj igjeve

114. Tek pacienti me AIDS, trimetoprim/sulfametazolo (cotrimosazolo) prdoret n terapin e: A) B) C) D) E) Aspergilozs s prhapur Tuberkulozit extrapulmonar Pneumonis nga P. Carinii Meningiti nga criptokoku Retiniti nga citomegalovirus

115. Cila prej alternativave nuk sht shenj e amebiazs intestinale nga E. Histolytica? A) B) C) D) E) Fee me gjak Leukocitoz n gj akun periferik Prania e mukusit n fee Dhimbje abdominale Temperatur modeste ose q nuk ka fare

116. Kuadri hematologjik tek Leshmanioza karakterizohet nga: A) B) C) D) E) Rritje t neutrofileve Anemi mikrocitike Leukopeni Rritje t trombociteve L euko citoz

Prgjigja e sakt shtC 117. Trizmusi (trisma) tek tetanozi sht: A) B) C) D) E) Hiperton i muskuj ve dorsal Spazma t sfintereve Relaksim muskular t gj eneralizuar Hiperton t muskuj ve maseter Hiperton t muskuj ve sternokleidomastoide

118. Cila mund t trasmetoj smundjen nga mbartsi i tifos i shruar: A) Pshtyma B) Gjaku

C) Lott D) Djersa E) Feet

119. Infeksioni nga HPV sht: A) I gj ithpranishm dhe prek t paktn nj her n jet 75% t popullsis nga 15-49 vj e B) Transmetohet me rrug seksuale C) Esht prgjegj s pr t paktn 1/3 e smundjeve seksualisht t tras m etueshme D) Prgj egj s pr t paktn 500.000 raste t reja t karcinoms cervikale, do vit n bot E) T gj itha prgjigj et

120. Si transmetohet Giardiasis? A) B) C) D) E) Prdorimit t ushqimit t gatuar Prdorimit t ujit t ndotur Me an t lkurs Me rrugt respiratore Asnjra prej prgj igjeve

121. Cila prej smundjeve infektive nuk trasnmetohet nga kontakti direkt me t smurin? A) B) C) D) E) Lija e dhenve HAV Ethet tifoide B ruceloza P olio m eliti

122. Gastroenteriti i shkaktuar nga rotavirus sht i prhapur n t gjith botn. Cila sht rruga kryesore e transmetimit t virusit? A) B) C) D) E) Oro-fekale R espiratore H ematike Lim fatike Kontakti me siprfaqe t kontaminuara

123. Nj grua e re do t shkoj n Afrikn qndrore pr turizm. Profilaksia kundr malarias me meflokine: A) Nuk duhet br pasi efikasiteti i saj sht i diskutueshm B) Bhet vetm nse do t shkoj n vende endemike pr P. Vivax C) Duhet br nj dit para se t niset dhe duhet prfunduar nj jav para se t kthehet D) Duhet br nj jav para se t niset, t vazhdoj gjat gjith qndrimit n at vend dhe t prfundohet 4 jav pas kthimit n vendin e saj E) Nuk bhet tek grat shtatzna, pasi sht treguar se rreziku pr tu pickuar nga mushkonja Anopheles sht mj aft i vogl pr kt gj endje

124. Leishamania sht: A) B) C) D) E) Elmint (krimb) Virus me ARN Bakter gram + Protozoar Micetet (krpudh)

125. Simptoma ose shenja pothuajse konstante gj at smundjes s rubeols sht: A) B) C) D) E) Kolla Zmadhimi i limfonodujve retronukal Temperatur>40 celsius H opistoton Diarre sekretore me gjakosj e

126. N ciln shtat ditsh t etheve tifoide hemokultura sht m shpesh pozitive? A) B) C) D) E) Shtat ditshi i I Shtat ditshi i II Shtat ditshi i III Shtat ditshi i IV Shtat ditshi i V

127. Profilaksia e pneumonis nga P. Carinii tek personat me HIV bhet me: A) M eflokine B) C otrim osazol C) Amfotericina B

D) Amoxicilina dhe ac. Clavulanic E) Fluconazol

REUMATOLOGJI 1. Artriti tuberkular sht tipik: A) B) C) D) E) P oliartikular M onoartikular Me zhvillim hiperostozues Nuk duket radiologj ikisht Nuk bn abses

2. Cila prej alternativave nuk sht e vrtet rreth artritit psoriazik: A) >70% e pacientve j an t prekur nga psoriaza e lkurs shum vite prpara se t zhvillohet artriti B) N pak raste prek artikualcionin sakro-iliak C) Kolona vertebrale mund t shfaq entezit q ngj ason me artritin D) Bn pjes tek Spondiloartropatit siero-negative E) Terapia me metotresat, drejtuar dmit artikular, prmirson gj ithnj dhe lezionet e lkurs

3. Simptoma m karakteristike e smundjes s Horton sht: A) B) C) D) E) Nuk ndjehet mir Cefale intense, parosistike Dhimbje artikulare A m bliodipi Marrje mendsh

4. N artritin reumatoid cilat jan artikulacionet q preken m shpesh? A) B) C) D) E) Artikulacionet e vogla t duarve Tibio-tarsike T bainit Brrylit Tem poro-m andibulare

5. Mbi bazn e cilve prej elementve laboratorik, spondiloartriti konsiderohet si

sie r o n e g a ti v ? A) Faktori reumatoid negativ B) Indekset aspecifike t inflamacionit negativ C) ANCA negativ

D) ANA negativ E) Mungesa e faktorit reumatoid dhe ANA negativ

6. Cili prej antitrupave antinukleojan tipik pr diagnozn e Lupus Eritematoz Sistemik A) B) C) D) E) A ntiperinuklear Anti-ADN me dopio helik Anti-ADN me helik teke Anti mukoza gastrike Anti RNP

7. N artritin reumatoid, prve artikulacione, cili organ n prgjithsi nuk sht i prfshir? A) B) C) D) E) Lkura P ulmonet Zemra Sistemi nervor Hepari

8. Faktori reumatoid sht: A) B) C) D) E) Imunoglobulin IgM drejtuar drejt porcionit Fc t IgG Autoantitrup anti-brtham (ANA) Enzim e indit sinovial Protein e fazs akute Asnjra prej prgj igjeve

9. Cili sht kuadri radiologjik tipik i artritit reumatoid? A) B) C) D) E) Erozione t epifizs Mikrofraktura iuxtartikulare Osteoskleroz subkondriale Geoide diafizare Osteofite

1 0. Lumbalgj ia ose dorsalgjia nokturne, q zgj on personin n ort e vona t nats, t orienton drejt diagnozs s:

A) B) C) D) E)

Skolioza O steoartroza S pondilolisteza Spontiliti ankilozant Artriti reumatoid

1 1. Pozitiviteti pr ANCA (antitrupa anti-citoplazm t neutrofileve) sht markues diagnostik pr: A) B) C) D) E) Sarkoidoz Vaskulit sistemik Artrit reumatoid Smundja mikse e indit lidhor (MCT D) Lupus Eritemaroz Sistemik

1 2. Cila prej smundj eve artikulare mund t prek traktin cervikal t kolons vertebrale? A) B) C) D) E) Artriti reumatoid Sindroma e Reiter Sindroma fibromialgjike O steoporoza Polimialgjia reumatike

1 3. Antitrupat anti-nukl (ANA) krkohen kur dyshohet pr? A) B) C) D) E) Artrit reumatoid O steoartroza Sarkoidoza K onektiviti Tiroiditi autoimun

1 4. Cila prej simptomave t smundjes reumatike haset m rrall: A) B) C) D) E) Pleuriti reumatik Endokarditi reumatik Miokarditi reumatik Te mperatur Takikardi persistuese

1 5. Kuadri klinik me dhimbje artikulare, t paktn tre artikulacione t enjtura, ngurtsi n mngj es >30 min, pozitivitet pr faktorin reumatoid ose antitrupat anti-CCP t bn t mendosh pr: A) Artrit gotoz B) S pondiloartrit C) Artrit reumatoid D) Polimialgj i E) O steoartroz

1 6. Cila nga zonat sht tipike q preket nga frakturat e shkaktuara nga osteoporoza e tipit II? A) B) C) D) E) Vertebrat lumbare Vertebrat dorsale Radiusi Vertebrat cervikale Qafa e femurit

1 7. Tek Lupus Eritemaroz Sistemik vrehet m shpesh: A) B) C) D) E) Leukocitoz nga rritja e limfociteve qarkulluese (limfocitoz) L eukopeni Po li gl ob uli Tromb ocitoz T gj itha prgjigj et

1 8. Sindroma e Sj ogren: A) B) C) D) E) Esht m e shpesht tek femrat e reja Esht patlogji pediatrike Shfaqet kryesisht tek femrat n mosh t mes me Nuk shoqrohet asnjher me artritin reumatoid Mund t shkaktoj shtim t lotve

1 9. Cila prej smundj eve mund t konsiderohet si artrit reaktiv sieronegativ? A) B) C) D) E) M. Crohn Gotta (podagra) Diabeti Hepatiti viral M. Paget

2 0. Tek artriti reumatoid, dmtimi artikular fillon nga: A) B) C) D) E) Membrana sinovjale Kartilagjeni artikular Kocka nnkondriale Asnjra prej prgjigjeve Kpsula artikulare

2 1. Cila prej karakteristikave klinike nuk ndodh tek fibromialgj ia? A) B) C) D) E) Cefalea e tipit tensiv Vaskuliti E m ikrania Vshtirsit konj itive Sindroma e kolonit t irrituar

2 2. Cfar sht piku i mass kockore? A) B) C) D) Vlera e densitetit mineral kockor para menopauzs Vlera e densitetit mineral kockor e harritur n moshn 25 vjeare Vlera minimale e densitetit mineral kockor e gj etur n rastin e nj frakture Vlera maksimale e densitetit mineral kockor e gjetur gjat terapis pr o ste o p oro z n E) Vlera mesatare e densitetit mineral kockor e gjetur gjat densimetris me rreze X

2 3. Alternimi karakteristik i ezofagut n prani t sklerozs sistemike, sht: A) B) C) D) E) Devijimi i saj n disa drej time Formimi i divertikuj ve nga trheqja Formimi i divertikuj ve nga pulsimi Hipotoni dhe ngurtsi t pareteve dhe zgjerim t saj Zbrazje e vonuar prej spazmave kardiale

2 4. Smundja reumatike shkaktohet nga: A) Streptokoku viridans B) Streptokoku beta-hemolitik i gr A C) Pneumokoku

D) Stafilokoku E) Enterokoku

2 5. Cila prej alternativave nuk sht karakteristike e spondilitit ankilozant? A) B) C) D) E) Dhimbje t kolons, q prmirsohen gj at lvizj es Dhimbje akute t orbits, unilaterale Lezione papulo-skua moze n pllmbn e duarve dhe at plantare Fillim i ngadalt i simptomave Oligoartropati periferike asimetrike

2 6. Lezioni inflamator kronik shkatrrues q prek krcin nazal sht tipike e: A) B) C) D) E) M. Wegener M . Behget Lupus eritematoz sistemi k Morphea Sklerodermi difuze

2 7. Cila sht shfaqje tipike n artritin psoriazik? A) B) C) D) E) Perikarditi Luksacioni i radiusit Shfaqja e dhimbj es artikulare pas sforcos fizike Datiliti Fenomeni Raynaud

2 8. Cila patologji sht prgj egj se pr prfshirjen poliartikulare nga shkaqe in fla m ato re ? A) B) C) D) E) O steoartroza Hemokromatoza A m iloidoza Ethet reumatike Hemofilia

2 9. Arteriti me qeliza gjigjante (arteriti temporal ose i Horton) sht smundje e rnd pasi: A) Provokon lehtsisht fraktura kockore

B) C) D) E)

Provokon humbje t shikimit n mnyr t menj hershme Nuk ka mundsi kurimi Provokon lehtsisht insufienc renale Lehtson zhvillimin e neoplazive

3 0. Antigj eni HL-A B27 sht i pranishm n 85% t pacientve t prekur nga: A) B) C) D) E) Artriti reumatoid Spondiliti ankilozant Gotta Artriti psoriazik Ethet akute reumatike

3 1. Cila prej gj endj eve t mposhtme shoqrohet me pranin e antitrupave antinukel: A) Lupusi sistemik B) Sindroma nga antitrupa antifosfolipide primitive C) Poliarteriti nodoz D) Sindroma nga lodhja kronike E) Fibro m ialgji

3 2. Cila prej smundj eve artikulare nuk sht me baz imunologjike? A) B) C) D) E) Artropatia nga LES Artropatia gotoze (podagra) Artropatia e S. S Felty Reumatizmi artikular akut Artriti reumatoid

3 3. Tek artriti reumatoid cili sht deformimi karakteristik i gishtave t duarve? A) B) C) D) E) N ekstension Hiperfleksion i gishtitt mesit Hiperfleksion i gishtit t vogl Devijim n ann radiale Devijim n ann ulnare

34.T ek artroza nodujt e gjetur jan: A) Nodujt e Heberden

B) C) D) E)

Nodujt Osler Nodujt Schmorl Nodujt n nnlkur Nodujt reumatoid

3 5. Cila prej patologjive nuk bn pjes tek spondiloartropatit: A) B) C) D) E) Artriti psoriazik Artriti reaktiv Smundja e Reiter Polimialgj ia reumatike Spondiliti ankilozant

3 6. Cila alternativ rreth ostoporozs nuk sht e vrtet: A) B) C) D) E) Frakturat vertebrale mund t jen asimptomatike Frakturat osteoporotike me dhimbje intense dhe t lokalizuar Rreziku pr fraktura rritet me uljen e densitetit mineral kockor Zona e prekur m shum nga frakturat sht artikulacioni tibio-tasik Kshillohen ushtrimet fizike pr forcimin e kolons vertebrale

3 7. Antitrupat anti-ENA: A) B) C) D) E) Jan markues t smundjes Shfaqen n reumatizmin artikular akut Jan gjithnj t pranishme n nefritin nga lupus Gj enden vetm n raste t rralla t lupusit Nuk jan asnj her t pranishm n konektivitin mi ks

3 8. Cila prej alternativave rreth artritit t podagrs (gotta) nuk sht e vrtet: A) B) C) D) E) Fillimi sht n prgjithsi monoartikular Shfaqjet artikulare radiologj ike nuk j an t tipit eroziv Shifrat e larta t uricemis kan pak vler pr t vendosur diagnozn Terapia hipouricemizante nuk duhet nisur gjat atakut akut Dieta dhe ndryshimi i stilit t jetess mund t j en t dobishme pr t ulur shpeshtsin gotoz dhe evitimit t prdorimit t mj ekimeve

3 9. Tek artriti reumatoid n faz akute nuk sht e kshillueshme:

A) B) C) D) E)

Terapia termale Terapia me medikamente biologj ike Sinoviektomia Terapia imunosupresive Terapia me FANS

4 0. Cila prej shenjave radiologj ike nuk sht karakteristike e osteoporozs vertebrale: A) Hipertransparenc e trupave vertebral dhe rritje e kontrastit t kontureve t strukturs B) Pamje konkave e kontureve t strukturs C) Deformim n formn e pyks e trupave vertebral D) Rritje e spesorit t kortikales E) Rrallim i imazhit trasekular i kocks spongioze

4 1. Cila prej t dhnave laboratorike sht karakteristike pr osteomalacin? A) B) C) D) E) Hiperkale mi H iperfosforemi H iperkaliuri H iposfore m i Rritje t fosfatazs alkaline n gj ak

4 2. Eritema nodoze sht karakteristik e: A) B) C) D) E) Pneumonia streptokoksike Skleroza multiple Sarkoidoza S klerodermia Fruthi

4 3. Lezioni m karakteristik n kocka shkaktuar nga Lupusi sht: A) B) C) D) E) Fraktura spontane Osteoporoza n form pullash O steoskleroza Nekroza aseptike Osteomieliti akut

4 4. Tek polimioziti sht e shpesht t hasen t gjitha alternimet e mposhtme, prve njrs:

A) B) C) D) E)

Rritja e CK Hiposteni/mialgji i muskulaturs s shpatullave Rritje e AST dhe LDH Hepatiti Alternime elektromiografike

4 5. Alternimi okular karakteristik i sindroms s Sj ogren sht: A) B) C) D) E) Katarrakta Rritja e presionit endookular Atrofia e nervit optik Keratokongiuntiviti i that Iridocikliti

4 6. Cili prej alternimeve sht tipik pr artrozn? A) B) C) D) E) Osteoporoza iuxtaartikulare Osteoporoza e prhapur Lacerimi i kapsuls Hollimi i kartilagj enit (krcit) Hiperplazi e sinoviociteve t tipit A

4 7. Cila prej formave t artrozs sht e trashgueshme? A) B) C) D) E) L u m boartroza G onartroza Artroza e brrylit Artroza e shpatulls Artroza nodoze e gishtave

4 8. Glikozaminoklikanet j an: A) B) C) D) E) Substanca patologjike q gj enden n likidin sinovial kur ka artrit septik Prbrs normal t substancs baz t kartilagj enit artikular Prbrs normal t membrans sinoviale Produkte t degradimit t fibrave t kolagj enit Prbrs jo normal t kartilagj enit tek personat me kodrokalinoz

4 9. Sindroma CREST konsiderohet si variant i sklerozs sistemike (sklerodermis) dhe akronimi CREST do t thot: A) B) C) D) E) Kalkuloz renale, Raynaud, endokrinopati, sklerodatili, teleangjektazi Kalinoz e nnlkurs, Raynaud, ezofa gopati, sklerodatili, teleangjektazi Kalkuloz reanle, ezofagopati, endometrioz, sklerodatili, tiroidit Klainoz e nnlkurs, Raynaud, endometrioz, sklerodatili, timom, Asnjra prej prgj igjeve

ENDOKRINOLOGJI 1. Cili hormon ose substanc e ngj ashme me hormonin, prodhohet m shpesh nga karcinoma e p u l m o n it? A) B) C) D) E) ACTH (ose substanc e ngj ashme me ACT H) ADH Gonadotropina korionike Pa rato r moni MSH

2. Struktura kimike baz e hormoneve steroidee derivon nga: A) B) C) D) E) Acidet yndyrore t pangopura Ciklopentanoperidrofenantren D i-naftalene Struktur me katr unaza pirrolesh Acidi arakidonik

3. Komplikime t mundshme t kriptorkidizmit jan: A) B) C) D) E) Hernia inguinale Torsioni i testikulit Neoplazi e testikulit Infertilitet T gj itha prgjigj et

4. Cili prej markuesve t gj akut t orienton drejt diagnozs s karcinoms midulare t tiroides? A) B) C) D) Tireoglobulina Antigj eni karcinoembriogj enetik (CEA) K alcitonina Ferritina

E) Alfa-fetoproteina

5. Feokromocitoma bilaterale sht m e shpesht tek: A) B) C) D) E) Pacientt me MEN2A Pacientt me MEN3 Pacientt me paragangliom Pacientt e gjinis femrore Pacientt duhanpirs

6. Difekti viziv m i shpesht tek pacientt me adenom hipofizare sht: A) B) C) D) E) Hemianopsia bitemporale Hemianopsia nazale K atarrakta S kotomat Ulje t aftsis vizive

7. N dietn e nj pacienti me diabet melitus, sa prqind duhet t j en karbohidrate? A) B) C) D) E) 80% 50-55% 30% 20-25% 10%

8. Hipopituitarizmit total (panhipopituitarizm) tek i riu sht prgj egjs pr t gj itha alternativat e mposhtme, prve njrs: A) B) C) D) E) Hipotiroidiz m Rritje trupore t vonuar Munges t zhvillimit seksual Insufienc t surrenaleve H ipoparatiroidizm

9. Kuadri endokrinologj ik n smundj en e Cushing karakterizohet nga: A) Rritje t sekretimit t kortizolit dhe ACT H B) Rritje t sekretimit t kortizolit dhe aldosteronit C) Rritje t sekretimit t kortizolit dhe ulje t sekretimit t ACTH

D) Rritje t sekretimit t ACTH dhe ulje t sekretimit t kortizolit E) Difekt t steroidogjenezs surrenalike dhe hiperplazi t gj ndrave surrenale

1 0. Sekretimi i kortizolit tek nj person normal: A) B) C) D) E) Stimulon sekretimin e aldosteronit Zhvillon nj qark me feedback negativ n nivelin e hipofizs Zhvillon nj qark me feedback pozitiv n nivelin e hipofizs Zhvillon nj qark me feedback pozitiv n nivelin e surrenaleve Inhibon n mnyr direkte sekretimin e insulins

1 1. Cili trajtim sht m i prshtatshm, n kushtet e hiperkalemis t rnd shkaktuar nga karcinoma e paratiroides, ndrkoh q pritet pr rezeksionin kirurgjikal? A) B) C) D) E) Doz maksimale e diuretikve tiazidik Pamidronat disodium 90 mg n 500 ml solucion fiziologjik pr 4 or Kalcitonina e salmonit 100 U. s/c Hidrokortizon 100 mg iv 2 L solucion fiziologj ik pr perfuzion t shpejt dhe m pas 250 ml/or shoqruar me furosemid

1 2. Cila prej gjendjeve nuk sht karakteristike pr krizn nga tireotoksikoza: A) B) C) D) E) Hipertermi e rnd Prgjigje e mir ndaj dozave masive t steroidve Nse nuk trajtohet mortaliteti sht rreth 70% Takikardi e rnd Tremor

1 3. Cili prej medikamenteve mund t shkaktoj hiperuricemi dhe me shum mundsi nj atak gotoz (podagra)? A) B) C) D) E) Diuretikt tiazidik Penicilina Acidi askorbik Vera p a mil Aspirina

1 4. Heqja kirurgj ikale e paratiroideve shoqrohet me ko mplikimin:

A) B) C) D) E)

Hiperkale mi Tetani H ipofosfate m i Hiperidroksiprolinuri Hipertension arterial

1 5. Cili prej hormoneve nuk prodhohet nga hipofiza? A) B) C) D) E) TSH FSH ACTH GH (hormoni i rritj es) Kalcitonina

1 6. Defiiti i GH (hormoni i rritjes) tek i rrituri shoqrohet me: A) B) C) D) E) Difekt t funksionit sistolik Rritje t libidos (dshirs seksuale) Ulje t kolesterolemis Anoreksi, nauze dhe t vjella Ulje progresive t gjatsis trupore

1 7. Cfar sht FAI (free androgen index)? A) B) C) D) E) Testosteroni total Testosteroni i biodisponueshm Raporti midis testosteronit total dhe SHBG Indeksi i androgj enizimit t indeve Njsia matse e prodhimit t androgj enit

1 8. Gj at terapis s koms ketoacidoze, bikarbonatet j epen n mnyr t ngadalt pr t evituar: A) B) C) D) H ipopotase m in Rritjen e lshimit t O2 nga hemoglobina n inde Uljen paradoksale t pH n likidin cerebrospinal Hipopotasemin e rnd, me ulje t lshimit t O2 nga hemoglobina n inde dhe ulj en paradoksale t pH n likidin cerebrospinal E) Koha e dhnies nuk ka asnj rndsi. E rndsishme sht sasia e bikarbonateve q jepen

19. Cila prej gjendjeve t mposhtme nuk shoqrohet me anemi? A) B) C) D) E) Hipotiroidiz mi Sindroma Cushing Hi pogonadizmi H ipertiroidizmi H iposidere m ia

2 0. Cila prej alternativave nuk sht pjes e kuadrit klinik t smundjes Cushing: A) B) C) D) E) Obezitet central Facies lunaris Hipertension arterial Hiponatr e mi Hiperglicemi

2 1. N trajtimin e hipertiroidizmit mund t nevojiten t gj ith medikamentet e mposhtme, prve njrit? A) B) C) D) E) Digitali B etab llokue s M etimoroli Propiltiuracili Tiroxina

2 2. Me pseudohermafrodizm femror kuptoj m gjendjen e karakterizuar nga: A) Pranin njkohsisht t vezoreve dhe testikujve B) Pranin e gjenitale t jashtme femrore tek persona me gj enotip mashkullor C) Pranin e vezoreve dhe strukturave mulleriane normale, shoqruar me gjenitale t jashtme mashkullore, tek personat me gjenotip femror D) Pranin e gjenitaleve t j ashtme femrore tek personat me testikuj E) Homoseksualitet femror i fshehur

2 3. N trajtimin e nj pacienti diabetik q do ti nnshtrohet nj ndrhyrje kirurgj ikale madhore sht e rndsishme q: A) T mos jepet glukoz e ndoven B) T rregullohet doza e insulins pas operacionit, duke u bazuar tek vlerat e glicemis dhe glikozuris C) Dhnia e t njjts doze t insulins

D) Dhnia e laktatit M/6 E) Mbajtja e pacientit pa ngrn pr 3 dit para operacionit

2 4. Cilat j an shkaqet kryesore t vdekshmris nga diabeti melitus tip 2? A) B) C) D) E) Nefropati a Iktus Pankreatiti N eo pl az it Kardiopatia iskemike

2 5. Cilt jan medikamentet kryesore q prdoren n traj timin e ketoacidozs diabetike? A) B) C) D) E) A ntibiotik Antagonistt e kaliumit B eta-bllokues Hidratimi dhe insulina Diuretikt

2 6. Cili ekzaminim diagnostik bazohet n vetit e qelizs tiroide pr t prqndruar n mnyr aktive j odin? A) B) C) D) E) Dozimi i tireotropins (T SH) Tireoshint igraf is Echocolordoppler i tiroides Dozmi i tiroksins plazmatike t lir (FT4) Dozimi i tireoglobulins

2 7. Hiperparatiroidizmi mund t shkaktohet nga: A) B) C) D) E) Adenoma e izoluar Adenoma multiple Hiperplazi e paratiroideve T gj itha prgjigjet Karcinoma e paratiroideve

2 8. Cila prej endokrinopative nuk sht prgj egjse pr hipertensionin endokrin? A) Morbusi Conn B) Feokromocitoma

C) Hipertiroidizmi D) M. Cushing E) M. Adison

2 9. Cila prej alternativave nuk sht e vrtet pr feokro mocitomn? A) B) C) D) E) Esht tumor tipik i fmijris Esht tumor sporadik ose familjar, por kryesisht i rral Esht tumor jo gjithnj beninj Tumor q nganjher shoqrohet me neoplazi t tjera endokrine Kryesisht prodhon noradrenalin

3 0. Sindroma androgj enitale nga defiiti i 21-hidroksilazs shkakton: A) B) C) D) E) Prodhim t shtuar t 17 alfa-hidroksiprogesteron Redukton prodhimin e delta4- androstenedione Prodhim t shtuar t kortizolit Prodhim t shtuar t aldosteronit Hipotrofi t surrenaleve

3 1. Hipertensioni arterial shkaktuar nga adenoma e surrenales, q prodhon aldosteron, vlersohet m mir me an t: A) B) C) D) E) Dozimi t aldosteronit n gj ak, gj at qndrimit shtrir Dozimit t thjesht t potasemis Dozimit n gj ak t aldosteronit dhe aktivitetit t renins si shtrire dhe n kmb Testi pr supresimin (bllokimin) me desametazon 2 mg pr 2 dit Dozimin e kortikosteronit gjat kateterizimit venoz selektiv

3 2. Cili sht shkaku m i shpesht i hipoglicemis s induktuar? A) B) C) D) E) Terapia me beta-bllokues Prdorimi i alkolit Terapia me sulfanilure Terapia me biguanide Hipoglicemia faktike

3 3. Prania e mikroalbuminuris me 80 mg/24 or tek nj pacient diabetik sht tregues pr: A) Nefropati fillestare

B) C) D) E)

Asnj alternim n nivelin renal Sindroma nefrozike Kalkuloza renale Cistiti hemorragjik

3 4. Cfar sht saktsia e nj dozimi hormonal? A) B) C) D) Prqndrimi minimal i matshm i nj hormoni t caktuar Aftsia e nj antitrupi pr tu lidhur me nj molekul t caktuar Riprodhueshmria e t njjts vler pr nj kampion q prsritet 10 her Diferenca midis prqndrimit t hormonit nga dozimi i gj etur dhe atij q n vrtet sht E) Besueshmria klinike

3 5. Cili sht treguesi m i prshtatshm pr kontrollin metabolik pr nj periudh prej disa muaj sh tek nj pacient diabetik? A) B) C) D) E) Glikozuria e 24 orve Prqndrimi i HbA1c Glicemia mesatare ditore e prsritur do jav Prqndrimi i fruktozamins Prqndrimi i peptidit C bazal dhe pas dhnies s glukagonit

3 6. Obeziteti kontribuon n shfaqjen e diabetit pasi: A) B) C) D) E) Redukton fazn e hershme t sekretimit t insulins pas ngarkess me glukoz Rrit shpejtsin e degradimit t insulins Indet periferike prgjigjen m pak ndaj insulins Kryesisht shoqrohet me marrje t yndyrnave Rrit sekretimin e glukagonit

3 7. Tek nj pacient me ketoacidoz diabetike lkura sht: A) B) C) D) E) E njom Ikterike E zbeht E dehidratuar E skuqur

3 8. TRH prve stimulimit pr t sekretuar TSH, stimolon dhe sekretimin e:

A) B) C) D) E)

ACTH Prolaktina Lipotropina GH MSH

3 9. Cili sht mekanizmi patogj enetik prgj egjs pr diabetin mellitus tip 1? A) B) C) D) E) Infektiv Autoimunitar Degj enerativ N eoplazik Infla m ator

4 0. Cili prej medikamenteve anti-hipertensiv nuk shkakton dizfunksion erektil? A) B) C) D) E) Diuretikt tiazidik Simpatikolitikt qndror B eta-bllokues Spir on ol oa kton i ACE-inibitor

4 1. Cila nga kto substanca ka efekt kryesor reduktimin e insulino-rezistencs? A) B) C) D) E) Sulf an iluree A ntibiotik K emioterapikt Biguanidt Antagonistt e kaliumit

4 2. Testi i etj es tek diabeti insipidus hipofizar shkakton? A) B) C) D) E) Reduktim t poliuris dhe ozmolaritetit urinar me 50% Reduktim t diurezs dhe rritje t ozmlaritetit urinar Rritje t diurezs dhe ozmolaritetit urinar Pa modifikime t diurezs, pa modifikime t ozmolaritetit urinar Reduktim t poliuris dhe rritje t ozmolaritetit urinar me 50%

4 3. Ekzaminimi m i prshtatshm pr vlersimin e funksionalitetit t surrenaleve sht: A) B) C) D) E) Dozimi i kortizolit t lir n urin Dozimi i kortizolemis n orn 8 Test stimulimi me ACTH n doz t ult Matjen e kombinuar midis kortizolit t lir n urin dhe ACTH Ritmin ditor t kortizolit (kampione gj aku t marra nga ora 8.00-16.00)

4 4. N far niveli vepron aldosteroni? A) B) C) D) E) Nivelin kockor Nivelin glomerular Nivelin e tubulit distal t veshks Nivelin e sinapseve neuromus kulare Nivelin e tubuj ve proksimal t veshkave

4 5. Cila alternativ sht e vrtet rreth tiroiditit subakut? A) B) C) D) E) Esht m e shpeshta ndr tiroiditet Ka etiologji virale Ka etiologji autoimune Nuk ka infiltrim autoimun Pj esa m e madhe e pacientve nuk ka prmirsim spontan t funksionit t tiroides

4 6. Pr t diagnostikuar akromegalin, cila prej alternativave t mposhtme sht m e r d s i s hmja? n A) Pamundsia e ngarkess s glukozit pr t reduktuar nivelin e HGH (Human Growth Hormon) B) Rritje t perimetrit t koks C) N Rx-in e koks, ka zgj erim t sels turcike D) Visheromegali (zmadhim t organeve) E) Zmadhim t ekstremiteteve

4 7. Cila prej alternativave rreth diabetit melitus tip 2 nuk sht e vrtet? A) B) C) D) E) Shoqrohet me insulino-rezistenc Shoqrohet me obezitet Nuk ka familj aritet Shoqrohet me hipertension arterial Shoqrohet me retinopati

4 8. Obeziteti, steriliteti, hirsutizmi dhe vezorja policistike karakterizojn: A) B) C) D) E) S. Meniere S. Turner S. Frolich S. Stein-Leventhal S. Klinefelter

4 9. Cila prej alternativave t mposhtme nuk t bn t mendosh pr diagnozn e Sindroms s Zollinger-Ellison: A) B) C) D) E) Pozitivitet t testit respirator ndaj ureazs Niveli i rritur i gastrins n gjak Ulerat multiple gastriointestinale Pozitivitet i testit t stimulimit t sekretins Rritje t sekrecioneve acide gastrike

5 0. Cila prej gj endjeve t mposhtme mund t jet shkak i sindroms nga sekretimi i pamj aftueshm i ADH (SIADH)? A) B) C) D) E) Karcinoma bronkiale Morbusi i Conn H ipertiroidizmi H iperparatiroidiz m i Smundja Cushing

5 1. Cila prej gj endjeve t mposhtme mund t jet shkak i sindroms nga sekretimi i pamj aftueshm i ADH (SIADH)? A) B) C) D) E) Karcinoma bronkiale H ipokalie m ia Tumori i prostats H iperkale m ia Sindroma Cushing

5 2. Ciln prej gjendjeve t mposhtme nuk e shkakton prdorimi kronik i amiodaronit? A) Hipertiroidizmi i komplikuar B) Hipotiroidizmi subklinik

C) Fibroza pulmonare D) H ipogonadiz mi E) Turbullimi i kristalins

5 3. Cili prej parametrave nuk rritet n kushtet e defiitit t rnd t insulins: A) B) C) D) E) Gl ukoza Am in oa ci det Acidet yndyrore Vllimi i urins PH

5 4. Cili prej medikamenteve nuk sht i kshillueshm n mnyr specifike pr trajtimin e diabetit mellitus? A) Insulina B) Biguanidt C) Glitazonikt D) S ulfanilure E) M etim azoli

5 5. Diabeti mellitus tip 2 shkaktohet nga: A) B) C) D) E) Sekretim i pamj aftueshm i insulins + insulinorezistenc Kryesisht rezistenc ndaj insulins Kryesisht defiit t insulins Insulit autoimun Obezitet

5 6. Karcinoma midulare e tiroides bn pjes n ciln prej sindromave? A) B) C) D) E) Sindroma nga neoplazit endokrine multiple (MEN) tip I Poliendokrinopati autoimune tip I MEN tip II MEN tip III MEN tip IV

5 7. Esht e vrtet q statinat: A) Reduktoj n sintezn e VLDL

B) C) D) E)

Rrisin numrin e receptorve pr HDL Inhiboj n n mnyr konkuruese HMG-CoA reduktazn Aktivoj n lipazat lipoproteike Reduktojn metabolizmin hepatik t LDL

5 8. Cila prej alternativave nuk sht karakteristike pr kraniofaringiomn? A) B) C) D) E) Esht tumor cistik i zons selare dhe paraselare Esht tumori m i shpesht i zons hipotalamo-hipofizare n moshat pediatrike Karakterizohet nga prania e kalfikimeve radiologj ikisht t dukshme Esht tumor q merr origjin nga mbetjet embrionale t xhepit t Rathke Shtim t sekretimit t GH

5 9. N karcinomn midulare t tiroides kemi shtim t sekretimit t: A) B) C) D) E) Tireoglobu lin s TSH K alitonins T3 dhe T4 Adrenomedulina

6 0. Sa prqind e noduj ve jo funksionues t tiroides (t ftoht) mund t zhvilloj tumor malinj? A) B) C) D) E) 5-10% 60-70% 21-30% 31-50% >70%

6 1. Cila prej gjendjeve nuk shkakton tetani? A) B) C) D) E) Hipoparatiroidizmi i rnd Litiaza renale nga hiperparatiroidizmi primar Stenoza e pilorit P a r a t i ro d e k t o m i a i Rakitizmi

6 2. Cila simptom sht karakteristike tek nj pacient mashkull me adenom hipofizare q sekreton PRL?

A) B) C) D) E)

Crregullimi i dshirs seksuale hipoaktive (e ulur) Priapiz m H iperidroz Munges t ejakulimit Orkit

6 3. Shkaku m i shpesht i hipotiroidizmit tek i rrituri sht: A) B) C) D) E) Hipotiroidizmi i induktuar nga j odi Tiroiditi kronik autoimun (Hashimoto) Tiroiditi subakut Tiroiditi i Riedel Tumore t tiroides

6 4. N ciln prej gjendjeve t mposhtme nuk jepet nj doz m e lart me levotiroksin n kushtet e hipotiroidizmit: A) B) C) D) E) Trimestri i III i shtatzanis Sulfati ferroz K oles tir a mina Takikardia Rritje n pesh

6 5. Sindroma e Plummer-Vinson karakterizohet nga: A) B) C) D) E) Purpura trombocitopenike Divertikuli i ezofagut me uler peptike Piroz retrosternale me regurgitim acid Anemi, dizfagi dhe glosit Sindrom gastro-ezofageale

6 6. Nj grua 50 vj eare, prej kohsh me tiroiditin kronik t Hashimotos, paraqitet n urgj enc n gj endje kome, pas disa episodesh me t vjella dhe dhimbje abdominale. Esht e dizidratuar dhe lkura e hiperpigmenuar. TA 65/40 mmHg, Fc 100/min. Potasemia 6.6 mEq/L. Diagnoza m e mundshme sht: A) B) C) D) E) Koma ketoacidoze Koma hipoglicemike Kriza hiposurrenalike Kriza nga tireotoksikoza Koma mixedematoze

6 7. Ketoacidoza diabetike: A) B) C) D) E) Shfaqet pothuajse vetm n diabetin tip 1 Esht komplikanc akute e t dy tipeve t diabetit Esht kryesisht si pasoj e insulinorezistencs Shkaktohet kryesisht nga defiiti i bikarbonateve Shfaqet tek podagra (gotta)

6 8. Paratiroidet prodhojn: A) B) C) D) E) Paratormon (PTH) Vitamin D PTH-rp K alitonin Fosfataz alkaline

6 9. Cilat masa terapeutike m t prshtatshme duhen marr n rastin e hipotensionit t rnd pas heqjes s feokromocitoms? A) B) C) D) E) Kortik osteroid M ineralokortikoid Agj ent beta-stimulues Gjak ose plazm Klorur potasiumi

7 0. Cili prej peptideve nuk gj endet n pankreasin endokrin? A) Glukagoni B) Sekretina C) Insulina D) S o m atostatina E) Glukagoni dhe stomatostatina

7 1. Terapia zvendsuese me testosteron n hipogonadizmin mashkullor rekomandohet n rastin e: A) Nivelit t testosteronit total < 300 (10.4 nmol/L) ng/ mL n dy matje t prsritura B) Vlera t testosteronit t lir n limitin inferior t norms n dy matje t prsritura C) Dizfunksioni erektil rezistent ndaj PDE5-inibitor

D) T gj itha prgjigjet E) Shtim t estrogj eneve n cirrozn hepatike

7 2. Mekanizmi patogj enetik prgjegjs pr morbusin Basedow sht me natyr: A) B) C) D) E) Infektive N eoplazike Autoimune Traum atike Asnjra nga prgj igj et

7 3. Sekretimi hormonal jo normal i paratormonit mund t shfaqet tek: A) B) C) D) E) Karcinoma bronkiale me qeliza si kokrra elbi Karcinoma bronkogjene squamoze K oriokarc ino ma E m angioblastoma Fibrosarkoma retroperitonela

7 4. Cili prej tumoreve nuk bn pj es tek MEN tip 1? A) B) C) D) E) M ikroprolaktinoma Tumori karcinoid M elanoma Adenoma e kortikosurrenales Lipoma

7 5. N ciln prej patologjive sht m e shpesht prania e gjinekomastis? A) B) C) D) E) Pankreatiti kronik Bronkiti kronik obstruktiv Glomerulonefriti kronik Retokoliti uleroz Cirroza hepatike

7 6. Diagnoza e hiperparatiroidizmit primitiv bazohet tek: A) PTH i pandryshuar (PTHi) i rritur, ulje t kalemis, rritje t fosforemis B) Rritje t kalemis, rritje t fosforemis, PTHi i pa dozueshm C) PTHi i rritur, kalemi e rritur

D) Rritje t kalemis, rritje t fosfatazs alkaline E) Rritje t kalemis, me zon hiperakumuluese n shintigrafin e kockave

7 7. Forma m e shpesht e hipertiroidizmit tek i moshuari sht: A) B) C) D) E) Adenoma toksike Morbusi i Basedow Hipertiroidizmi i induktuar nga j odi Tiroiditi subakut Struma multinodulare toksike

7 8. Cila prej alternativave rreth hiperkaliemis nuk sht e sakt: A) B) C) D) E) Mund t jet sekondare nga acidoza metabolike Mund t jet sekondare nga mbi prdorimi i furosemidit Mund t jet sekondare nga hemotransfuzionet masive Mund t shkaktoj aritmi t rnda kardiake Hiposurrenalizm kronik

7 9. Kriza hipoglicemike shfaqet m lehtsisht tek nj person: A) B) C) D) E) Obez I dobt Me neoplazi gastrike Alkolisti kronik Kaketik

8 0. Cili nuk ht rol fiziologjik i paratormonit? A) B) C) D) E) Rritja e riabsorbimit t kaliumit e fosforit nga kockat Rritja e konvertimit t kalifediolit n kalitriol Reduktimi i riabsorbimit t fosfateve e bikarbonateve nga veshka Reduktim i riabsorbimit t kaliumit nga veshka Rritje t sekretimit t kaltonins

8 1. N sindromn Klinefelter testikujt jan: A) Me dimensione t reduktuara dhe konsistenc t rritur B) Me dimensione t reduktuara dhe konsistenc normale C) Me dimensione normale dhe konsistenc t rritur

D) Me dimensione normale dhe konsistenc normale E) Me dimensione t rritura dhe konsistenc t rritur

8 2. Tek akromegalia, cili segment skeletik sht m i prekuri nga proliferimi (shtimi) nnperiosteal: A) B) C) D) E) Baini Sternumi Mandibula Femuri Peroniumi

8 3. Cila prej alternativave nuk shkakton shtim t sekretimit t prolaktins? A) B) C) D) E) Thithja e kapecolit t gjirit Stresi Gjumi Aktiviteti fizik Dieta hiperkalorike

8 4. Nuk shkaktohet shtim i sekretimit t sekretins n ciln prej gj endjeve: A) B) C) D) E) Reduktim t vllimit plazmatik Reduktim t Na Variacionet hemodinamike t veshks n lidhje me fluksin intrarenal t gj akut Dhnia e beta-bllokuesve Hi otensioni p

8 5. Cila prej simptomave nuk sht karakteristike e M. Cushing tek nj grua: A) B) C) D) E) Obeziteti android Hipotensioni arterial P oliglobulia H irsutizmi Akne

8 6. Acidoza tek diabeti i pa ko mpensuar shkaktohet nga: A) Rritja e formimit t acidit laktik B) Lidhet direkt me shpejtsin glukoneogj enezs

C) Lidhet direkt me shpejtsin e lipolozs ose t ketogj enezs D) Shkaktohet nga rritja e prqndrimit t acidit karbonik n gj ak E) H ipokaliemia

8 7. Pankreatiti akut mund t shfaqet gj at: A) B) C) D) E) H iperkolesterole m is H ipertrigliceride m is Hiperkilomikronemis Form e kombinuar H iper-HDL

8 8. Morbusi i Addison karakterizohet nga: A) B) C) D) E) Humbja me urinn e joneve t klorit e kaliumit Humbja me urinn e joneve t magnezit e kaliumit Alkaloz metabolike, hiperkolesterolemi Humbja me urinn e j oneve t natriumit Acidoz metabolike

8 9. Forma m e shpesht e hipertiroidizmit tek moshat e reja dhe adulte sht: A) B) C) D) E) Adenoma toksike Morbusi i Basedow Hipertiroidizmi i induktuar nga j odi Tiroiditi subakut Struma multinodulare toksike

9 0. Cila prej alternativave nuk sht karakeristike pr hipoparatiroidizmin: A) B) C) D) E) Dhimbje muskulare Parestezi Bronkospazma/laringospazma Kalfikime t nn lkurs Stipsi (kapsllk)

9 1. Cila prej patologjive nuk shkakton hipogonadizm mashkullor hipergonadotrop? A) S. Klinefelter B) Aplasia e qelizave t Leydig

C) Anorki D) H iperprolaktinemi E) Kriptorkidizm bilateral

9 2. GH stimulon sekretimin e cilit prej hormoneve? A) B) C) D) E) TSH GHRH IGF-1 ACTH GnRH

9 3. Cili prej haplotipeve HLA shoqrohet m shpesh me diabetin tip 1: A) B) C) D) E) DR3 DR4 DR3, DR4 DR2 B8

9 4. Hormonet tiroidee derivoj n nga metabolizmi i: A) B) C) D) E) Tirozins Acidi iopanoik Pentaiodotironina A ce t il ko lina Kolesteroli

9 5. Cilat jan karakteristikat klinike m t shpeshta t akromegalis? A) B) C) D) E) Visheromegali, reduktim t vllimit t gj ymtyrve t siprme, obezitet t trupit Visheromegali, rritje t vllimit t gj ymtyrve, facies luani Poliuri, polidipsi Gjinek oma sti Struma e prhapur

9 6. Terapia e tiroidit subakut t De Quervain bazohet tek: A) Antibiotik B) Tiroksina

C) Kortizonik D) Tireostatik E) S ulfa m idet

97. Diagnoza laboratorike e hipotiroidizmit primitiv bazohet tek: A) B) C) D) E) Niveli i hormoneve tiroidee t lira normal, TSH normal Niveli i hormoneve tiroidee t lira normal, T SH i pa dozueshm Niveli i hormoneve tiroidee t lira i reduktuar, TSH normal Niveli i hormoneve tiroidee t lira reduktuar, TSH i rritur Niveli i hormoneve tiroidee t lira reduktuar, T SH i pa dozueshm

9 8. Cili ekza minim sht i kshillueshm pr diagnozn e hershme t nefropatis diabetike? A) B) C) D) E) Echografia renale A zotemia K reatinine mia Mikroalbuminuria RMN renale

9 9. Cila prej shenjave nuk sht e pranishme n oftalmopatin basedowiane: A) B) C) D) E) Ezoftalmi Diplopia Kemoza Neuriti optik Hemianopsia bitemporale

100. Kortizoli: A) B) C) D) E) Esht hipoglicemizant Esht hiperglicemizant Nuk vepron tek glucidet Stimulon sintezn pankreatike pr insulin Ka funksion t ngjashm me insulinn

101. Tumoret hipofizare jo funksionale paraqesin kuadrin klinik me: A) Rritje t gj ymtyrve t siprme

B) C) D) E)

Rritje t libidos (dshirs seksuale) Cefale dhe reduktim t fushpamj es H iperidroz G jinek oma sti

102. Diabeti melitus tip 1 sht: A) B) C) D) E) Smundje gjenetike Smundje poligjenike, e trashgueshme, q ndikohet dhe nga faktort ambiental Nj variant i MODY (Maturity, Onset Diabetesof the Young) Karakterizohet nga sekretimi me tepric i insulins Prek ekskluzivisht fmijt

You might also like